SlideShare una empresa de Scribd logo
1 de 44
Descargar para leer sin conexión
SEMINARIOS DE FISICOQUIMICA –INGENIERÍA QUÍMICA (PRIMERA PARTE) 1
SEMINARIOS
CATEDRA DE FISICOQUÍMICA
(Primera Parte)
CARRERA: INGENIERIA QUÍMICA
Profesor: Dr. Alberto L. Capparelli
Departamento de Química
Facultad de Ciencias Exactas
UNLP
SEMINARIOS DE FISICOQUIMICA –INGENIERÍA QUÍMICA (PRIMERA PARTE) 2
CATEDRA DE FISICOQUIMICA I SEMINARIO Nro 1
GAS IDEAL Y SU APLICACION A SITUACIONES DE INTERES EN FISICOQUIMICA
PROBLEMA 1:
El modelo del gas ideal se sustenta en las leyes de Boyle y de Gay-Lussac, que surgen del análisis del
comportamiento experimental en condiciones de bajas presiones y altas temperaturas. Analice si a temperatura
ambiente (300K) y 1 atm puede afirmar si los siguientes gases se hallan en lo que usted entiende como “alta o baja
temperatura”. Los gases son He (-267,9 °C), H2 (-239,9 °C), O2 (-118,8 °C), metano (-82,5 °C), CO2 (31,1 °C), SO2
(157,2 °C). Entre paréntesis la temperatura crítica de cada sustancia.
PROBLEMA 2:
A.- DETERMINACIÓN DEL PESO MOLECULAR DE UNA SUSTANCIA EN LA ESCALA DEL GAS
IDEAL. MÉTODO DE DENSIDADES LÍMITES.
Los siguientes datos de presión, volumen y densidad han sido obtenidos para un cierto gas a un temperatura T.
p/atm 0.750000 0.500000 0.250000
Vm/L mol-1
29.8649 44.8090 89.6384
/(gL-1
) 1.07144 0.714110 0.356975
(1) Represente el producto pVm en función de la presión. Teniendo en cuenta que en límite de bajas presiones el
producto pVm tiende a lo esperado por la ley de Boyle, evalúe la temperatura a la que se realizaron las medidas.
(2) Si la ley del gas ideal fuese válida para este sistema, la relación /p o su inversa p/ debería ser constante e
independiente de la presión. Verifique que esta relación no es constante en forma en forma gráfica y analíticamente.
(3) El comportamiento previo se ajusta en este intervalo de presiones a una dependencia lineal y=a + b.x, donde
y=/p por un lado, mientras que x= p. Una dependencia similar se obtiene se toma y= p/ con x=p. En este caso la
dependencia lineal es de la forma y=a’+b’.x, ¿Cómo obtiene de este análisis el peso molecular del gas en la
escala del gas ideal a partir de cualquiera de las dos formas de construir la dependencia lineal? Nota: la
representación /p versus p se denomina método de las densidades límites.
(4) Halle qué relación existe entre a y a’ por un lado y entre b y b’ por el otro en las expresiones estudiadas
previamente.
(5) La dependencia lineal observada es una expresión simplificada de lo que estudiaremos como desarrollos del
virial para gases. A presiones bajas y moderadas, pVm=RT + A2.p+…, En este tipo de desarrollo, A2 se conoce
como segundo coeficiente del virial, y este coeficiente es una función de la temperatura y de la naturaleza del gas.
Evalúe el segundo coeficiente del virial y exprese en términos de los coeficientes del virial que relación existe entre
b por un lado y b’ por el otro en términos de A2.
PROBLEMA 3:
CONSTRUCCIÓN DE UNA ESCALA DE TEMPERATURA EN BASE AL GAS IDEAL.
A.- La dependencia de la presión con la temperatura (C) a volumen constante puede escribirse como p=p0.[1+V.t],
En el caso del volumen medido como función de t (C) a presión constante, V=V0.[1+p.t]. Los coeficientes V y p
se determinan experimentalmente, y se observa que ambos son una función explícita o implícita de la presión. La
representación de estos coeficientes versus la presión muestra experimentalmente que tienden a un único valor
cuando p0, siempre que las determinaciones se realicen en contacto con una única fuente térmica. Si los
experimentos se realizan en contacto con agua en su punto triple como fuente térmica. Explique el procedimiento
que permita asignar al agua en su punto triple una temperatura en la escala del gas ideal.
B.- A presión constante el volumen molar de un gas puede expresarse como donde a es una función de la presión.
Calcule el valor de esta magnitud para el gas ideal empleado los valores de a experimental para nitrógeno e
hidrógeno dados en la siguiente tabla:
gas Nitrógeno Hidrógeno
p/torr 508,2 1095,3 511,4 1105,3
106
p/(°C) 3660,2 3659,0 3667,9 3674,2
PROBLEMA 4:
LEY DE DALTON. PESOS MOLECULARES PROMEDIOS.
La ecuación del gas ideal puede aplicarse en mezclas de gases ideales.
A.- (1) Defina peso molecular promedio de una mezcla <M> (2) calcule cual es el valor de esta magnitud para una
muestra de aire sintético a 25°C y 1 atm. Considere a este sistema como una mezcla de 80 % V/V de N2 y 20 % V/V
de O2.
B.- Una mezcla gaseosa contiene la siguiente composición en peso por cada cien gramos de muestra (% p/p):
Hidrógeno (6%), CO (68%), nitrógeno (11%), dióxido de carbono (14%) y metano. (i) Exprese la composición de
esta mezcla en volumen por ciento (emplee la ley de Amagat) y fracción molar (ii) calcule el peso molecular medio
de la mezcla (iii) la densidad a 400 °C y 1,5 atm.
SEMINARIOS DE FISICOQUIMICA –INGENIERÍA QUÍMICA (PRIMERA PARTE) 3
PROBLEMA 5:
APLICACIÓN DE LA LEY DE DALTON AL ESTUDIO DEL EQUILIBRIO QUÍMICO EN GASES.
A.-En un recipiente se introducen n0 moles de una sustancia gaseosa A2. La presión y la temperatura son constantes
en el experimento. La sustancia se descompone parcialmente según la estequiometria A2(g) 2A(g) hasta alcanzar
el equilibrio. Si se disocian n moles de A2, y si se define el grado de disociación =n/n0, halle las expresiones para
(1) el número de moles totales en equilibrio y el peso molecular promedio. Considere que el peso molecular de A2 es
MA2 (2) las fracciones molares y presiones parciales de los gases en equilibrio (3) la constante de equilibrio en
función de las fracciones molares y de las presiones parciales
B.- Se ha determinado el peso molecular promedio <M> de una muestra de ácido acético a distintas temperaturas y 1
atm de presión. Los resultados se indican en la siguiente tabla:
t/°C 124,8 134,8 144,8 154,6 164,8
<M> 90,35 87,00 81,99 77,73 74,14
El comportamiento de este sistema indica que el ácido acético debe estar asociado en fase gaseosa, de acuerdo con el
equilibrio: 2CH3COOH(g) (CH3COOH)2(g). La dependencia de la constante de equilibrio con la temperatura
está descripto por la ecuación de Van't Hoff, (lnKp/T)p=H
/RT2
, donde H
es la entalpía de reacción en
condiciones de 1 atm de presión para reactivos y productos (1) empleando los datos del enunciado evalúe el grado de
disociación del dímero a las distintas temperaturas (2) calcule la constante de equilibrio a estas temperaturas y evalúe
la entalpía de reacción correspondiente (3) calcule la densidad de la mezcla gaseosa a la temperatura de ebullición
normal del ácido acético (t/°C=118)
C.- En un balón se introduce cloruro de amonio y se calienta hasta la temperatura de 596,9 K. En estas condiciones
la presión es 0,253 atm y la densidad de 0,1373 g/dm3
. Analice en que extensión está disociado el reactivo. Evalúe el
peso molecular promedio de la mezcla.
PROBLEMA 6:
ECUACION DE CLAUSIUS-CLAPEYRON.
A.- Como discutiremos en otros capítulos, la presión de vapor (p) de equilibrio entre un líquido (L) y un vapor (v)
satisface la ecuación de Clapeyron,
m
m
V
T
H
dT
dp


 donde 1
.
2
. m
m
m H
H
H 

 y 1
.
2
. m
m V
V
V 

 son
respectivamente los aumentos de entalpías y volúmenes molares de la sustancia en cada fase. Halle la forma de la
ecuación de Clausius-Clapeyron en condiciones alejadas del punto crítico. Tenga presente que la densidad del vapor
es bastante menor que la del líquido y admita que el vapor se puede describir como si fuese un gas ideal.
B.- La ecuación de Clausius-Clapeyron toma la forma 2
ln
RT
H
dT
p
d mV
V 
 . Integre esta ecuación admitiendo que la
entalpía molar de vaporización no depende de la temperatura. Vincule esta ecuación con la que usted emplea en el
laboratorio sobre presión de vapor de una sustancia pura.
C.- En general, la hipótesis asociada a la independencia de la entalpía de vaporización con la temperatura no es
correcta. Como se discutirá más adelante, una buena aproximación de esta dependencia toma la forma
vHm=C1+Cp,m.T, donde Cp,m es la diferencia de las capacidades caloríficas molares entre las fases gaseosa y
líquida. Introduzca esta expresión para Cp,m en la de Clausius-Clapyron e integre en forma indefinida. Verifique
que bajo esta consideración la presión de vapor del éter etílico obedece a la ecuación: T
R
B
RT
A
C
pV ln
ln 

 .
Encuentre la relación entre los coeficientes de la ecuación integrada y las constantes en la expresión de vHm.
Explique cómo procedería a determinar C1 en la ecuación para vHm.
D.- Se conocen valores de la presión de vapor del éter etílico con la temperatura. Las mismas se presentan en la
siguiente tabla en un amplio intervalo de temperatura.
t/°C -50,9 -10 0 20 34,5 50 70 100 120
pv/torr 8,77 112,3 184,9 439,8 760 1276 2304 4855 7531
(1) Empleando la integral definida entre dos temperaturas consecutivas en esta tabla, verifique que la entalpía de
vaporización no es constante. Represente vHm como función de la temperatura media en cada intervalo. (2) Si se
admite que la presión de vapor se describe con la ecuación T
R
B
RT
A
C
pV ln
ln 

 , evalúe los parámetros A,
B y C a partir de los datos de la tabla trabajando en el intercalo entre 0 y 50 C usando los siguientes procedimientos
(i) resolviendo un sistema de tres ecuaciones con tres incógnitas, (ii) por análisis regresión. (3) Calcule la entalpía de
vaporización a la temperatura de ebullición normal y la entropía de vaporización. (4) Analice si esta sustancia
cumple con la regla de Trouton, según la cual la entropía de vaporización a esa temperatura se halla
aproximadamente entre 84 y 96 J/K.mol (5) analice si las aproximaciones realizadas en la deducción de la ecuación
indicada anteriormente permiten su aplicación en las cercanías del punto crítico. JUSTIFIQUE.
m
m
V
T
H
dT
dp



SEMINARIOS DE FISICOQUIMICA –INGENIERÍA QUÍMICA (PRIMERA PARTE) 4
PROBLEMA 7:
A.- 1 mol de éter etílico se introduce en un recipiente de 10 litros a 10°C en el que previamente se ha practicado
vacío. (1) Halle la presión de vapor del éter etílico a 10 C empleando la información del Problema 6. (2) calcule
cuantos moles de vapor pasan de la fase líquida a la fase vapor cuando se introduce el mol del éter en el recipiente.
Suponga comportamiento ideal del vapor (3) ¿cuál debe ser el volumen del recipiente para que, al incorporar el mol
del éter etílico líquido, éste se vaporice completamente.
B.- Si el volumen del recipiente que contiene el mol de éter etílico fuese de 200 litros a 0°C, (1) ¿cuántas fases tiene
el sistema? (2) En un diagrama P vs log V señale a escala como debe ser el comportamiento del sistema si se
incrementa la presión de a 10 torr. Como información adicional y de utilidad para esta construcción, se conoce que
la densidad del éter líquido a esta temperatura d=0,736 g/ml.
PROBLEMA 8:
A.- Dos balones de igual volumen, en los que se ha practicado vacío, se hallan conectados por un capilar de volumen
despreciable. Inicialmente, se introduce nitrógeno a 200 °C y se lee una presión de 1 atm. Luego, se cierra el sistema
y uno de los extremos se coloca en un recipiente con agua e hielo a 0°C y el otro en agua a 100°C. Calcular la nueva
presión de equilibrio.
B.- Una mezcla de ciclohexeno e hidrógeno, con esta última en exceso se hace reaccionar sobre platino como
catalizador. La mezcla gaseosa, inicialmente ejerce una presión de 500 torr y la que sale del reactor una presión de
450 Torr ambas medidas en las mismas condiciones de volumen y temperatura. Si la reacción es completa, calcular
la fracción molar de los reactivos en la mezcla inicial.
C.-Un compuesto de fórmula C4H4(g) se quema en presencia de oxígeno en exceso, en un reactor a volumen
constante. La presión inicial es de 2 atm, mientras que la presión final es de 1,9 atm. La mezcla con los productos y
el oxigeno en exceso se hace pasar a través de un tren de secado, que retiene el agua. Se mide la presión después del
secado y se lee una presión de 1,8 atm. Con la información suministrada, determinar la cantidad de moles de
reactivos inicial y la temperatura a la cual se investigó esta reacción. La entalpía de vaporización del agua es de 42
kJ/mol. Considere que volumen del reactor es de 1 litro.
PROBLEMA 6:
Diagramas de fases y curvas de Andrews. Considere la sustancia agua. Para este compuesto se dispone de la
siguiente información: A 25°C, la presión de vapor del agua en equilibrio con su líquido es 23,3 Torr. Su
temperatura de ebullición normal es 100°C. Su temperatura, volumen y presión crítica valen 647.4 K, 55,3 cm3
.mol-
1
y 218 atm. Se sabe además que la densidad del agua líquida es aproximadamente 1 g.cm-1
a 25 y que el coeficiente
de expansión cúbico   1x10-4
K-1
y el de compresibilidad   5x10-5
atm-1
. (a) Asimismo, estime la entalpía de
vaporización y calcule la presión de vapor del agua a 150C y 200 C (suponga que el vHm no cambia con la
temperatura). (b) Esquematice en un diagrama p versus volumen el comportamiento de las isotermas que describe
esta sustancia en un amplio intervalo de volúmenes a las temperaturas de 25, 100, temperatura crítica y por encima
de esta temperatura. (IMPORTANTE). Considere que  y  no dependen de p y de T.
BIBLIOGRAFIA:
1.- Cualquiera de los textos generales de Fisicoquímica
2.- H. Avery & D.J. Shaw, Cálculos Básicos en Química Física
3.- H. Avery & D.J. Shaw, Cálculos Superiores en Química Física
4.- A. Woods, Problemas en Química Física
5.- Adamson, Understanding Physical Chemistry. Existe una versión en español.
SEMINARIOS DE FISICOQUIMICA –INGENIERÍA QUÍMICA (PRIMERA PARTE) 5
CATEDRA DE FISICOQUIMICA I SEMINARIO 2
TEORIA CINETICA DEL GAS IDEAL
PROBLEMA 1:
A.- (1) Calcular el número de moléculas por unidad de volumen en un gas a 300 K a las presiones de 1 atm, 100
atm y 0.001 atm (2) ¿cuál es la presión a la que debería hallarse 1 mol de gas a 300 K para que su densidad sea
comparable a la de un líquido (= 1 g, cm-3
) y cuantas moléculas debería encontrar por unidad de volumen?
Considere los gases hidrógeno, oxígeno y metano como ejemplos.
B.- (1) ¿Qué entiende por espacio de velocidades? ¿Cómo lo construye? (2) ¿Qué entiende por moléculas de la
clase c? Emplee para su explicación el espacio auxiliar de velocidades moleculares.
C.- ¿Cuál es la fracción de moléculas, que independientemente de sus velocidades tienen valores de 
comprendidos respectivamente entre 44,5 y 45,5  y de  entre 59,5 y 60,5.
PROBLEMA 2:
A.- (1) Calcular la energía cinética media de traslación de los siguientes gases a 300 K: hidrógeno, oxígeno y
metano (1) calcular la velocidad cuadrática media de cada uno de los gases anteriores.
B.- Calcular las temperaturas a la cual el oxigeno y el metano deben tener la misma velocidad cuadrática media.
C.- La masa molar de un gas vale 60 g.mol-1
. En ciertas condiciones experimentales el número de moléculas por
unidad de volumen es 4x1019
moléculas.cm-3
. Si la velocidad cuadrática media es de 400 m/s, calcular la presión
del gas sobre la base de la expresión deducida a partir de la teoría cinética
D.- ¿Cuál hubiera sido la expresión correspondiente para la presión total en términos de la teoría cinética para un
sistema constituido por una mezcla gaseosa ideal?
PROBLEMA 3:
Cuando una molécula choca elásticamente contra una pared inmóvil, el valor de la componente normal de la
velocidad no varía. Analice la situación para el caso de una pared que se desplaza como consecuencia de las
colisiones moleculares, tal como ocurre con un pistón. Considere por simplicidad un proceso adiabático y
demuestre que la velocidad con que se le entrega trabajo mecánico al émbolo es exactamente igual a la velocidad
de disminución de la energía molecular con la consecuente disminución de la temperatura del gas. (Consultar el
texto de F. Sears, Termodinámica e Introducción a la Teoría Cinética de Gases y Termodinámica Estadística, 1ra
Edición)
PROBLEMA 4:
A.- (1) ¿Cuántos impactos moleculares recibe por segundo 1 cm2
de superficie cuando se lo expone al aire a la
presión de 1 atm a 300 K (2) ¿Cuál será la altura de un cilindro de la misma área que contiene este número de
moléculas a 1 atm y 300 K?
B.- Un recipiente cerrado contiene agua líquida en equilibrio con su vapor a 100 C. En estas condiciones 1 g de
vapor ocupa un volumen aproximado de 1670 cm3
. La entalpia de vaporización del agua a esta temperatura es de
2250 julios/g. (1) ¿cuántas moléculas hay en 1 cm3
de vapor? (2) ¿Cuántas moléculas de vapor alcanzan cada cm2
de superficie de líquido cada segundo? (3) Comparar la energía cinética media de las moléculas en fase gaseosa
con la energía requerida para transferir a esta moléculas desde la fase líquida a la fase vapor? Discutir el resultado.
PROBLEMA 5:
A.- Un recipiente de paredes delgadas de volumen V que se mantiene a temperatura constante, contiene un gas que
escapa lentamente a través de un orificio pequeño de área A. La presión en el exterior es lo suficientemente baja
como para que las moléculas no retornen al recipiente. Hallar una expresión que permita conocer la dependencia de
la presión en el interior del recipiente como función del tiempo.
B.- Un recipiente está dividido en dos compartimientos de volúmenes iguales mediante un tabique de paredes
delgadas. En uno de ellos se hace vacío, mientras que en el otro se llena con un gas hasta la presión P0. Si en un
instante de tiempo que tomamos como el inicial se practica una abertura pequeña de área A, deducir una expresión
que de cuenta de la evolución de la presión en los compartimientos como función del tiempo.
C.- En una línea de vacío, cuyo volumen es 1 litro, la presión se reduce a 0,001 torr. En un instante dado, se
produce una abertura de 10-6
cm2
de área, y comienzan ingresar las moléculas presentes en el aire a temperatura
ambiente. Suponiendo que las paredes son delgadas, hallar una expresión para la dependencia de la presión en el
interior del recipiente como función del tiempo.
D.- En un recipiente de características similares a la del inciso B, se le aísla adiabáticamente, de manera que no se
transfiera calor de un compartimiento al otro. Uno de los compartimientos se mantiene a la temperatura T1 y el otro
a la temperatura T2. ¿Qué relación existe entre las presiones de ambos compartimientos cuando se alcanza el
equilibrio? Discuta el resultado.
SEMINARIOS DE FISICOQUIMICA –INGENIERÍA QUÍMICA (PRIMERA PARTE) 6
PROBLEMA 6: Evaluación de la presión de equilibrio entre una fase condensada y un vapor.
A.- El fenómeno de efusión puede emplearse para medir la presión de vapor de sólidos o de líquidos de alta
temperatura de fusión. Holden et al, determinaron la presión de vapor del Be sólido por este método. El orificio de
efusión fue de 0,318 cm de diámetro y se encontró una pérdida de peso de 9,54 mg en 60,1 minutos de tiempo a
1457 K. Calcule la presión de vapor. (Rpta: 9,55x10-6
atm)
B.- Se tiene un recipiente que contiene una mezcla de H2(g) y de O2(g) en la relación estequiométrica para dar
agua. La presión inicial de la mezcla es de 1 atm a 300 K. El volumen del recipiente es de 1 L. Calcule la
relación de moléculas de H2 a O2 que pueden escapar por efusión sin retornar al sistema al momento de
generarse una abertura de 0,01 cm2
.
C.- Se ha medido la velocidad de efusión (dw/dt=w/t) de Colesterol a distintas temperaturas (V. Oja et al, J.
Chem. Eng. Data 2009, 54, 730–734). La abertura a través de la cuál estas moléculas escapan del sistema en
equilibrio sólido-vapor (sublimación) tiene un diámetro de 0,65 mm. Los valores obtenidos a distintas
temperaturas se muestran en la siguiente tabla y el peso molecular del compuesto es 386,65.
T/K 386,6 392,0 397,3 403,1 408,2 413,8
dw/dt (g.s-1
) 4,302x10-8
7,724x10-8
1,420x10-7
2,548x10-7
4,439x10-7
7,564x10-7
(1) Evaluar la presión de vapor y la entalpía de sublimación (valor de literatura 142.5 kJ.mol-1
) recurriendo
a la ecuación de Clausius-Clapeyron. Recuerde además que la presión de vapor/sublimación en estas
condiciones se expresa como
M
RT
A
t
w
t
A
M
M
RT
RT
w
p
.
2
.
.
.
.
.
8
.
.
4
*




La ecuación así aplicada no tiene en cuenta problemas de diseño experimental que deben ser incluidas en el
cálculo. A los efectos del cálculo de la entalpía de sublimación, este factor no debe influir si no se modifica las
condiciones de trabajo o se cambia el dispositivo con el que se realizan los experimentos.
(2) La ecuación empleada requiere que las moléculas no choquen dentro de la abertura. En la literatura
se han propuesto distintas correcciones para considerar factores geométricos asociados con el diseño del
experimento y de la celda o recipiente de medida. Esta corrección tiene en cuenta estos factores. La relación
entre la presión calculada y la experimental es el factor de Clausing C, de manera que la expresión corregida a
emplear es la siguiente:
M
RT
A
t
w
C
p
.
2
.
*

 . Calcule el factor de Clausing sabiendo que a 397,3 K, la presión
de sublimación del colesterol vale 0,0990 Pa.
PROBLEMA 7:
A.- Diferencie entre la distribución componentes de velocidades y distribución escalar de velocidades o
velocidades moleculares.
B.- Discuta al menos dos métodos experimentales para verificar la ley de distribución de velocidades de Maxwell.
C.- Construir la funciones de distribución p(u), f(c) y de densidad de puntos representativos en el espacio de
velocidades para oxígeno a 100, 300, 500 y 1000 K.
D.- Para el caso de considerar 1 mol de moléculas, calcular (sin resolver integrales) cuántas de estas tienen (1)
componentes de velocidad sobre el eje u entre  y 1,01 (2) ídem entre u y v comprendidas simultáneamente entre
 y 1,01 (3) ídem, entre u, v y w comprendidas simultáneamente entre  y 1,01 . (4) ídem con velocidad c entre
 y 1,01.
PROBLEMA 8:
A.- Calcular la fracción de moléculas que tienen (1) componente sobre el eje u de velocidades entre - y  (2)
superiores a . Usar la tabla de la función error correspondiente.
B.- Calcular la fracción de moléculas que tienen velocidades moleculares menores que  (ii) idem >.
C.- Considere tres sistemas cada de los cuales contiene respectivamente hidrógeno, oxígeno y Hg. Calcule la
fracción de moléculas que a 100, 300, 500 K superan el valor de c0=1000 m/s. Discuta el resultado. Represente el
logaritmo de esta fracción vs 1/T y analice el resultado para los distintos gases.
PROBLEMA 9:
A.- Esquematice la función de distribución de energía para las moléculas de hidrógeno y oxígeno a 300 K.
B.- Represente la función de distribución de energía para las moléculas de oxígeno a 100, 300 y 500 K.
C.- Calcule la fracción de moléculas de (1) hidrógeno, (2) oxígeno y (3) metano que a superan la energía de 4184
J.mol-1
a 100, 300 y 500 K. Discuta los resultados. Represente el logaritmo de esta fracción vs 1/T y analice el
resultado para cada gas.
D.- Calcule la fracción de moléculas que a 300 y 310 K superan (i) la energía de k.300, donde k es la constante de
Boltzmann (ii) ídem de 20.k.300 (el número 300 se refiere a la temperatura en la expresión kT).
SEMINARIOS DE FISICOQUIMICA –INGENIERÍA QUÍMICA (PRIMERA PARTE) 7
PROBLEMA 10:
A.- Enuncie el principio de equipartición de la energía.
B.- Aplique este principio al cálculo de las capacidades caloríficas de (1) un gas monoatómico (2) un gas diatómico
(3) CO2 (4) agua. En el caso de estos dos últimos ejemplos, estime cuantos modos normales de vibración están
activos a 300 K.
C.- Muestre que el principio de equipartición justifica la ley experimental de Dulong y Petit para sólidos
monoatómicos, según la cual, CV,m=3R.
PROBLEMA 11:
El principio de equipartición falla en describir el comportamiento de las sustancias puras a bajas temperaturas.
Empleando los conceptos simples asociados con la cuantización de la energía, explique las razones de esta
discrepancia.
====================================================================
x
TABLA DE LA FUNCION ERROR erf(x) = (1/2)1/2
exp(-x2
).dx
0
x erf(x) x erf(x) x erf(x) x erf(x) x erf(x) x erf(x)
0.0 0.000 0.2 0.2270 0.5 0.5205 0.8 0.7421 1.2 0.9103 1.8 0.9890
0.1 0.1125 0.3 0.3286 0.6 0.6039 0.9 0.7969 1.4 0.9523 2.0 0.9953
0.4 0.4284 0.7 0.6778 1.0 0.8427 1.6 0.9763 3.0 0.9999
SEMINARIOS DE FISICOQUIMICA –INGENIERÍA QUÍMICA (PRIMERA PARTE) 8
CÁTEDRA DE FISICOQUIMICA 1 SEMINARIO 3
GASES REALES
PROBLEMA 1:
A.- a 250 K y 15 atm un gas ocupa un volumen molar 12% menor que el calculado a partir de la ecuación de estado
del gas ideal. Calcular (1) el factor de compresibilidad Z=pVm/RT (2) el volumen molar del gas.¿Qué tipo de fuerzas
predominan en el comportamiento observado del gas?
B.- A 300K y 20 atm, el factor de compresibilidad de un gas Z=0.86. Calcular: (1) el volumen que ocupan 8.2
milimoles del gas (b) un valor aproximado del segundo coeficiente del virial Bp (3) A partir de Bp evalúe los valores
de A2, BV a esta temperatura.
PROBLEMA 2:
A una temperatura dada, el volumen molar ocupado por argón a distintas presiones se da en la siguiente tabla:
p/atm 8,99 17,65 26,01 34,10 41,92 49,5 58,86 64,02
Vm/dm3
mol-1
2,00 1,00 0,667 0,500 0,400 0,333 0,275 0,250
Calcular: (1) la temperatura del gas y el valor Z a las distintas presiones. (2) representar Z vs P y comparar con el
comportamiento ideal. (3) calcular el error relativo que se comete al calcular estos volúmenes con la ecuación del
gas ideal.
PROBLEMA 3:
A.- Para 1 mol de metano a 25 °C, el desarrollo del virial en serie de potencias en 1/V para el factor de
compresibilidad es: Z= 1 - 4.28x10-3
(1/Vm) +2.1x10-3
(1/Vm)2
+1.3x10-5
(1/Vm)3
.
(1) Hallar la expresión de Z serie de potencias de p (2) que error comete al calcular el volumen ocupado por 1 mol de
metano mediante la ecuación del gas ideal a 0.5, 10, 50 y 100 atm. (3) Hallar gráfica y analíticamente los valores de
presión a los cuales Z=1 y Z toma su valor mínimo.
B.- Los coeficientes del virial del metano a distintas temperaturas se presentan en la siguiente tabla
t/°C 0 25 50 100 150 200 250 300 350
Bv10² -5.34 -4.28 -3.42 -2.1 -1.11 -0.42 0.149 0.598 0.97
Cv103
2.39 2.1 2.15 1.834 1.64 1.542 1.42 1.36 1.33
Dv105
26 15 1.3 2.7 3.5 4.3 5.2 5.7 5.9
Cp106
---- 0.45 ? 1.486 ? 1.252 0.993 ? ?
Dp109
30 18 8.2 4.3 2.1 1.03 0.58 0.32 0.17
Las unidades son: Bv/L mol-1
, Cv/L²mol-2
, Dv/L3
mol-3
, Cp/atm-2
, Dp/atm-3
.
(1) Calcular los valores de Bp y Cp correspondientes al desarrollo de Z en potencias de p. Se puede probar (ejercicio
optativo) que Cp=Cv/(RT)2
– (Bv/RT)2
(2) encontrar los coeficientes del desarrollo del producto pVm en serie de
potencias en p y en 1/Vm. (3) determine la temperatura de Boyle de esta sustancia representado Bv vs T. (4)
represente Z vs p a una temperatura inferior, a una igual y a una superior a la de Boyle. (6) encuentre una expresión
que le permita relacionar la densidad del gas real con la correspondiente al gas ideal (7) represente Bv vs 1/T y
discuta el comportamiento observado (8) Muestre por análisis de regresión que un ajuste de Bv/Vc= a + a1/Tr + a2/Tr
2
es también una representación adecuada del comportamiento de Bv, donde Vc es el volumen crítico molar y Tr es la
temperatura reducida T/Tc. Calcule las constantes de esta ecuación (9) Por regresión no lineal, muestre que Bv/Vc= 
+ 1 . exp(3/Tr) representa adecuadamente el comportamiento de este coeficiente. Calcule las constantes de esta
relación (10) Halle una expresión que relacione las constantes k y los parámetros ak de las expresiones planteadas
en los incisos 8 y 9.
PROBLEMA 4:
La densidad del NO a bajas presiones y a 0°C se listan a continuación
P/atm 1.000 0.8000 0.5000 0.3000
/g L 1.3402 1.0719 0.66973 0.40174
(1) Determine el peso molecular del gas por el método de las densidades limites (Represente P/ vs P). (2) Estime el
factor de compresibilidad a 10 atm.
PROBLEMA 5:
A.- En la Tabla 1 que se adjunta, se listan los valores de las constantes críticas de distintos gases. Calcular el factor
de compresibilidad de esas sustancias en el punto crítico. Enuncie la ley de los estados correspondientes.
B.- (1) Sobre el diagrama de compresibilidad generalizado (Figura 1) represente el comportamiento del metano a
distintas temperaturas reducidas. (2) Verifique que a la misma temperatura reducida, el segundo coeficiente del virial
reducido Bp,r es el mismo para los gases que cumplen la ley de los estados correspondientes.
C.- Determine la presión y la temperatura a la cual un mol de (1) NH3 (2) Xe (3) CH4, se comportan en estados
correspondientes a un mol de O2 a 25C y 1 atm.
SEMINARIOS DE FISICOQUIMICA –INGENIERÍA QUÍMICA (PRIMERA PARTE) 9
D.- Verifique que el segundo coeficiente del virial reducido (Bv/Vc) para los gases de la tabla 2 cumplen con la ley
de los estados correspondientes.
PROBLEMA 6:
A.- La ecuación de Van der Waals es una ecuación de estado a 3 constantes:(p+a/Vm²)(Vm-b)=RT por mol de
sustancia. (1) Empleando los datos de la Tabla 1, calcule las constantes a y b (2) Muestre que el modelo no permite
calcular la constante general de los gases. Compare el valor calculado con el valor experimental (3) Como Zc=8/3
para este gas, el Vc no es una magnitud independiente y se puede calcular como Vc=3.RTc/8pc empleando las otras
propiedades criticas. Con esta consideración, reevalúe las constates a y b del modelo, reemplazando Vc por esta
última expresión.
B.- Desarrolle la ecuación de Van der Waals en la forma del virial y encuentre las expresiones de los coeficientes del
virial en P y en 1/V.
C.- Calcular la temperatura de Boyle de estas sustancias y cuando sea posible, compare con valores experimentales.
D.- Represente el comportamiento de Bv en función de T para el metano en términos de este modelo y compare con
los valores experimentales, indicados en el Problema 3.
E.- Escriba la ecuación de Van der Waals en términos de la ley de los estados correspondientes.
PROBLEMA 7:
A.- Estimar el volumen molar del Cl2 a 300 K y 2,3 atm utilizando la ecuación de los gases ideales y la ecuación de
Van der Waals.
B.- Calcular el radio de la molécula de cloro a partir del conocimiento del co-volumen indicado en la tabla adjunta.
C.- Calcular la presión que ejercen 0,8 moles de N2 en una vasija de 100 litros a 25C.
PROBLEMA 8:
OTRAS ECUACIONES DE ESTADO
A.- Halle la expresión para el segundo coeficiente del virial para un gas que obedece la ecuación de estado de
Berthelot. Encuentre para este gas la expresión simplificada de Z en términos de las variables reducidas
B.- Halle la expresión para el segundo coeficiente del virial para un gas que obedece de Dierterici. Muestre que Bv
es el mismo que para el gas de Van der Waals.
C.- La ecuación de Redlich-Kwong es la siguiente; RT
b
V
b
V
V
T
a
p m
m
m
RK





 )
(
)
)
(
(
2
1
con
3
1

c
Z .
Analice si es posible hallar la expresión para el segundo coeficiente del virial para un gas que obedece la ecuación
de Redlich-Kwong. Los coeficientes de esta ecuación valen
c
c
RK
p
T
R
a
2
5
2
4275
,
0 
 y
c
c
p
T
R
b 
 086645
,
0 .
PROBLEMA 9:
Si el comportamiento de un gas real a 2 temperaturas distintas admite los desarrollos para el producto PV:
T1 PV= 10.086 - 0.10 P + 4.0 10-5
P²
T2 PV= 49.99 + 0.0094 P + 6.0 10-5
P²
Estime (1) la temperatura de Boyle. (2) las constantes a y b de Van der Waals (3) su peso molecular, si la densidad
del gas a 1 atm y 0C vale 0.0011g/cm3
.
PROBLEMA 10:
Empleando la ecuación de Rayleigh para el segundo coeficiente del virial Bv = 2  No 

(1-exp(-/kT))r² dr discutida
en la clase teórica, verifique
A.- Que el modelo de Van der Waals se caracteriza por un potencial del tipo
 para r<
(r)=
-A/r6
para r>
(1) Sobre la base de las constantes a y b de Van der Waals, estime las constantes  y A de este modelo para
alguno de los gases indicados en la Tabla 1 y contraste con la información experimental disponible o existente
en tablas (2) Evalúe el potencial de interacción entre las moléculas -A/6
a la distancia de máximo acercamiento
entre las esferas duras- y correlacione el potencial de interacción atractivo a  y la entalpías de vaporización,
temperaturas de ebullición, etc. (3) ídem entre el valor de la constante A y la polarizabilidad molecular.
B.- Para un modelo de potencial (de pozo) del tipo
 para r<
(r)= -u para <r<1,5  con u una constante que represente el valor del potencial en el pozo
0 para r>1,5 
SEMINARIOS DE FISICOQUIMICA –INGENIERÍA QUÍMICA (PRIMERA PARTE) 10
puede obtenerse la siguiente expresión para el segundo coeficiente del virial (expresado como coeficiente del virial
reducido)
Bv/Vc = 0.438 - 0.881 Tc/T - 0.757 Tc²/T²
Represente el comportamiento indicado por esta ecuación y compare con el correspondiente para el metano. Discuta
el significado de estas correlaciones (Compare el comportamiento con el indicado en la Figura 3).
PROBLEMA 11:
Analice el comportamiento de la presión de vapor reducido y describa que debe esperar de la entalpía de vapor de las
sustancias en estado reducido (Figura 4)
==============================TABLAS Y FIGURAS================================
TABLA 1: Parámetros críticos y temperaturas de Boyle de distintos gases
Gas Pc/atm Vc/ml.mol-1
Tc/K TBK Gas Pc/atm Vc/ml.mol-1
Tc/K TBK
He 2.26 57.76 5.21 22.64 Br2 102.0 135.0 584.0
Ne 26.86 41.74 44.44 122.1 HCl 81.50 81.0 324.7
Ar 48.00 75.25 150.7 411.5 BrH 84.0 -- 363.0
Kr 54.27 92.24 209.4 575.0 IH 80.8 -- 423.2
Xe 58.00 118.8 289.8 768.0 CO2 72.85 94.0 304.2 714.8
H2 12.8 64.99 33.23 110.0 H2O 218.3 55.3 647.4
N2 33.54 90.10 126.3 327.2 NH3 111.3 72.5 405.5
O2 50.14 78.00 154.8 405.9 CH4 45.6 98.7 190.6 510.0
F2 55.00 -- 144.0 C2H4 50.50 124.0 283.1
Cl2 76.1 124.0 417.2 C2H6 48.20 148.0 305.4
C6H6 48.6 260.0 562.7
TABLA 2: Segundo coeficiente del virial Bv (ml/mol) para distintos gases y temperaturas
Gas 100 K 200 K 373 K 600 K
He 11.4 12.0 11.3 10.4
Ne -4.8 10.4 12.3 13.8
Ar -187.0 -21.7 -4.2 11.9
Kr -62.9 -28.7 2.0
Xe -153.7 -81.7 -19.6
H2 -2.5 13.7 15.6 --
N2 -160.0 -10.5 6.2 21.7
O2 -197.5 -22.0 -3.7 12.9
CO2 -149.7 -72.2 -12.4
CH4 -53.6 -21.2 8.1
aire -167.3 -13.5 3.4 19.0
TABLA 3: Constantes a y b de Van der Waals
Gas a /atm.L-2
.mol2
b /ml.mol-1
Gas a /atm.L-2
.mol2
b /ml.mol-1
He 0.03412 23.7 CO2 3.592 42.67
Ne 0.2107 17.09 SO2 6.714 56.36
Ar 1.345 32.19 H2O 5.464 30.49
Kr 2.318 39.78 H2S 4.431 42.87
Xe 4.194 51.05 NH3 4.174 37.07
H2 0.2444 26.61 CH4 2.253 42.78
N2 1.390 39.13 C2H4 4.471 57.14
O2 1.360 31.83 C2H6 5.489 63.80
Cl2 6.493 56.22 C6H6 18.00 115.4
CO 1.485 39.85
SEMINARIOS DE FISICOQUIMICA –INGENIERÍA QUÍMICA (PRIMERA PARTE) 11
Figura 1: Diagrama de compresibilidad generalizado (Gouq-Jen Su, Ind. Eng. Chem., 38 (1946), 803)
Figura 2: Estados correspondientes en gases simples (Tomado de E.A. Guggenheim, Termodinámica, Editorial
Tecnos, pg. 177, 1970)
SEMINARIOS DE FISICOQUIMICA –INGENIERÍA QUÍMICA (PRIMERA PARTE) 12
Figura 3: Segundo Coeficiente del virial reducido (Bv/Vc) versus la temperatura reducida (T/Tc). La curva inferior
expande el comportamiento del segundo coeficiente del virial reducido para temperaturas reducidas menores a la
unidad.
Figura 4: Comportamiento de la
Presión de vapor reducida (p*/pc) en
función de la inversa de la temperatura
reducida.
Observar que esta magnitud satisface la
ley de los estados correspondientes.
SEMINARIOS DE FISICOQUIMICA –INGENIERÍA QUÍMICA (PRIMERA PARTE) 13
CATEDRA DE FISICOQUIMICA I SEMINARIO 4
PRIMER PRINCIPIO DE LA TERMODINAMICA
PROBLEMA 1:
CALCULO DE TRABAJO DE EXPANSION EN DISTINTOS SISTEMAS.
A.- Calcule el trabajo ejecutado cuando 50g de Fe se disuelven en HCl en: (1) un recipiente de volumen constante
(2) un recipiente abierto a la presión de 1 atm. Considere que el experimento se realiza a 300 K.
B.- Un mol de CaCO3 se calienta a 700 C. En esas condiciones de temperatura la sustancia se descompone
completamente. La reacción se lleva a cabo en un recipiente provisto de un pistón móvil, inicialmente apoyado
sobre la muestra y sometido a una presión de 1 atm. ¿Cuál es el trabajo ejecutado durante la descomposición?
¿Cuál es el trabajo si el proceso se realiza en un recipiente abierto?
C.- Calcular el trabajo realizado cuando 1 mol de agua se: (1) calienta desde 25 a 55 C a 1 atm de presión (2) se
incrementa la presión desde 1 a 30 atm a 25 C. Los coeficientes de expansión isobárico () y de compresibilidad
isotérmico () valen /K-1
=2.1 x 10-4
y /atm-1
=49.4 x 10-6
respectivamente. Considere la densidad de agua
=1g/cm3
.
D.- Calcular el trabajo de expansión asociado: i) a la evaporación de 1g de agua líquida a 100 C y 1 atm ii) a la
fusión de 1g de hielo a 0 C. La densidad del hielo es =0.9998g/cm3
.
E.- A 300K los coeficientes de expansión isobárico para cobre y para neón valen respectivamente =5.01 x 10-5
K-
1
y =3.3 x 10-3
K-1
. ¿Cuál es el aumento de volumen cuando 50 cm3
de ambas sustancias se calientan en 5 C?
¿Cuál es el trabajo ejecutado si el calentamiento se realiza a 1 atm de presión?
F.- ¿Cuál es el trabajo de expansión que realiza 1 mol de un gas cuyo comportamiento PVT a 0 C puede
expresarse por la ecuación Z = 1 - 21.7 x 10-3
(1/Vm) + 1200 x 10-6
(1/Vm)².
(1) si se somete a una expansión en el
vacío desde un volumen de 5 litros hasta que su volumen se triplica. (2) ídem pero contra un presión externa de 1
atm. (3) se somete a una expansión reversible desde 5 litros hasta que su volumen se triplica.
G.- repita los cálculos del inciso anterior considerando que el gas es ideal.
PROBLEMA 2:
Se desea diferenciar una transformación reversible de otra que no lo es. Para ello se imagina el siguiente proceso.
Un mol de gas ideal se confina a un recipiente cilíndrico de altura suficiente. La tapa superior es un pistón que por
si solo ejerce una presión de 1 atm. Sobre él se colocan 2000 pesitas idénticas, de forma que la presión total,
incluido el pistón, sea de 2 atm. El sistema se halla en contacto con una fuente a 300K y por simplicidad
exteriormente se ha practicado vacío, de manera tal que al retirarse todas las pesas, la presión del gas es de 1 atm.
Con el sistema en esas condiciones se realizan los siguientes procesos -en todos los casos se parte siempre de las
condiciones iníciales- (1) se retiran todas las pesas y se libera el pistón. (2) se retira la mitad de las pesas en una
primera etapa y luego se retiran las pesas restantes. (3) se retiran números idénticos de pesas en cuatro etapas. (4)
se retiran números idénticos de pesas en diez etapas. (5) se retiran las pesas de a una en cada etapa.
Calcular el trabajo ejecutado en cada etapa y representarlo en un diagrama p-V. Discuta los resultados y
compárelos con el trabajo que se hubiera ejecutado en un proceso estrictamente reversible.
PROBLEMA 3:
Considere un sistema similar al señalado en el problema anterior. Una vez que el sistema ha alcanzado el estado
final, se retorna a las condiciones iniciales incorporando las pesas en cantidades similares a las retiradas en cada
etapa. Represente en un diagrama P vs V. Discuta el resultado.
PROBLEMA 4:
El rendimiento en un ciclo de Carnot es independiente de su tamaño y de la naturaleza de la sustancia que se emplea.
Verifique estas dos condiciones considerando que entre 400 y 200K se hace funcionar una máquina de Carnot:
A.- empleando un gas que obedece a la ecuación del gas ideal. (1) Suponga que en la etapa de expansión isotérmica
el volumen se duplica, (2) Suponga que en la expansión isotérmica el volumen se quintuplica.
B.- empleando un gas que obedece a la ecuación del gas de esferas duras o rígidas. Para este gas (dU/dV)T=0.
Suponga condiciones similares a las de los incisos del punto A.
C.- empleando un gas que obedece a la ecuación de Van der Waals. Para este gas (dU/dV)T=a/Vm². Suponga
condiciones similares a las de los incisos del punto A.
PROBLEMA 5:
Un mol de una sustancia se somete a un ciclo de Carnot (a b c d) en la región líquido-vapor tal como
se detalla en la figura adjunta. En el estado -a-, se tiene que la sustancia se halla en estado líquido mientras que en
el estado -b- se ha vaporizado completamente. La presión de vapor es de 3 atm y la temperatura de 400K. La
expansión adiabática se realiza hasta que la presión de vapor se reduce a 1 atm. En el estado -c-, el sistema se halla
constituido por un líquido en equilibrio con su vapor, cuyo título en vapor es 0.97. Si la entalpía de vaporización de
SEMINARIOS DE FISICOQUIMICA –INGENIERÍA QUÍMICA (PRIMERA PARTE) 14
esa sustancia es de 10 Kcal/mol y no se modifica con la temperatura, calcular: (1) el rendimiento de ciclo. (2) el
trabajo ejecutado en las etapas isotérmicas. (3) el trabajo total en las etapas adiabáticas. (4) el trabajo en el ciclo. (5)
el título de vapor en el estado -d-.
PROBLEMA 6:
Calcular el aumento de U y de H que acompaña a la transformación irreversible de Hg sobreenfriado a -45 C en
Hg sólido a esa temperatura y 1 atm de presión. La temperatura de fusión de esta sustancia es -39 C y la entalpía
de fusión de 560 cal/mol. Se conocen las capacidades caloríficas del sólido (CP(s)= 6.4 cal K-1
.mol-1
) y del líquido
(CP(l)= 7.1 - 1.6x10-3
T cal K-1
.mol-1
).
PROBLEMA 7:
A.- un mol de gas ideal se somete a una expansión isotérmica contra una presión externa constante e igual a 1 atm
desde una condición inicial de 10 atm y 300K hasta que su volumen se triplica. Calcular q, w, y los aumentos de U
y H.
B.- ídem para un gas que obedece a la ecuación de estado P(V-b)=RT. (b=0.038 lt/mol)
C.- calcular q, w, y los aumentos de U y H para un gas de Van der Waals, desde un volumen inicial idéntico al
calculado en el punto -a- hasta que su volumen se triplica. Las constantes del modelo a aplicar valen b=0.038
L.mol-1
y a=1.39 L².atm.mol-2
.
PROBLEMA 8:
Analizar (1) la expansión libre del gas ideal. (2) la expansión libre del gas de esferas rígidas. (3) la expansión
isotérmica en el vacio del gas de Van der Waals. (4) la expansión adiabática en el vacío del gas de Van der Waals.
Emplee Cv y Cp correspondiente al gas monoatómico.
PROBLEMA 9:
A.- un mol de gas ideal se somete a una expansión adiabática contra una presión externa constante e igual a 1 atm
desde una condición inicial de 10 atm y 300K hasta que su volumen se triplica. Calcular q, w, y los aumentos de U
y H.
B.- ídem para un gas que obedece la ecuación de estado P(Vm-b)=RT. (b=0.038 l/mol)
C.- calcular q, w, y los aumentos de U y H para un gas de Van der Waals, desde un volumen inicial idéntico al
calculado en el punto -a- hasta que su volumen se triplica. Las constantes del modelo a aplicar valen b=0.038
L.mol-1
y a=1.39 L².atm.mol-2
.
PROBLEMA 10:
La diferencia entre Cp y Cv está dado por la relación Cp-Cv = [ P + (U/V)T]( V/T)P. Sobre la base de esta
ecuación
A.- calcular el coeficiente (U/V)T para Cu(s), sabiendo que Cp-Cv = 0,159 J/mol a 1 atm y 25°C, =4,96 10-6
K-1
y =8,93 g.cm-3
.
B.- estime el trabajo y el aumento de energía interna cuando 1000 g de este material se comprimen aplicando una
presión externa constante de 100 atm a 25°C.
PROBLEMA 11:
a
e
b
c
f
d
P/atm
V
T1=400 K
T2
3
1
SEMINARIOS DE FISICOQUIMICA –INGENIERÍA QUÍMICA (PRIMERA PARTE) 15
A.- Un mol de gas ideal se somete a un proceso reversible no especificado desde sus condiciones iniciales a 0°C y
1 atm. En este proceso, el volumen del sistema se duplica, observándose un H=500 cal/mol, mientras que el calor
puesto en juego es de 400 cal/mol. Calcule la temperatura final, el U y el trabajo realizado por el sistema en este
proceso.
B.- Un mol de un gas ideal se somete a una transformación cíclica, como se indica en la siguiente diagrama P-V.
Calcular, empleando el menor número de relaciones termodinámicas, (1) el trabajo y el calor neto en el ciclo (2) el
H y U entre C y A (3) el calor absorbido al pasar de A  C por el camino ABC (4) ídem ADC.
Observe que H no coincide con los valores de los calores calculados en los incisos (3) y (4) a pesar de que la
presión inicial y final es la misma. Justifique su respuesta.
PROBLEMA 3:
Las entalpías de combustión del grafito y del diamante valen respectivamente –94,030 y –94,484 kcal.mol-1
. Las
densidades son 2,250 y 3,513 g.cm-3
respectivamente. Calcular, U, H, q y w para la transformación:
diamante  grafito a 25°C.
A
B
C
D
3
2
1
P/atm
1 2 3 V/l.mol-1
SEMINARIOS DE FISICOQUIMICA –INGENIERÍA QUÍMICA (PRIMERA PARTE) 16
CATEDRA DE FISICOQUIMICA I SEMINARIO 5
SEGUNDO PRINCIPIO DE LA TERMODINAMICA
PROBLEMA 1:
Trabajando con la información pertinente de la literatura, explique, plantee y/o calcule el aumento de entropía
asociado a los problemas del seminario previo.
PROBLEMA 2:
A.- 1 kg de agua evoluciona según un ciclo de Carnot en la región líquido-vapor, entre 180 y 40°C. De tablas de
vapor de agua disponibles en la literatura se dispone la siguiente información (Los estados que se mencionan
corresponden a los que se muestran en al figura correspondiente del seminario anterior.
estado t/°C p/ kg.cm2
(U-Uo ) kcal/kg (H-Ho) kcal/kg S kcal/K.kg
a 180 10,225 181,9 182,2 0,5107
b 180 10,225 616,7 663,2 1,5721
e 40 0,0752 39,98 39,98 0,1366
f 40 0,0752 580,3 614,7 1,9718
(1) Calcular el rendimiento del ciclo,(2) calcular U, H, q y w en cada etapa del ciclo a  b  c  d, y el
título del vapor en c y d. (3) calcular S del sistema y del exterior en cada etapa del ciclo. (4) Si el vapor,
inicialmente en el estado b sufre una expansión irreversible hasta que la temperatura es 40°C, y se observa que, en
valor absoluto, el título del vapor difiere en un 5% respecto del valor que hubiera observado para el proceso b  c,
calcular el S del sistema, del exterior y del universo en esta expansión adiabática irreversible.
PROBLEMA 3:
A.- Una muestra de Al(s) de 1.75 Kg de masa se enfría reversiblemente a presión constante desde 300K a 265K.
¿Cuál es el aumento de entropía en el sistema?
B.- Un bloque de cobre de 500g de masa inicialmente a 293K está en contacto térmico con un calentador eléctrico
de 1000 ohm de resistencia y masa despreciable. Durante 15 seg circula una corriente eléctrica de 1 A. ¿Cuál es el
cambio de entropía del Cu? CP= 24.4 J/mol.K
C.- Si el bloque anterior se lo sumerge en una corriente de agua que se encuentra a 293K, cual es el cambio de
entropía de (1) el metal. (2) del agua.
PROBLEMA 4:
Analice si es posible que (1) una isoterma y una adiabática pueden interceptarse más que una vez (2) una isoterma y
una adiabática puedan tener más de punto de contacto (3) dos adiabáticas no pueden interceptarse mutuamente.
PROBLEMA 5:
A.- Sea S
la expansión de un gas real referida a las condiciones de gas ideal a la presión de 1 atm y sea S la
entropía experimental a la misma presión.(1) deduzca una expresión que vincule ambas cantidades a la misma
presión y temperatura. (2) aplique la ecuación hallada en el inciso anterior para evaluar la diferencia S
-S para el
gas nitrógeno a su temperatura de ebullición trabajando con la ecuación de Berthelot. Considere que el factor de
compresibilidad a bajas presiones según este modelo de gas puede escribirse como: r
r
r
p
T
T
Z )
6
1
).(
128
1
(
1 2



 .
B.- Calcule la diferencia de entalpía A 25 °C de 1 mol de O2(g) a 1 atm de presión y del mismo gas en las
condiciones ideales. Emplee la ecuación previa.
C.- Calcule la diferencia de CP-CV a 25 °C y a las presiones de 1, 10 y 200 atm para O2. Considere el modelo de gas
de Van der Waals.
PROBLEMA 6:
A.- 10g de hielo a 0 °C se agregan a 25g de agua a 90 °C en un vaso Dewar. El calor de fusión del hielo es de 80
cal/g. Si se desprecia la capacidad calorífica del termo, calcule la temperatura final y los aumentos de entalpía y de
entropía del sistema, fuentes y universo. Considere CP=18 cal/K.mol
B.- Una masa m de agua a la temperatura T1 se mezcla adiabática e isobáricamente con otra masa igual de agua a
T2. Demuestre que S = 2 m CP ln (0.5(T1+T2)/(T1T2)1/2
).
SEMINARIOS DE FISICOQUIMICA –INGENIERÍA QUÍMICA (PRIMERA PARTE) 17
EFECTO JOULE-THOMSON
PROBLEMA 7:
A.- Encuentre una expresión que le permita determinar el coeficiente de Joule-Thomson en términos de los
coeficientes del virial. Demuestre que para un gas real, este coeficiente no es necesariamente nulo a bajas presiones
y aún a p  0. Contraste este comportamiento con un gas ideal.
B.- Encuentre una expresión para el coeficiente de Joule-Thomson en términos de los parámetros a y b del gas de
Van der Waals.
C.- El valor del coeficiente de Joule-Thomson para el amoníaco a 300 °C y 40 atm es de 0.370 K.atm-1.
Compare
este valor con (1) los obtenidos a partir de los siguientes datos P-V-T para el amoníaco de 40 atm. En estas
condiciones Cp,m=46,02 J.K-1
.mol-1
.
t/°C 225 250 275 300 325
Vm/cm3
mol-1
962 1017 1076 1136 1186
(2) con los obtenidos a partir de la ecuación de Van der Waals (Rta ≈0,28 K.atm-1
).
PROBLEMA 8:
A.- La variación del coeficiente de Joule-Thomson para aire a distintas temperaturas y 1 atm de presión está dado
por: (dT/dp)H = J-T = -0.1975 + 138.3 - 319.0/T² en K.atm-1
Mientras que CP/cal/K.mol = 6.5 + 1 10-3
x T. Halle una expresión para (H/P)T como función de la temperatura.
(1) Evalúe las temperaturas de inversión a 1 atm de presión para esta sustancia. (2) Encuentre una expresión que
permita evaluar la curva de temperatura de inversión en términos de los coeficientes del virial.
PROBLEMA 9:
A.- En la figura 1 se presentan curvas isoentálpicas para N2. ¿Cuál es el cambio de temperatura que experimenta el
gas cuando se lo somete al experimento de Joule-Thomson desde 400 atm y 300K hasta una presión de 1 atm.
B.- En la siguiente tabla se presentan las temperaturas de inversión máxima y mínima para N2 a distintas presiones:
p/atm 1 20 60 100 180 220 300 376
Tsup/ °C 348 330 299.6 277.2 235 212.5 158.7 40
Tinf/ °C -- -167 -162.4 -156.5 -134.7 -117.2 -68.7 40
Represente estos datos en un diagrama T-P y compárelos con la curva de inversión predicha sobre la base del
modelo del gas de Van der Waals.
C.- Al diseñar un refrigerador se ha empleado un freón cuyo coeficiente de Joule-Thomson vale 1.2K/atm. ¿Cuál es
la diferencia de presión que debe emplearse en la expansión para que la temperatura del gas refrigerante disminuya
en 5 °C.
PROBLEMA 10:
A.- Un cierto gas sufre una expansión de Joule-Thomson. En el proceso se observa que el trabajo neto es cero.
¿Puede asegurar que el gas se comporta idealmente?
B.- Calcule el aumento de S que experimenta el gas ideal en ese experimento, cuando se lo expande desde 10 atm y
300K hasta que la presión se reduce a 1 atm.
ISOENTALPICAS Y CURVA DE INVERSIÓN
PARA N2. OBSERVE EN ESTE DIAGRAMA
LA POSICIÓN REGIÓN CRÍTICA Y DE
EQUILIBRIO LÍQUIDO-VAPOR
VÁLVULA DE EXTRANGULACIÓN
E INTERCAMBIADOR DE CALOR
EN RÉGIMEN ESTACIONARIO
SEMINARIOS DE FISICOQUIMICA –INGENIERÍA QUÍMICA (PRIMERA PARTE) 18
CONDENSACIÓN DE UN GAS POR MEDIO DEL EFECTO JOULE-THOMSON.
OBSERVE QUE EL MISMO GAS ENFRIADO ACTÚA COMO REFRIGERANTE
DEL GAS QUE INGRESA AL SISTEMA
SEMINARIOS DE FISICOQUIMICA –INGENIERÍA QUÍMICA (PRIMERA PARTE) 19
CÁTEDRA DE FISICOQUIMICA I SEMINARIO 6
ENERGÍA LIBRE
PROBLEMA 1:
A.- (1) Calcule el aumento de energía libre que experimenta 1 mol de gas ideal de 10 a 1 atm de presión a 300K,
cuando se lo somete a la expansión libre. (2) Si el gas desde el estado final descrito en el inciso anterior se
comprime isotérmicamente aplicando una presión externa constante de 10 atm a 300K, ¿Cuál es el aumento de G
que experimenta el sistema? ¿Es éste proceso espontáneo? Justifique su respuesta.
B.- El coeficiente de compresibilidad de un agua líquida vale =5 10-5
atm-1 a 25°C y 1 atm de presión. Calcular el
aumento de energía libre G que acompaña al proceso H2O (líq, 760 torr)  H2O(líq, pv= 23,70 torr) a 25°C.
PROBLEMA 2:
A.- Calcule el aumento de G que acompaña a un cambio de fase reversible.
B.- En las proximidades del punto de transición las presiones de vapor del S(r) y S(m) -rómbico y monoclínico
respectivamente- pueden expresarse como log p(r)/torr = 11.866-5267/T y log p(m)/torr = 11.364-5082/T (1)
Hallar el punto de transición. (2) el aumento de S, H y G a esa temperatura. (3) el aumento de S, H y G a 70C .
C.- La entalpía de fusión del agua vale 80 cal/g y las capacidades caloríficas del líquido y del sólido 1 y 0.5 cal/K
respectivamente. Evalúe los aumentos de S y G para el proceso H2O (l, -5C, 1 atm) a H2O (s, -5C, 1 atm).
PROBLEMA 3:
A.- La entalpía estándar de formación para la reacción 2Hg(l)+Cl2(g)Hg2Cl2 es de -264.6 kJ.mol-1
. La energía
libre de formación es de -210.2 kJ.mol-1
. Evalúe el aumento de entropía que acompaña a esta reacción. La
información se da a 25C y 1 atm de presión.
B.- Para la reacción C6H6(g)+3H2C6H12(g), G
está dado por
G
/kJ.mol-1
=-179.3- 0.375T+ 0.12lnT- 1.05x10-4
T²+ 1.3x10-8
T3
.
Evalúe los aumentos de S, H, y de CP a 400 K.
C.- La FEM de la batería de plomo está dada por E/V = 1.91737 + 56.1x10-3
t + 1.08x10-6
t², con t en C.
Evalúe G
, S
, H
y CP

a 0; 25 y 60 C.
D.- A 25C. la entalpía estándar de combustión de la glucosa es H=-5645 kJ/mol y la correspondiente función de
Gibbs G
=-5797 kJ/mol. Si la temperatura se eleva a 35C, ¿Cuál será el trabajo independiente de volumen w*
por mol de reacción que debería obtenerse?
PROBLEMA 4:
A.- Suponiendo que el agua puede tratarse como un líquido incompresible, estime el aumento de energía libre
cuando sobre 100 cm3
de ese líquido la presión se incrementa de 1 a 100 atm a 25C. ¿Cuál es el error que se
comete al suponer que el volumen no cambia durante el proceso? La densidad del agua y el factor de
compresibilidad isotérmico valen 0.997 g/cm3
y 4.94x10-4
atm respectivamente.
B.- En la transición de aragonita a calcita a 25C y 1 atm, el volumen se incrementa en 2.73 mm3
/mol y la energía
libre estándar disminuye en 0.794 kJ/mol. La entalpía de formación de la aragonita y de la calcita valen (en kJ/mol)
-1207.8 y -1207.6 respectivamente . Calcular (1) el incremento de la función de Helmholtz (A) y el aumento de
entropía. (2) ¿cuál es la presión necesaria para que la aragonita sea fase estable a 25C?
C.- A 25C y 1 atm de presión, el aumento de entropía que acompaña a la transformación diamante  grafito es de
3.26 J.K-1
.mol-1
. Las entalpías de combustión de ambas sustancias pueden extraerse de tablas. Calcular el
incremento en la función de Gibbs y en la función de Helmholtz. ¿Cuál es la forma estable en esas condiciones?
D.- Calcular el incremento de energía libre, cuando 1 mol de agua es trasladado por destilación isotérmica a 25C,
desde agua pura a una disolución al 9.9% de glicerina en agua, cuya presión de vapor de agua a esa temperatura es
de 23.30 torr. La presión de vapor de agua pura es de 23.70 torr.
PROBLEMA 5:
FUGACIDAD:
En estos ejercicios emplee métodos de regresión no lineal y/o el método de Simpson para resolver integrales
numéricamente.
A.- La fugacidad del oxígeno a 200K puede determinarse a partir del conocimiento del factor de compresibilidad Z
a distintas presiones:
p/atm 1 4 7 10 40 70 100
Z 0.99701 0.98796 0,97880 0.96956 0.8734 0,7764 0,6871
B.- El factor de compresibilidad del N2 a 0C a distintas presiones es de
p/atm 50 100 200 400 800 1000
Z 0.9846 0.9846 1.0365 1.2557 1.7959 2.0641
SEMINARIOS DE FISICOQUIMICA –INGENIERÍA QUÍMICA (PRIMERA PARTE) 20
Con esta información (1) representar la fugacidad vs p. (2) representar el coeficiente f/p vs p. (3) calcular el
aumento de energía libre, cuando 1 mol de N2 se comprime de 1 a 1000 atm. ¿Cuál hubiera sido el error relativo en
G, si se admite comportamiento ideal de la sustancia?
C.- (1) enuncie la regla de Lewis para estimar la fugacidad de un componente gaseoso en una mezcla. (2) Merz y
Whitaker (J.Am.Chem.Soc. 1925, 50, 1552) midieron a distintas presiones, el volumen molar parcial de hidrógeno
a 0C en mezclas de este gas con nitrógeno. Los valores son:
p/atm 50 100 200 300 400
Vm/ cm3
.mol-1
466.4 241.3 129.1 91.1 72.5
Calcule la fugacidad del hidrógeno a esa temperatura y 400 atm si estos datos corresponden a una mezcla de
fracción molar 0,60 en H2.
PROBLEMA 6:
A.- Calcular la fugacidad del nitrógeno admitiendo que el gas cumple con (1) la ecuación de Berthelot. (2) de Van
der Waals. La información que requiere la puede extraer de tablas.
B.- Para un gas a 0C, PVm=RT(1.- 9.93x10-4
P + 2.19x10-6
P².). Calcular la fugacidad a 1, 10 y 50 atm y la presión
a la que el volumen ideal coincide con el volumen real. Calcule además la energía libre del gas real comparada con
la del gas ideal. Determine la presión a la cual estas magnitudes son iguales.
C.- La presión de vapor del cloro líquido es de 3.66 atm a 0C y su volumen molar de 6.01 dm3
mol-1
. Calcular su
fugacidad.
D.- Calcular la fugacidad del vapor de agua a 25C, si su presión de vapor y su volumen molar son de 23.76 torr y
43.4 dm3
g-1
.
SEMINARIOS DE FISICOQUIMICA –INGENIERÍA QUÍMICA (PRIMERA PARTE) 21
CÁTEDRA DE FISICOQUÍMICA I SEMINARIO 7
PROPIEDADES MOLARES PARCIALES Y PROPIEDADES COLIGATIVAS
PROBLEMA 1:
A.- Defina qué entiende por propiedad molar parcial (F) y propiedad molar aparente (F).
B.- Para un sistema de dos componentes, F = a + b m + c m2
(m molalidad). Encuentre las expresiones
correspondientes para las propiedades molares parciales de ambos componentes.
C.- Para soluciones acuosas de electrolitos 1:1 valentes, la propiedad molar aparente F se expresa mas
convenientemente en función de la raíz cuadrada de la molalidad F = a + b m1/2
. Hallar las propiedades
molares parciales del solvente y soluto.
PROBLEMA 2:
A.- El volumen de una solución acuosa de NaCl a 25º
C se expresa como una función de la molalidad según:
V/ml mol-1
= 1002,9 + 16,40 m + 2,5 m2
- 1,2 m3
. Si el volumen molar del agua pura es 18,069 cm3
.mol-1
, hallar
las expresiones correspondientes para el volumen molar parcial y el volumen molar aparente (1) del NaCl (2)
agua.
B.- Los siguientes datos corresponden a los volúmenes de soluciones de NaCl en agua a 25º
C a distintas
molalidades.
m/mol kg-1
0 0,255 0,511 1,03 2,56 5,14
V/cm3
mol-1
1002,96 1007,45 1012,22 1022,11 1053,81 1112,36
Evalúe los volúmenes parciales del solvente y del soluto.
C.- Cuando 1 mol de MgSO4 se disuelve en un gran volumen de agua, el volumen total disminuye en 1,4
cm3
/mol. ¿Cuál es el volumen molar parcial del soluto? Justifique el resultado.
D.- Para evaluar propiedades molares parciales, puede recurrirse al denominado método de las ordenadas al
origen. (1) Explique cómo se procede a su aplicación- ver por ejemplo el texto de Atkins, 3ra Edición- (2)
Recurra a este método para tratar los siguientes datos que corresponden a las densidades de HNO3 en agua:
100 w 2,162 10,98 20,8 30,0 39,2 51,68 62,6 71,57 82,33 93,4 99,6
 1,01 1,06 1,12 1,18 1,24 1,32 1,38 1,42 1,46 1,49 1,51
En la tabla w se da en fracción en peso de HNO3 y la densidad en g/cm3
.
PROBLEMA 3:
A.- La capacidad calorífica molar aparente de sacarosa está dada por:
a (Cp) = 152,5 + 1,13 m - 0,0466 m2
cal / k mol
Encuentre las expresiones para las capacidades molares parciales. La capacidad calorífica molar del agua es de
18,02 cal.K-1
mol-1
.
B.- Para la disolución de NaCl en 1000 g de agua para dar soluciones de molalidad m, se midió la entalpía de
solución (en cal/mol) a 18º
C:
m 1,0 0,64 0,36 0,25 0,09 0,04 0,01
H 1067 1141 1197 1212 1226 1220 1202
Calcule (1) la entalpía de solución de 1 mol de NaCl en un volumen muy grande de agua; (2) la entalpía molar
parcial del NaCl para dar soluciones 0,04 y 0,25 molal.
PROBLEMA 4:
En la siguiente tabla se presentan valores asociados al proceso de disolución de una solución acuosa 0,1 M de
NaCl. El cambio de entalpía correspondiente al proceso de agregar la cantidad suficiente de agua para diluir la
solución desde una concentración 0,1 M hasta un valor c, se indica en esta tabla como H0,1c y son cambios de
entalpía por mol de soluto. Evaluar el aumento de entalpía correspondiente al proceso de dilución infinita Hc0
de una solución de concentración c que contiene n2 moles de NaCl.
c/ M 0,05 0,025 0,0125 0,00625 0,00305 0,00153 0,00076 0,00039
H0,1c/cal.mol-1 12,8 27,8 42,9 52,9 61,5 67,4 73,1 75,7
Evalúe las entalpías molares parciales correspondientes al solvente y soluto.
PROBLEMA 5:
A.- Que entiende por potencial químico? Defina esta magnitud en términos de G, A, H, U y S, especificando
claramente cuáles son las variables que se deben fijar para que estas magnitudes puedan ser empleadas como
definiciones alternativas del potencial químico.
B.- Demuestre que al formar una solución ideal a partir de dos componentes líquidos, no deben observarse
cambios de volumen, de energía interna ni de entalpía;
SEMINARIOS DE FISICOQUIMICA –INGENIERÍA QUÍMICA (PRIMERA PARTE) 22
C.- Evalúe el aumento de entropía asociada a la mezcla de componentes líquidos en cantidades n1, n2, .., ni,..
para dar una solución ideal. Exprese este resultado por mol de mezcla y demuestre que siempre es positivo.
PROBLEMA 6:
A.- Un teorema importante del análisis matemático indica que una función f(x,y,z) es una función homogénea
de orden m, si f(x,y,z)= m
(f/x) + m
(f/y) + m
(f/z). Si m=1, la función se dice homogénea de
orden 1. La energía libre G de un sistema G(n1, n2, ..., ni, ..) a T y P constantes es una función homogénea del
número de moles de orden 1. Luego, G =  ni.i. Analice si este teorema asegura que, a relación de moles
constantes, el valor de cada i es independiente del tamaño del sistema. (Lea un texto de análisis matemático)
B.- Demuestre que i = Hi – T.Si, donde las magnitudes presentes en el segundo miembro de esta ecuación
representan la entalpía y la entropía molar parcial del i-ésimo componente.
SEMINARIOS DE FISICOQUIMICA –INGENIERÍA QUÍMICA (PRIMERA PARTE) 23
CATEDRA DE FISICOQUIMICA SEMINARIO 8
SOLUCIONES IDEALES-PROPIEDADES COLIGATIVAS
PROBLEMA 1:
A.- Las mezclas de benceno y tolueno se comportan casi idealmente. A 30º
C las presiones de vapor de benceno
y tolueno puros son 118,2 y 35,7 Torr respectivamente. Determine la presión parcial y la composición en
equilibrio con una solución que consiste de 100 g de ambas sustancias.
B.- Represente la variación de la presión de vapor de cada uno de los constituyentes de esta mezcla y de la
presión total como función de la composición.
C.- Si admite que ambas sustancias cumplen la regla de Trouton reconstruya el gráfico de temperatura de
ebullición-composición. Las temperaturas de ebullición normales son 80,2º
C y 110,6º
C para benceno y tolueno
respectivamente.
PROBLEMA 2:
A.- Las presiones de vapor de metano líquido a 173,2 y 153,2 K valen 25,7 y 11,84 atm respectivamente. Estime
la solubilidad ideal del metano en cualquier solvente a 25º
C.
B.- A 300 K las presiones de vapor de equilibrio de HCl(g) sobre una solución muy diluída de GeCl4 (l) son:
x HCl 0,005 0,012 0,019
p / kPa 32,0 76,9 121,8
Analice si este sistema obedece la ley de Henry en ese rango de concentraciones y estime la constante de
Henry.
C.- (1) Cuál es su significado físico que le asigna a KH? (2) La dependencia con la temperatura de la constante
de Henry para el sistema anterior está dada por:
ln KH / kPa = 12,147 - 1010 / T (K)
¿Qué información puede obtenerse a partir de esta ecuación?
D.- Una planta para carbonatar agua de uso doméstico opera con CO2 a 10 atm. Estime la composición de la
soda que se produce. ¿Qué información requiere para este cálculo? Busque esta información en tablas.
E.- Las presiones de vapor de cloruro de metilo en equilibrio con una solución acuosa de esta sustancia a la
temperatura de 25º
C son:
m/mol kg-1
0,029 0,051 0,106 0,131
p/torr 205,2 363,2 756,1 945,9
Calcule la constante de Henry, explicando el procedimiento.
PROBLEMA 3:
A.- La presión de vapor de una muestra de 300 g de benceno es de 400 torr a 60,6º
C pero desciende a 386 torr
cuando se le adicionan 19 g de un compuesto orgánico no volátil. Calcular su peso molecular.
B.- ¿Cuál es el punto de congelación de 250 cm3
de agua azucarada que contiene 7,5 g de sacarosa (C12H22O11)
aproximadamente 5 cucharadas de azúcar?
C.- Estime el punto de congelación de 100 g de agua que contienen 2 g de NaCl.
D.- Al estudiar soluciones acuosas de Th(NO3)4 se observó un descenso de la temperatura de fusión de 0,07031
para una concentración 0,009 molal. ¿Cuál es el número aparente de iones por moléculas de soluto?
Para estos cálculos considere kc = 1,86 K/mol kg-1
.
PROBLEMA 4:
A.- Las temperatura de fusión y entalpía de fusión del Pb son 327º
C y 5,2 kJ/mol respectivamente. Calcular la
solubilidad ideal del Pb en Bi a 298º
C.
B.- La entalpía molar del naftaleno está dada por la expresión: H(T) = Ho
+ 4,8 T. Su temperatura de fusión
es 80,2º
C y su entalpía de fusión a esa temperatura es 4540 cal/mol. (1) Determine la solubilidad ideal de
naftaleno en p-diclorobenceno (tf = 53,2º
C) a 50º
C. Este sistema presenta un eutéctico simple a 30,2º
C y su
composición es 0,394 en fracción molar de naftaleno. (2) Reconstruya el diagrama correspondiente y aplique la
regla de las fases a cada situación de equilibrio de fases.
PROBLEMA 5:
A.- Se midió la presión osmótica de soluciones de polietileno en tolueno. La presión se expresa en términos de
la altura del solvente. A 25º
C se obtuvieron los siguientes resultados:
c / g l-1
2,042 6,613 9,521 12,602
h / cm 0,592 1,910 2,750 3,600
Calcular la masa molecular relativa del polímero (densidad del tolueno 0,867 g cm-3
, densidad de las soluciones
1,004 g cm-3
).
SEMINARIOS DE FISICOQUIMICA –INGENIERÍA QUÍMICA (PRIMERA PARTE) 24
B.- La masa molecular relativa de una nueva enzima fue determinada disolviéndola en agua midiendo su presión
osmótica en diferentes soluciones, extrapolando hasta concentración cero. Se obtuvieron los siguientes
resultados:
c / mg cm-3
3,221 4,618 5,112 6,722
h / cm 5,746 8,238 9,119 11, 9
¿Cuál es la masa molecular de la enzima?
SEMINARIOS DE FISICOQUIMICA –INGENIERÍA QUÍMICA (PRIMERA PARTE) 25
CÁTEDRA DE FISICOQUIMICA SEMINARIO 9
SOLUCIONES REALES-ACTIVIDAD
PROBLEMA 1:
En la siguiente tabla se dan las presiones de vapor (en KPa) de equilibrio y las composiciones del vapor y del
liquido (en fracción molar) para el sistema tolueno (A)-metiletilacetona (B) a 303.13K
xA 0.000 0.0898 0.2476 0.3577 0.5194 0.6036 0.7188 0.8019 0.9105 1.000
yA 0.000 0.0410 0.1154 0.1762 0.2772 0.3393 0.4450 0.5435 0.7284 1.000
pT/kPa 36.066 34.121 30.900 28.626 25.239 23.402 20.698 18.592 15.496 12.295
Considere el vapor como gas ideal. Calcule: (1) La presión parcial de cada componente en la mezcla y represéntela
gráficamente. Estime la constante de Henry para cada componente. (2) Calcule el factor de actividad adoptando
como estados de referencia (a) la ley de Raoult para cada componente (b) la ley de Raoult para el solvente y de
Henry para el soluto. (3) Represente los potenciales químicos correspondientes y discuta el comportamiento de las
pendientes en los extremos y en x=0.5. Analice las mismas sobre la base de la relación de Gibbs-Duhem.
PROBLEMA 3:
Para soluciones de n-propanol en agua se obtuvieron los siguientes valores de las presiones parciales de cada
componente
mol%(n-prop) 0 9.04 16.97 30.47 41.14 55.5 73.9 82.0 100
p agua/Torr 92 87.2 87.9 84.5 83.0 78.2 60.9 49.2 0.0
p n-prop/Torr 0 49.2 51.7 54.6 57.4 60.2 68.4 72.1 90
Calcule la actividad de cada componente empleando el estado de referencia (1) solvente o ley de Raoult para cada
uno (2) solvente para el agua y soluto para el n-propanol. Estime la KH de n-propanol.
PROBLEMA 4:
Lewis y Storch midieron la presión de vapor de Br2 sobre CCl4 a muy bajas concentraciones. Los datos
experimentales obtenidos a 25C fueron:
103
xBr2 3.94 4.20 5.99 10.2 13.0 23.6 23.8 25.0
pBr2/Torr 1.52 1.60 2.39 4.27 5.43 9.57 9.87 10.22
(1) Adoptando como estado de referencia la solución ideal diluida (ley de Henry), calcular la presión de vapor que
le hubiera correspondido al bromo puro si la ley de Henry se hubiera cumplido en todo el intervalo de
composiciones. (2) La presión de vapor del bromo puro a esa temperatura es 213 torr. ¿Cual es el carácter de la
desviación del sistema respecto a la ley de Raoult? (3) Calcule la actividad del bromo puro tomando como estado
de referencia (a) la ley de Raoult (b)la ley de Henry. Discuta los resultados.
PROBLEMA 5:
A.- (1) Las medidas de presión de vapor de Hg en amalgamas de talio a 325C pueden representarse por
log p1/(p1
*
x1)= -0.096/(1+0.263x1/x2)2
,
donde p1 y p1
*
son las presiones de vapor del Hg en las amalgamas y del Hg puro a esa temperatura, x1 es la
fracción molar del Hg. Calcular la actividad del talio en estas soluciones si se elige como estado de referencia i) la
solución diluida ii) la ley de Raoult. Discuta las diferencias. (2) Verificar que la variación de energía libre al diluir
una amalgama de talio de 0.5 a 0.2 en talio es independiente del estado de referencia elegido.
B.- A partir de medidas de FEM se ha determinado la actividad de Tl en amalgamas de Hg a 20C. En la siguiente
tabla se presentan los resultados obtenidos para los factores de actividad del talio como soluto
x2 0.0 0.005 0.01 0.05 0.1 0.2 0.3 0.4 0.5
f2 1.0 1.06 1.15 1.8 2.48 4.98 6.6 7.57 7.98
Calcular la actividad del Tl y del Hg a diferentes concentraciones.
PROBLEMA 6:
A.- Las presiones de vapor del metano liquido a 173.2 y 153.2K valen 25.7 y 11.84 atm respectivamente. Estime la
solubilidad ideal del metano en cualquier solvente a 25C.
B.- A 300K las presiones de vapor de equilibrio de HCl(g)sobre una solución muy diluida de GeCl4(l) son:
xHCl 0.005 0.012 0.019
p/kPa 32.0 76.9 121.8
Muestre que este sistema obedece la ley de Henry en ese rango de concentraciones y calcule la constante de Henry.
C.- Una planta para carbonatar agua de uso doméstico opera con CO2 a 10 atm. Estime la composición de la soda
que se produce. ¿Que información requiere para este cálculo?
D.- Las presiones de vapor de cloruro de metilo en equilibrio con una solución acuosa de esta sustancia a la
temperatura de 25C son
SEMINARIOS DE FISICOQUIMICA –INGENIERÍA QUÍMICA (PRIMERA PARTE) 26
m/mol kg-1
0.029 0.051 0.106 0.131
p/Torr 205.2 363.2 756.1 945.9
Calcule la constante de Henry.
PROBLEMA 7: Determinación de los factores de actividad (El sistema es exactamente el estudiado en el
laboratorio). Para mezclas de acetona- benceno se obtuvieron los siguientes datos de presión de vapor en
función de la composición en acetona:
xlíquido xvapor Ptotal(mmHg) xlíquido xvapor P(mmHg) xlíquido xvapor P(mmHg)
0.0 0.0000 95.3 0.4 0.6210 178.7 0.8 0.8640 213.1
0.1 0.3232 131.3 0.5 0.6810 188.3 0.9 0.9307 220.3
0.2 0.4661 152.5 0.6 0.7380 197.1 1.0 1.0000 229.2
0.3 0.5531 167.3 0.7 0.8007 205.3
A.- Construya el diagrama presión de vapor de acetona y de benceno como función de la fracción molar e
indique que tipo de desviaciones presenta respecto de la ley de Raoult
B.- Estimar el valor de la constante de Henry para ambos componentes usando la información de las soluciones
0,1 en acetona y 0,1 en benceno.
C.- Los coeficientes de actividad para ambos componentes en las soluciones de xacetona = 1, 0,2; 0,4 ; 0,6; 0,8 y 1
(1) considerando el estado de referencia solvente (ley de Raoult) para ambos componentes (2) el estado de
referencia solvente para la acetona y la solución real diluida como estado de referencia (ley de Henry) para el
benceno (3) el estado de referencia solvente para el benceno y la solución real diluida como estado de referencia
(ley de Henry) para la acetona.
D.- representar el potencial químico (a menos de una constante) para ambos componentes
E.- verificar que el potencial de la acetona pura es independiente del estado de referencia elegido
F.- haciendo uso de la regla de Trouton y del conocimiento que la temperatura de ebullición de la acetona es
56,5°, estime su constante de Henry y el factor de actividad correspondiente de la acetona pura a esta
temperatura (56,5°).
PROBLEMA 8:
Modelo de Van Laar.
El modelo de Van Laar para soluciones regulares muestra que el factor de actividad puede representarse como
ln k =  (1-xk)²,
donde  es un parámetro a ser evaluado a partir del análisis de los comportamientos límites de la presión de
equilibrio (ver S. Glasstone, Tratado de Fisicoquímica o Termodinámica para Químicos para el tratamiento
general. Se conoce también como ecuación de Margulles).
(1) Halle las expresiones para las pendientes de las presiones totales para xi=0 y xi=1,
(2) Establezca el vínculo entre estas pendientes con las constantes de Henry y las presiones de vapor.
(3) Empleando la información del problema 1 del presente seminario, estime el valor del parámetro ß promedio.
PROBLEMA 9: El modelo de van Laar (soluciones regulares) expresa el coeficiente de actividad de cada
componente en una mezcla de moléculas de no-electrolitos de igual volumen molecular, en términos de las
energías de interacción entre las distintas moléculas. En este modelo, siendo una hipótesis importante, se admite
que la entropía por mol de mezcla en la solución real es la misma que la que debiera medirse al formar una
solución ideal. Bajo estas condiciones, el potencial químico de cada componente se expresa como: 1 = 1
*
+ kT
ln x1 + x2
2
u0
. Los distintos componentes se representan por los subíndices 1 y 2. El término energético u0
=
1,2 -1/2 (1,1 + 2,2 ) mide la extensión de la desviación del comportamiento predicho para una solución ideal en
términos de la ley de Raoult. En esta igualdad, 1,2 , 1,1 y 2,2 son las energías de interacción de pares de
moléculas; del tipo 1-2, 1-1 y 2-2. Una expresión similar vale para el componente 2. Considerando que el
componente 1 está en equilibrio con su vapor y que éste se comporta como ideal:
1. Encuentre una expresión para la presión de vapor en función de la composición.
2. Grafique la presión de vapor en función de la composición para  = u0
/kT = -3, 0, 1, 3.
3. Analice que debe esperarse para las desviaciones cuando  > 2. Aplicar al sistema fenol-agua.
4. Represente en este caso, en forma cualitativa el comportamiento del potencial químico de uno de los
componentes y verifique que existe una región donde el sistema no puede existir en dos fases en equilibrio
(un líquido y un vapor), y que esta situación conduce a la separación de dos fases líquidas parcialmente
miscibles en equilibrio con un mismo vapor.
SEMINARIOS DE FISICOQUIMICA –INGENIERÍA QUÍMICA (PRIMERA PARTE) 27
PROBLEMA 11: Emplee la información de los problemas 7 y 9.
1.- Si admite como válida la aproximación de Van Laar, pk=xk.pk*.exp[(1-xk)2
], estime el parámetro  y las
constantes de Henry para estas sustancias a esta temperatura (25C) para una solución x =0,5.
2.- Reconstruya el gráfico de temperaturas de ebullición normales-composición para este sistema. La temperatura
de ebullición de B es 79.6C.
3.- Encuentre la expresión entre la entalpía del líquido puro y la entalpía del líquido en el estado hipotético que se
obtiene por extrapolación de la ley de Henry hasta xk=1. Analice como deben ser estas diferencias de entalpía para
desviaciones positivas y negativas. Considere ideal al vapor y que el volumen de un líquido no se modifica
apreciablemente ante cambios de presión. Recuerde la ecuación termodinámica de estado para el volumen.
SEMINARIOS DE FISICOQUIMICA –INGENIERÍA QUÍMICA (PRIMERA PARTE) 28
CATEDRA DE FISICOQUIMICA I SEMINARIO 9
EQUILIBRIO QUIMICO
PROBLEMA 1:
A.- Cual de los siguientes diagramas de G y H como función de T satisface el teorema del calor de Nerst?
Justifique.
H H
G G
T/K T/K
B.- Indique si este enunciado del tercer principio de la termodinámica es correcto: En el 0 K de temperatura la
entropía de cualquier sustancia es cero. Justifique.
PROBLEMA 2:
A.- Se ha determinado la Cp

(en J/K mol) del Pb(s) entre 10 y 298 K. Los resultados obtenidos son los siguientes:
T/K 10 15 20 25 30 50 70 100 150 200 250 298
Cp
 2.8 7.0 10.8 14.1 16.5 21.5 23.3 24.5 25.3 25.8 26.2 26.6
Calcule la S
del Pb a 298 K. Para la integración por debajo de 10 K emplee la ecuación de Debye a bajas
temperaturas para Cv.
B.- Se ha medido la Cp

(en J/K mol) de la sal K4Fe(CN)6. Los datos obtenidos son:
T/K 0 10 20 30 40 50 60 70 80 90 100 120 130 140
Cp
 0 2.09 14.43 34.44 62.55 87.03 111 131,4 149,4 165,3 179.6 205 216,6 227,3
T/K 150 160 170 180 190 200 60
Cp
 237.6 247.3 256.5 265.1 273 287.03 280.3
Cual es la entropía de esta sal a 200 K y 1 atm?
PROBLEMA 3:
En el estudio termodinámico de N2 se ha obtenido la siguiente información a partir de datos térmicos
1
1
0
.
.
2
,
27
)
( 


  mol
K
J
dT
T
C
T
S
t
T p
t
Ttrans= 35.61 K, Ht=0.229 kJ.mol-1
1
1
.
4
,
23
)
(
)
( 



  mol
K
J
dT
T
C
T
S
T
S
f
t
T
T
p
t
f
Tfus= 63.14 K, Hf= 0.721 kJ/mol
1
1
.
4
,
11
)
(
)
( 



  mol
K
J
dT
T
C
T
S
T
S
e
f
T
T
p
f
e
Teb= 77.32 K, Hv= 5.58 kJ/mol
(1) ¿Cual es la entropía del N2 a 77.32 K? (2) ¿Cual es la entropía del N2 a 25C y 1 atm de presión? Emplee la
capacidad calorífica de tablas para este cálculo. (3) ¿Cual es el aumento de energía libre que experimenta el N2 si
se lo sometiera a una expansión adiabática reversible entre 25 y -25C. Suponga comportamiento ideal del gas.
PROBLEMA 4:
Se ha estudiado la dependencia de la Cp del NF3 desde 12.85 K hasta 140 K.
T/K 12.95 15 20 25 30 35 40 45 55 58.41 60 65 70 75 80
Cp 2.154 3.077 5.031 6.719 8.1 9.193 10.12 10.99 11.95 16.62 16.53 16.62 17.27 17.17 17.07
T/K 85 90 95 100 105 110 115 120 125 130 135 140
Cp 16,98 16,9 16,82 16,75 16,71 16,69 16,7 16,74 16,81 16,93 17,08 17,26
Se conoce que los cambios de entalpía en los distintos cambios de fase son:
Cambio de fase T/K H/Kcal mol-1
transición cristalina 56,62 381,8
fusión 66,37 95,1
ebullición 144,15 2769,0
G
H
H
G
SEMINARIOS DE FISICOQUIMICA –INGENIERÍA QUÍMICA (PRIMERA PARTE) 29
La Cv esta dada por la ecuación de Debye Cv=(48/5).R..(T/)3
, donde  es la temperatura caracteristica de Debye
para este solido (=120.8 K). Evalúe gráficamente la entropía absoluta del NF3 a 140 K.
PROBLEMA 5:
A.- Una mezcla constituida inicialmente por 3 partes de H2 y 1 de N2 se deja reacción ar a 200C y 1 atm. En estas
condición es se determinó la formación de 15.3% en volumen de NH3. Calcular Kp, Kx y G. Si a esta mezcla se
le introduce He hasta que la presión se eleva a 2 atm, calcular KX.
C.- Grado de avance . Sea la reacción: N2 (g) + 3 H2 (g)  2 NH3 (g). Si inicialmente hay 10 moles
de N2 en un exceso de H2 diga cuantos moles de cada sustancia han reaccionado o se han formado
cuando  = 0,1.
C.- Consideremos la reacción 2A(g) + B2(g)  A2B2(g), en la que se supone comportamiento ideal de
las especies. Se parte de 1 mol de A y 1 mol de B2 y 0,5 moles de A2B2 y se observa que el equilibrio
se establece cuando el grado de avance es 0,1. Calcule las fracciones molares en el equilibrio y si la
presión total es de 1 atm, calcular la constante de equilibrio en términos de las fracciones molares y la
constante de equilibrio Kp. La presión de trabajo es 2 atm.
D.- analice qué papel juega la energía libre de mezcla en la posición del equilibrio químico.
PROBLEMA 6:
B.- Al calentar vapores de PCl5 se descompone parcialmente en PCl3(g) y Cl2(g). A 1 atm de presión y 230C, la
densidad de la mezcla es 4.8 g/l. Calcular Kp, Kx y G.
C.- Para la reacción Cl2(g) 2 Cl(g), a 1000 K y 1 atm, Kp= 2.45 10-7
, cuando las presiones se dan en atm.
Calcular las composiciones en equilibrio en estas condiciones.
D.- Para la reacción de Cl2(g) + CO(g) Cl2CO(g) (fosgeno), se ha determinado que G= -48.77 Kcal/mol a
25C. Calcular Kp y Kx.
E.- Para esta reacción, G/cal.mol-1
=-24100 + 4 T lnT + 3.5 T Calcular la presión parcial de Cl2(g), H y S a
200 C y 1 atm. Encuentre CP a 200C y 1 atm, y una función de la temperatura para este.
PROBLEMA 7:
A.- Se ha medido el volumen ocupado por N2O4 en equilibrio con NO2 a 27C y 111C a P = 1 atm, siendo 2.95 l y
6.07 l respectivamente. El sistema se ha preparado agregando 9.2 g de N2O4. Sobre la base de esta información
calcular (1) el grado de disociación ,  rH,  rG (2) temperatura a la cual se igualan las presiones de dióxido y
tetróxido a la unidad (3) Composición a 111C (4) la presión a la cual =0.5 a 111C.
B.- Para la reacción de formación de H2O(g) se ha estudiado la dependencia de la constante de equilibrio con la
temperatura. Los datos obtenidos son:
T/K 1000 1200 1400 1500 1750 2000
lg Kp 10.059 7.893 6.34 5.716 4.365 3.528
Además se dispone de la siguiente información sobre las capacidades caloríficas
O2(g) Cp/cal K-1
mol-1
= 8.27 + 2.58 10-4
T - 1.877 105
T-2
(Válido entre 273 y 5000 K)
H2(g) Cp/cal K-1
mol-1
= 6.65 + 6.9 10-4
T (Válido entre 273 y 3500 K)
H2O(g) Cp/cal K-1
mol-1
= 7.2 + 2.7 10-3
T (Válido entre 273 y 2000 K)
(1) Obtener una expresión para Kp como función T (2) calcular la entropía de la reacción a 298 K y 1 atm (3) el
aumento de entropía a 298 K y 1 atm (4) la composición del sistema a 1 y 2 atm, resolviendo numérica y
gráficamente.
PROBLEMA 8:
Cuando el NH4Cl se calienta su presión de vapor aumenta de 427C a 459C, la presión de vapor se incrementa de
608 kPa a 1115 kPa. Calcular (1) Kp a 427C y el aumento de la función de Gibbs en condiciones estándar (2) los
aumentos de entropía y entalpía a esa temperatura.
PROBLEMA 9: Cálculo de entropías de sustancias puras.
a.- Ecuación de Debye para Cv a bajas temperaturas. Debye demostró que, a bajas temperaturas, la capacidad
calorífica a volumen cte. puede escribirse: CV = 464,6 (T/)3
, donde  es un parámetro propio de cada sólido
cristalino conocida como su temperatura de Debye y su valor puede consultarse en tablas. Para el Ar,  = 84 K.
(a) Calcule CV del Ar a 10K. (b) Con este valor calcule la entropía del Ar a 10K. (c) Calcule el aumento de
entropía que debiera observarse si un mol de un sólido cristalino se calienta desde 0 hasta 15 K a 1 atm de
presión. (d) Analice porque puede utilizar Cv en lugar de Cp.
b.- Cálculo de entropías a partir de datos térmicos
En el estudio termodinámico de N2 se ha obtenido la siguiente información a partir de datos térmicos
SEMINARIOS DE FISICOQUIMICA –INGENIERÍA QUÍMICA (PRIMERA PARTE) 30
1
1
1
1
1
1
1
1
1
0
.
580
.
5
32
.
77
.
4
.
11
ln
.
.
271
.
0
14
.
63
.
4
.
23
ln
.
.
229
.
0
61
.
35
.
2
.
27
ln
.
























mol
kJ
H
K
T
mol
K
J
T
d
C
mol
kJ
H
K
T
mol
K
J
T
d
C
mol
kJ
H
K
T
mol
K
J
T
d
C
m
f
f
p
T
Tf
m
f
f
p
T
Tt
m
t
t
p
T
e
f
t
a.- ¿Cual es la entropía del N2 líquido y del N2 gaseoso a 77.32 K?
b.- Calcule es la entropía del N2(g) a 25C y 1 atm de presión (i) conociendo la dependencia experimental
2
6
3
1
1 10
50
.
0
.
10
77
.
3
58
.
28
/
T
x
T
x
mol
JK
Cp 

 


(ii) recurriendo al principio de equipartición para
estimar Cp.
c.- ¿Cual es el aumento de energía libre que experimenta el N2(g) si se lo sometiera a una expansión adiabática
reversible entre 25 y -25C. Suponga comportamiento ideal del gas.
PROBLEMA 9:
A.- Considere la reacción N2O4(g)  2 NO2(g). En la siguiente tabla se presenta información termodinámica para
estas sustancias a 298 K y 1 atm.
sustancia fH
/kJ.mol-1
S
/J.K-1
mol-1
Cp

/J.K-1
mol-1 (1) Calcule la constante de equilibrio a
298 K y a 398 K. (2) determine la
composición del sistema a 398 K y 1
atm.
N2O4(g) 11,1 304,4 79,2
NO2(g) 33,2 240,1 37,2
B.- Empleando la información que se indica en la tabla siguiente y la contenida en la previa, evalúe las energías
libres de formación de las especies involucradas en la reacción de este problema a 298 y a 373 K. En este caso
evalúe la constante de equilibrio empleando las energías libres de formación.
sustancia fH
/kJ.mol-1
S
/J.K-1
mol-1
Cp

/J.K-1
mol-1
N2(g) -- 191,6 29,1
O2(g) -- 205,2 29,4
PROBLEMA 10:
Se ha estudiado en fase gaseosa la producción de ciclohexano por hidrogenación del benceno. Las entalpías de
formación y las entropías en estado estándar a 298 K se muestran en la siguiente tabla. Por su parte, en esta tabla se
indican los coeficientes a, b y c que expresan la dependencia de la capacidad calorífica estándar de cada sustancia
función de la temperatura. Estos coeficientes son válidos en el intervalo de temperatura entre 25 y 525°C.
Sustancia fHm

Sm

Cp,m/J.K-1
.mol-1
= a + b.T + c.T2
kJ.mol-1
J.K-1
.mol-1
a b 104
c
Benceno 82,93 269,03 -14,81 0,378 -1,52
Ciclohexeno -123,13 298,32 52,13 0,599 -2,30
Hidrógeno 0 130,66 27,12 0,0038 0
Con la información precedente estudie las siguientes situaciones y analice los alcances de las distintas
consideraciones que se emplean en cálculos termodinámicos de esta naturaleza.
(1) Calcule las capacidades caloríficas a 298 K empleando la dependencia de esta propiedad con la temperatura que
se indica en la Tabla. Con esta información Calcule los aumentos de entalpía (fHm

) de entropía (rSm

) y de
energía libre (rGm

) correspondientes y evalúe la constante de equilibrio a 400 C suponiendo que Cp

no es
función de T.
(2) Evalúe los aumentos de entalpía (fHm

) de entropía (rSm

) y de energía libre (rGm

) correspondientes y
evalúe la constante de equilibrio a 400 C considerando la dependencia de Cp,m indicadas en la tabla para cada
sustancia. Compare estos resultados con los obtenidos en el punto previo. Calcule el error relativo que se comete al
suponer que las capacidades caloríficas no dependen de la temperatura.
(3) Calcule la capacidad calorífica promedio <Cp,m

> en el intervalo de temperatura entre 25 y 525C. Con esta
información calcule los aumentos de entalpía (fHm

) de entropía (rSm

) y de energía libre (rGm

)
correspondientes y evalúe la constante de equilibrio a 400 C empleando este valor promedio. Compare estos
SEMINARIOS DE FISICOQUIMICA –INGENIERÍA QUÍMICA (PRIMERA PARTE) 31
resultados con los obtenidos en el punto previo. Calcule el error relativo que se comete al suponer que las
capacidades caloríficas no dependen de la temperatura.
PROBLEMA 11:
A.- En el intervalo de temperatura de 450 a 715 K, los valores de la presión de equilibrio de H2(g) sobre -U(s) y
UH3(s) puede representarse mediante ln p(H2)= 69.32 - 14.64 103
/T - 5.65 ln T, donde p esta en Pa y T en K.
Encontrar una expresión de la entalpía estándar de formación del -UH3(s) como función de T y a partir de ella
calcule Cp.
B.- En el intervalo de temperatura de 350 a 470 K el aumento de entalpía standard para la disociación de
CaCl2.NH3(s) en NH3(g) y CaCl2(s) es constante e igual a 78.15 kJ/mol. La presión de disociación de equilibrio del
NH3(g) es de 12.8 torr a 400 K. Hallar una expresión para la función energía libre estándar de Gibbs como función
de T.
PROBLEMA 12: Se sabe que Gf

para la reacción: H2 (g) + (1/2) O2 (g)  H2O (g) es Gf

= -54,507 kcal
mol-1
. Se desea calcular Gf

de la reacción: H2 (g) + (1/2) O2 (g)  H2O (l) y con ella su constante de
equilibrio a 298.16K. La presión de vapor del agua a 298,16K es 23,8 Torr.
PROBLEMA 13: Encuentre una expresión para la constante de equilibrio para la siguiente reacción:
CO2 (g) + 2 NH3 (g) = CO(NH2)2(solución)+ H2O(l)
a partir de la igualdad de los respectivos potenciales químicos. Indique que estado de referencia elije para
evaluar la actividad de cada substancia y cuales son los estados estándares que le corresponden. Datos:
Gf

(CO2 , g) = -94,26 kcal mol-1
Gf

(CO(NH2)2 , solución) = -48.720 kcal mol-1
Gf

(NH3 , g) = -3,976 kcal mol-1
Gf

(H2O , líquido.) = -56,69 kcal mol-1
Calcule la constante de equilibrio.
PROBLEMA 14: Coeficiente de reparto de una especie entre dos líquidos/solvente. Demuestre que el
coeficiente de partición de un soluto entre dos solventes distintos es, en soluciones diluidas, igual a la relación
entre las constantes de Henry.
SEMINARIOS DE FISICOQUIMICA –INGENIERÍA QUÍMICA (PRIMERA PARTE) 32
FISICOQUIMICA I
SEMINARIOS SOBRE TEMAS RELACIONADOS CON LOS TRABAJOS PRACTICOS
ELECTROQUIMICA
PROBLEMA N° 1
En la medida de la fuerza electromotriz de una pila, se aplicó el método de Poggendorff o de oposición. En el
trabajo práctico correspondiente se ha representado el circuito básico del potenciómetro Student. Este
instrumento consta de 16 resistencias calibradas en serie, de 10 ohms cada una, de manera que la resistencia
total sea de 160 ohms. Estas resistencias están agrupadas de manera tal que una de ellas pueda variarse en forma
contínua entre 0 y 10 ohms. Las 15 restantes se varían en pasos de 10 ohms. Durante su calibración, debe
asegurarse que por el instrumento circules 10 mA. De esta manera la caída total de tensión entre la entrada y la
salida corresponden siempre a 1,6 voltios.
(1) Esquematice el sistema de resistencias internas indicado en la figura antes mencionada y señale en una
escala adecuada cuál es la resistencia a intercalar para que la caída de tensión en las condiciones de medida sea
de 1,2 voltios.
(2) En la calibración usted ha intercalado dos resistencias externas en paralelo. Si la FEM de la pila patrón vale
1,0185 voltios, ¿cuál es el valor de la resistencia equivalente que debe intercalar para asegurar que la caída de
tensión sobre 101,85 ohms del potenciómetro correspondan a la FEM deseada? Suponga que su batería tiene una
FEM de 2 voltios.
(3) Con el valor de la resistencia equivalente hallado en el inciso anterior estime cuál será la menor variación
que puede obtener en la resistencia equivalente si en una de ellas ajusta un valor de 250 ohms.
Para problemas tipo del circuito de oposición elemental puede consultar cualquier texto sobre electricidad que
haya usado en los cursos de física.
PROBLEMA N° 2
Considere el sistema electroquímico T / M / S / M' / T' , donde T y T' representan las terminales eléctricas de la
pila y cuya naturaleza debe ser exactamente la misma con la que se construye el equipo potenciométrico (por
ejemplo Cu). Las barras indican una interfase entre las distintas partes del sistema, M y M' son dos metales
determinados y S es la solución, que aquí se ha simplificado al no indicar su naturaleza.
Se llama fuerza electromotriz de la pila a la diferencia de potencial medida entre los terminales T cuando no
existe circulación de corriente eléctrica.
(1) Justifique desde el punto de vista termodinámico por qué no debe circular corriente eléctrica al medir la
FEM de la pila.
(2) Justifique por qué T y T' deben ser del mismo material con el cual se construye el potenciómetro.
PROBLEMA N°3
A.- Considere un metal M que se sumerge en una solución de sus iones Mz+; si a+ es la actividad de estos iones
en la solución y aM la correspondiente a los iones en el retículo metálico, y por ejemplo a+ > aM entonces
ocurrirá espontáneamente el siguiente proceso: Mz+ + ze-  M. Cada vez que un ion se incorpora a la red del
metal, este aumenta su potencial eléctrico respecto del de la solución, es decir, se hace positivo respecto a la
solución. El proceso continúa hasta que la diferencia de potencial en la interfaz crece lo suficiente para oponerse
a la tendencia espontánea originada en la diferencia de actividades (potenciales químicos) entre los iones en la
solución y la red metálica.
(1) Muestre que la condición de equilibrio se establece cuando se igualan los potenciales electroquímicos i de
las especies consideradas, definidos en términos generales como 

 F
zk
k
k 
 donde  es el potencial
eléctrico de la fase considerada. (2) Encuentre que la diferencia de potencial metal solución (M - S), para el
proceso considerado en este enunciado puede escribirse como:
 = (M - S) = (1/zF) (+
 + z e(M) - M
 ) + (RT/zF) ln (a+ / aM)
donde los k
 corresponden a los potenciales químicos de los iones en la solución, en la red y de los electrones
en estado estándar (actividad unitaria a la presión de una atmósfera).
B.- Sobre la base de la expresión deducida anteriormente encuentre las expresiones para los potenciales de
electrodo correspondientes a los siguientes sistemas:
(1) electrodo de primera especie M / M2+ por ejemplo Cu / Cu++(a+)
(2) electrodo gaseoso, por ejemplo Pt / H2 (g, p) / H+ (a+)
(3) electrodo de segunda especie, por ejemplo Hg / Hg2Cl2(s) / Cl- (a-)
(4) electrodo redox o de tercera especie
(5) electrodo de amalgama.
SEMINARIOS DE FISICOQUIMICA –INGENIERÍA QUÍMICA (PRIMERA PARTE) 33
C.- Calcule el cambio de la diferencia de potencial a 25°C entre un electrodo de plata / cloruro de plata y una
solución saturada de cloruro de plata, cuando se agrega una solución 0,01 M de KCl. Considere el KpsAgCl =
1,0 10-10 a 25°C.
PROBLEMA N°4
A.- Si (T), (T'), (M), (M') y (S) son los potenciales eléctricos de cada una de las fases y además ED y
EI son los potenciales de electrodo correspondiente a los electrodos de la derecha (D) e izquierda (I), entonces la
FEM de la pila está dada por FEM = ED - EI .
Encuentre una expresión general para esta diferencia en términos de los potenciales eléctricos anteriormente
mencionados. Discuta la convención de signos.
B.- Encuentre la expresión de la FEM de la pila de Daniell.
C.- Cuando se enfrenta el electrodo de Ag/AgCl(s)/Cl(a-) contra el electrodo normal de H2 en un experimento
hipotético con todas las especies en estado estándar, se lee una FEM de 0,2224 V a la temperatura de trabajo. Se
observa que la corriente eléctrica se mueve desde el electrodo de Ag/AgCl(s)/Cl(a-) al ENH. Escriba la pila e
indique cuánto vale el potencial normal del electrodo en cuestión. Justifique el procedimiento y represente la
pila.
D.- Se enfrenta el electrodo de Pb(s)/PbSO4(s)/SO4
-2
(a_
) contra el ENH en un experimento hipotético en
condiciones estándar. La corriente eléctrica fluye desde el ENH al electrodo en cuestión. Si la FEM que se lee
en este experimento vale 0,36 V, halle el potencial normal del electrodo Pb(s)/PbSO4(s)/SO4
-2
(a_
). Justifique el
procedimiento y represente la pila.
PROBLEMA N°5
A.- Defina actividad iónica, media y actividad del electrolito para los ejemplos (i) HCl (ii) BaCl2, (iii) ZnSO4
(iv) FeCl3. Para estudiar el temario que sigue puede consultarse la siguiente literatura: Lewis & Randal (2° ed.):
Termodynamics; Guggenheim & Prue, Physicochemical Calculations y la literatura general indicada para el
curso.
B.- La actividad de un electrolito puede evaluarse por distintos procedimientos. En el caso de las soluciones de
HCl, las medidas pueden realizarse determinando las presiones de vapor, medidas de FEM de pilas del tipo
Pt / H2(g , 1 atm) / HCl(m) / AgCl / Ag o a partir del descenso crioscópico. El objetivo de este problema es la
familiarización del estudiante con estos tres procedimientos.
(1) A partir de medidas de descenso crioscópico Tf
En el texto de Glasstone (Tratado de Química Física, Capítulos IX y XII) y en el texto de Castellan
(Fisicoquímica) puede consultarse las deducciones de las expresiones que permiten evaluar coeficientes de
actividad media de electrolitos a partir del descenso crioscópico.
A partir de la siguiente información evaluar los coeficientes de actividad de las soluciones 0,3 molal (m) y 1 m.
m/(moles/Kg) 0,001 0,002 0,005 0,010 0,020 0,050
Tc103 / K 3,675 7,318 18,152 36,028 71,43 176,66
m/(moles/Kg) 0,100 0,200 0,300 0,500 0,700 1,000
Tc103 / K 325,09 706,4 1068,9 1822,5 2580,1 351
La constante crioscópica del agua es kc = 1,858 K moles-1Kg
(2) A partir de medidas de presión de vapor de soluciones de HCl
m/ moles/Kg 4 5 6 7 8 9 10
p 104 / atm 0,235 0,6974 1,842 4,579 11,10 23,39 55,26
± 1,74 2,38 3,23 4,36 5,95 7,97 10,61
± es el factor de actividad medio de esas soluciones. Verifique que en soluciones de electrolitos 1-1 valentes
como HCl, la presión de vapor debe depender del cuadrado de la molalidad en el límite de bajas
concentraciones, es decir lim m 0 (p/m2) = kH, donde kH es la constante de Henry correspondiente.
Determinar la constante de Henry y evaluar el factor de actividad medio de soluciones 5,5 y 8,5 m de HCl.
(3) A partir de medidas de FEM de la pila indicada anteriormente:
m 0,1238 0,05391 0,02563 0,01341 0,09138 0,005619 0,003215
FEM 0,34199 0,38222 0,41824 0,44974 0,46860 0,49257 0,52053
En la tabla anterior m se da en moles de HCl por 1000 g de solvente y la FEM en voltios.
SEMINARIOS PRIMERA PARTE-2015 (1).pdf
SEMINARIOS PRIMERA PARTE-2015 (1).pdf
SEMINARIOS PRIMERA PARTE-2015 (1).pdf
SEMINARIOS PRIMERA PARTE-2015 (1).pdf
SEMINARIOS PRIMERA PARTE-2015 (1).pdf
SEMINARIOS PRIMERA PARTE-2015 (1).pdf
SEMINARIOS PRIMERA PARTE-2015 (1).pdf
SEMINARIOS PRIMERA PARTE-2015 (1).pdf
SEMINARIOS PRIMERA PARTE-2015 (1).pdf
SEMINARIOS PRIMERA PARTE-2015 (1).pdf
SEMINARIOS PRIMERA PARTE-2015 (1).pdf

Más contenido relacionado

Similar a SEMINARIOS PRIMERA PARTE-2015 (1).pdf

Fisicoquimica presion de vapor
Fisicoquimica presion de vaporFisicoquimica presion de vapor
Fisicoquimica presion de vaporJean M Sanchez
 
1. Química 1 BQU01A FIQT Gases reales.pdf
1. Química 1 BQU01A FIQT Gases reales.pdf1. Química 1 BQU01A FIQT Gases reales.pdf
1. Química 1 BQU01A FIQT Gases reales.pdfTuestaMermaAlissonBr
 
GUIA PEDAGOGICA TERMODINAMICA SUSTANCIAS PURAS.pdf
GUIA PEDAGOGICA TERMODINAMICA SUSTANCIAS PURAS.pdfGUIA PEDAGOGICA TERMODINAMICA SUSTANCIAS PURAS.pdf
GUIA PEDAGOGICA TERMODINAMICA SUSTANCIAS PURAS.pdfCarlosJosFuentesApon
 
Cap. ii. comport. del gas nat. 1(1 49)
Cap. ii. comport. del  gas nat. 1(1 49)Cap. ii. comport. del  gas nat. 1(1 49)
Cap. ii. comport. del gas nat. 1(1 49)Pablo Campos Zenteno
 
VI-GASES IDEALES Y REALES, de la Universidad Nacional de Ingeniria
VI-GASES IDEALES Y REALES, de la Universidad Nacional de IngeniriaVI-GASES IDEALES Y REALES, de la Universidad Nacional de Ingeniria
VI-GASES IDEALES Y REALES, de la Universidad Nacional de Ingeniriacuentadepruebas13579
 
Diapositivas gas ideal+reales
Diapositivas gas ideal+realesDiapositivas gas ideal+reales
Diapositivas gas ideal+realesoriel arancibia
 
Problemas resueltos de gases ideales
Problemas resueltos de gases ideales Problemas resueltos de gases ideales
Problemas resueltos de gases ideales Sandy Palencia
 
Equilibrio Gaseoso 2023 ESGE.pptx
Equilibrio Gaseoso 2023 ESGE.pptxEquilibrio Gaseoso 2023 ESGE.pptx
Equilibrio Gaseoso 2023 ESGE.pptxdemamania
 

Similar a SEMINARIOS PRIMERA PARTE-2015 (1).pdf (20)

Fisicoquimica presion de vapor
Fisicoquimica presion de vaporFisicoquimica presion de vapor
Fisicoquimica presion de vapor
 
1. Química 1 BQU01A FIQT Gases reales.pdf
1. Química 1 BQU01A FIQT Gases reales.pdf1. Química 1 BQU01A FIQT Gases reales.pdf
1. Química 1 BQU01A FIQT Gases reales.pdf
 
Ejercicios tipo examen
Ejercicios tipo examenEjercicios tipo examen
Ejercicios tipo examen
 
FISICO QUIMICA GASES REALES.pptx
FISICO QUIMICA GASES REALES.pptxFISICO QUIMICA GASES REALES.pptx
FISICO QUIMICA GASES REALES.pptx
 
Física (II Bimestre)
Física (II Bimestre)Física (II Bimestre)
Física (II Bimestre)
 
GUIA PEDAGOGICA TERMODINAMICA SUSTANCIAS PURAS.pdf
GUIA PEDAGOGICA TERMODINAMICA SUSTANCIAS PURAS.pdfGUIA PEDAGOGICA TERMODINAMICA SUSTANCIAS PURAS.pdf
GUIA PEDAGOGICA TERMODINAMICA SUSTANCIAS PURAS.pdf
 
Cap. ii. comport. del gas nat. 1(1 49)
Cap. ii. comport. del  gas nat. 1(1 49)Cap. ii. comport. del  gas nat. 1(1 49)
Cap. ii. comport. del gas nat. 1(1 49)
 
Laboratorio 7
Laboratorio 7Laboratorio 7
Laboratorio 7
 
VI-GASES IDEALES Y REALES, de la Universidad Nacional de Ingeniria
VI-GASES IDEALES Y REALES, de la Universidad Nacional de IngeniriaVI-GASES IDEALES Y REALES, de la Universidad Nacional de Ingeniria
VI-GASES IDEALES Y REALES, de la Universidad Nacional de Ingeniria
 
Guía 3 Mezclas de gases ideales juan sandoval herrera
Guía 3 Mezclas de gases ideales juan sandoval herreraGuía 3 Mezclas de gases ideales juan sandoval herrera
Guía 3 Mezclas de gases ideales juan sandoval herrera
 
Diapositivas gas ideal+reales
Diapositivas gas ideal+realesDiapositivas gas ideal+reales
Diapositivas gas ideal+reales
 
Termodinamica (1)
Termodinamica (1)Termodinamica (1)
Termodinamica (1)
 
Termodinamica.doc
Termodinamica.docTermodinamica.doc
Termodinamica.doc
 
8 compresores
8  compresores8  compresores
8 compresores
 
8 compresores
8  compresores8  compresores
8 compresores
 
Cap 1 gases
Cap 1 gasesCap 1 gases
Cap 1 gases
 
Compresores 2010
Compresores 2010Compresores 2010
Compresores 2010
 
9 na semana cepre unmsm
9 na semana cepre unmsm9 na semana cepre unmsm
9 na semana cepre unmsm
 
Problemas resueltos de gases ideales
Problemas resueltos de gases ideales Problemas resueltos de gases ideales
Problemas resueltos de gases ideales
 
Equilibrio Gaseoso 2023 ESGE.pptx
Equilibrio Gaseoso 2023 ESGE.pptxEquilibrio Gaseoso 2023 ESGE.pptx
Equilibrio Gaseoso 2023 ESGE.pptx
 

Último

Sistema de Control Interno aplicaciones en nuestra legislacion
Sistema de Control Interno aplicaciones en nuestra legislacionSistema de Control Interno aplicaciones en nuestra legislacion
Sistema de Control Interno aplicaciones en nuestra legislacionPedroSalasSantiago
 
PRUEBA PRE ICFES ECONOMIA. (4) - copia.doc
PRUEBA PRE ICFES ECONOMIA. (4) - copia.docPRUEBA PRE ICFES ECONOMIA. (4) - copia.doc
PRUEBA PRE ICFES ECONOMIA. (4) - copia.docmilumenko
 
QUE REQUISITOS DEBO CUMPLIR PARA PENSIONARME.pdf
QUE REQUISITOS DEBO CUMPLIR PARA PENSIONARME.pdfQUE REQUISITOS DEBO CUMPLIR PARA PENSIONARME.pdf
QUE REQUISITOS DEBO CUMPLIR PARA PENSIONARME.pdflupismdo
 
ejemplo de tesis para contabilidad- capitulos
ejemplo de tesis para contabilidad- capitulosejemplo de tesis para contabilidad- capitulos
ejemplo de tesis para contabilidad- capitulosguillencuevaadrianal
 
41 RAZONES DE PORQUE SI ESTAMOS MAL EN MÉXICO
41 RAZONES DE PORQUE SI ESTAMOS MAL EN MÉXICO41 RAZONES DE PORQUE SI ESTAMOS MAL EN MÉXICO
41 RAZONES DE PORQUE SI ESTAMOS MAL EN MÉXICOlupismdo
 
Principios de economia Mankiw 6 edicion.pdf
Principios de economia Mankiw 6 edicion.pdfPrincipios de economia Mankiw 6 edicion.pdf
Principios de economia Mankiw 6 edicion.pdfauxcompras5
 
mercado de capitales universidad simon rodriguez - guanare (unidad I).pdf
mercado de capitales universidad simon rodriguez - guanare (unidad I).pdfmercado de capitales universidad simon rodriguez - guanare (unidad I).pdf
mercado de capitales universidad simon rodriguez - guanare (unidad I).pdfGegdielJose1
 
METODOS ESCALA SALARIAL EN ESTRUCTURAS.PPT
METODOS ESCALA SALARIAL EN ESTRUCTURAS.PPTMETODOS ESCALA SALARIAL EN ESTRUCTURAS.PPT
METODOS ESCALA SALARIAL EN ESTRUCTURAS.PPTrodrigolozanoortiz
 
Análisis de la Temporada Turística 2024 en Uruguay
Análisis de la Temporada Turística 2024 en UruguayAnálisis de la Temporada Turística 2024 en Uruguay
Análisis de la Temporada Turística 2024 en UruguayEXANTE
 
canasta basica de la india original 2 .pptx
canasta basica de la india original 2 .pptxcanasta basica de la india original 2 .pptx
canasta basica de la india original 2 .pptxarmandoantoniomartin1
 
LOS MIMBRES HACEN EL CESTO: AGEING REPORT.
LOS MIMBRES HACEN EL CESTO: AGEING  REPORT.LOS MIMBRES HACEN EL CESTO: AGEING  REPORT.
LOS MIMBRES HACEN EL CESTO: AGEING REPORT.ManfredNolte
 
Politicas publicas para el sector agropecuario en México.pptx
Politicas publicas para el sector agropecuario en México.pptxPoliticas publicas para el sector agropecuario en México.pptx
Politicas publicas para el sector agropecuario en México.pptxvladisse
 
El cheque 1 y sus tipos de cheque.pptx
El cheque  1 y sus tipos de  cheque.pptxEl cheque  1 y sus tipos de  cheque.pptx
El cheque 1 y sus tipos de cheque.pptxNathaliTAndradeS
 
TEMA 3 DECISIONES DE INVERSION Y FINANCIACION UNIVERISDAD REY JUAN CARLOS
TEMA 3 DECISIONES DE INVERSION Y FINANCIACION UNIVERISDAD REY JUAN CARLOSTEMA 3 DECISIONES DE INVERSION Y FINANCIACION UNIVERISDAD REY JUAN CARLOS
TEMA 3 DECISIONES DE INVERSION Y FINANCIACION UNIVERISDAD REY JUAN CARLOSreyjuancarlosjose
 
puntos-clave-de-la-reforma-pensional-2023.pdf
puntos-clave-de-la-reforma-pensional-2023.pdfpuntos-clave-de-la-reforma-pensional-2023.pdf
puntos-clave-de-la-reforma-pensional-2023.pdfosoriojuanpablo114
 
MANUAL PARA OBTENER MI PENSIÓN O RETIRAR MIS RECURSOS.pdf
MANUAL PARA OBTENER MI PENSIÓN O RETIRAR MIS RECURSOS.pdfMANUAL PARA OBTENER MI PENSIÓN O RETIRAR MIS RECURSOS.pdf
MANUAL PARA OBTENER MI PENSIÓN O RETIRAR MIS RECURSOS.pdflupismdo
 

Último (17)

Sistema de Control Interno aplicaciones en nuestra legislacion
Sistema de Control Interno aplicaciones en nuestra legislacionSistema de Control Interno aplicaciones en nuestra legislacion
Sistema de Control Interno aplicaciones en nuestra legislacion
 
PRUEBA PRE ICFES ECONOMIA. (4) - copia.doc
PRUEBA PRE ICFES ECONOMIA. (4) - copia.docPRUEBA PRE ICFES ECONOMIA. (4) - copia.doc
PRUEBA PRE ICFES ECONOMIA. (4) - copia.doc
 
QUE REQUISITOS DEBO CUMPLIR PARA PENSIONARME.pdf
QUE REQUISITOS DEBO CUMPLIR PARA PENSIONARME.pdfQUE REQUISITOS DEBO CUMPLIR PARA PENSIONARME.pdf
QUE REQUISITOS DEBO CUMPLIR PARA PENSIONARME.pdf
 
ejemplo de tesis para contabilidad- capitulos
ejemplo de tesis para contabilidad- capitulosejemplo de tesis para contabilidad- capitulos
ejemplo de tesis para contabilidad- capitulos
 
41 RAZONES DE PORQUE SI ESTAMOS MAL EN MÉXICO
41 RAZONES DE PORQUE SI ESTAMOS MAL EN MÉXICO41 RAZONES DE PORQUE SI ESTAMOS MAL EN MÉXICO
41 RAZONES DE PORQUE SI ESTAMOS MAL EN MÉXICO
 
Mercado Eléctrico de Ecuador y España.pdf
Mercado Eléctrico de Ecuador y España.pdfMercado Eléctrico de Ecuador y España.pdf
Mercado Eléctrico de Ecuador y España.pdf
 
Principios de economia Mankiw 6 edicion.pdf
Principios de economia Mankiw 6 edicion.pdfPrincipios de economia Mankiw 6 edicion.pdf
Principios de economia Mankiw 6 edicion.pdf
 
mercado de capitales universidad simon rodriguez - guanare (unidad I).pdf
mercado de capitales universidad simon rodriguez - guanare (unidad I).pdfmercado de capitales universidad simon rodriguez - guanare (unidad I).pdf
mercado de capitales universidad simon rodriguez - guanare (unidad I).pdf
 
METODOS ESCALA SALARIAL EN ESTRUCTURAS.PPT
METODOS ESCALA SALARIAL EN ESTRUCTURAS.PPTMETODOS ESCALA SALARIAL EN ESTRUCTURAS.PPT
METODOS ESCALA SALARIAL EN ESTRUCTURAS.PPT
 
Análisis de la Temporada Turística 2024 en Uruguay
Análisis de la Temporada Turística 2024 en UruguayAnálisis de la Temporada Turística 2024 en Uruguay
Análisis de la Temporada Turística 2024 en Uruguay
 
canasta basica de la india original 2 .pptx
canasta basica de la india original 2 .pptxcanasta basica de la india original 2 .pptx
canasta basica de la india original 2 .pptx
 
LOS MIMBRES HACEN EL CESTO: AGEING REPORT.
LOS MIMBRES HACEN EL CESTO: AGEING  REPORT.LOS MIMBRES HACEN EL CESTO: AGEING  REPORT.
LOS MIMBRES HACEN EL CESTO: AGEING REPORT.
 
Politicas publicas para el sector agropecuario en México.pptx
Politicas publicas para el sector agropecuario en México.pptxPoliticas publicas para el sector agropecuario en México.pptx
Politicas publicas para el sector agropecuario en México.pptx
 
El cheque 1 y sus tipos de cheque.pptx
El cheque  1 y sus tipos de  cheque.pptxEl cheque  1 y sus tipos de  cheque.pptx
El cheque 1 y sus tipos de cheque.pptx
 
TEMA 3 DECISIONES DE INVERSION Y FINANCIACION UNIVERISDAD REY JUAN CARLOS
TEMA 3 DECISIONES DE INVERSION Y FINANCIACION UNIVERISDAD REY JUAN CARLOSTEMA 3 DECISIONES DE INVERSION Y FINANCIACION UNIVERISDAD REY JUAN CARLOS
TEMA 3 DECISIONES DE INVERSION Y FINANCIACION UNIVERISDAD REY JUAN CARLOS
 
puntos-clave-de-la-reforma-pensional-2023.pdf
puntos-clave-de-la-reforma-pensional-2023.pdfpuntos-clave-de-la-reforma-pensional-2023.pdf
puntos-clave-de-la-reforma-pensional-2023.pdf
 
MANUAL PARA OBTENER MI PENSIÓN O RETIRAR MIS RECURSOS.pdf
MANUAL PARA OBTENER MI PENSIÓN O RETIRAR MIS RECURSOS.pdfMANUAL PARA OBTENER MI PENSIÓN O RETIRAR MIS RECURSOS.pdf
MANUAL PARA OBTENER MI PENSIÓN O RETIRAR MIS RECURSOS.pdf
 

SEMINARIOS PRIMERA PARTE-2015 (1).pdf

  • 1. SEMINARIOS DE FISICOQUIMICA –INGENIERÍA QUÍMICA (PRIMERA PARTE) 1 SEMINARIOS CATEDRA DE FISICOQUÍMICA (Primera Parte) CARRERA: INGENIERIA QUÍMICA Profesor: Dr. Alberto L. Capparelli Departamento de Química Facultad de Ciencias Exactas UNLP
  • 2. SEMINARIOS DE FISICOQUIMICA –INGENIERÍA QUÍMICA (PRIMERA PARTE) 2 CATEDRA DE FISICOQUIMICA I SEMINARIO Nro 1 GAS IDEAL Y SU APLICACION A SITUACIONES DE INTERES EN FISICOQUIMICA PROBLEMA 1: El modelo del gas ideal se sustenta en las leyes de Boyle y de Gay-Lussac, que surgen del análisis del comportamiento experimental en condiciones de bajas presiones y altas temperaturas. Analice si a temperatura ambiente (300K) y 1 atm puede afirmar si los siguientes gases se hallan en lo que usted entiende como “alta o baja temperatura”. Los gases son He (-267,9 °C), H2 (-239,9 °C), O2 (-118,8 °C), metano (-82,5 °C), CO2 (31,1 °C), SO2 (157,2 °C). Entre paréntesis la temperatura crítica de cada sustancia. PROBLEMA 2: A.- DETERMINACIÓN DEL PESO MOLECULAR DE UNA SUSTANCIA EN LA ESCALA DEL GAS IDEAL. MÉTODO DE DENSIDADES LÍMITES. Los siguientes datos de presión, volumen y densidad han sido obtenidos para un cierto gas a un temperatura T. p/atm 0.750000 0.500000 0.250000 Vm/L mol-1 29.8649 44.8090 89.6384 /(gL-1 ) 1.07144 0.714110 0.356975 (1) Represente el producto pVm en función de la presión. Teniendo en cuenta que en límite de bajas presiones el producto pVm tiende a lo esperado por la ley de Boyle, evalúe la temperatura a la que se realizaron las medidas. (2) Si la ley del gas ideal fuese válida para este sistema, la relación /p o su inversa p/ debería ser constante e independiente de la presión. Verifique que esta relación no es constante en forma en forma gráfica y analíticamente. (3) El comportamiento previo se ajusta en este intervalo de presiones a una dependencia lineal y=a + b.x, donde y=/p por un lado, mientras que x= p. Una dependencia similar se obtiene se toma y= p/ con x=p. En este caso la dependencia lineal es de la forma y=a’+b’.x, ¿Cómo obtiene de este análisis el peso molecular del gas en la escala del gas ideal a partir de cualquiera de las dos formas de construir la dependencia lineal? Nota: la representación /p versus p se denomina método de las densidades límites. (4) Halle qué relación existe entre a y a’ por un lado y entre b y b’ por el otro en las expresiones estudiadas previamente. (5) La dependencia lineal observada es una expresión simplificada de lo que estudiaremos como desarrollos del virial para gases. A presiones bajas y moderadas, pVm=RT + A2.p+…, En este tipo de desarrollo, A2 se conoce como segundo coeficiente del virial, y este coeficiente es una función de la temperatura y de la naturaleza del gas. Evalúe el segundo coeficiente del virial y exprese en términos de los coeficientes del virial que relación existe entre b por un lado y b’ por el otro en términos de A2. PROBLEMA 3: CONSTRUCCIÓN DE UNA ESCALA DE TEMPERATURA EN BASE AL GAS IDEAL. A.- La dependencia de la presión con la temperatura (C) a volumen constante puede escribirse como p=p0.[1+V.t], En el caso del volumen medido como función de t (C) a presión constante, V=V0.[1+p.t]. Los coeficientes V y p se determinan experimentalmente, y se observa que ambos son una función explícita o implícita de la presión. La representación de estos coeficientes versus la presión muestra experimentalmente que tienden a un único valor cuando p0, siempre que las determinaciones se realicen en contacto con una única fuente térmica. Si los experimentos se realizan en contacto con agua en su punto triple como fuente térmica. Explique el procedimiento que permita asignar al agua en su punto triple una temperatura en la escala del gas ideal. B.- A presión constante el volumen molar de un gas puede expresarse como donde a es una función de la presión. Calcule el valor de esta magnitud para el gas ideal empleado los valores de a experimental para nitrógeno e hidrógeno dados en la siguiente tabla: gas Nitrógeno Hidrógeno p/torr 508,2 1095,3 511,4 1105,3 106 p/(°C) 3660,2 3659,0 3667,9 3674,2 PROBLEMA 4: LEY DE DALTON. PESOS MOLECULARES PROMEDIOS. La ecuación del gas ideal puede aplicarse en mezclas de gases ideales. A.- (1) Defina peso molecular promedio de una mezcla <M> (2) calcule cual es el valor de esta magnitud para una muestra de aire sintético a 25°C y 1 atm. Considere a este sistema como una mezcla de 80 % V/V de N2 y 20 % V/V de O2. B.- Una mezcla gaseosa contiene la siguiente composición en peso por cada cien gramos de muestra (% p/p): Hidrógeno (6%), CO (68%), nitrógeno (11%), dióxido de carbono (14%) y metano. (i) Exprese la composición de esta mezcla en volumen por ciento (emplee la ley de Amagat) y fracción molar (ii) calcule el peso molecular medio de la mezcla (iii) la densidad a 400 °C y 1,5 atm.
  • 3. SEMINARIOS DE FISICOQUIMICA –INGENIERÍA QUÍMICA (PRIMERA PARTE) 3 PROBLEMA 5: APLICACIÓN DE LA LEY DE DALTON AL ESTUDIO DEL EQUILIBRIO QUÍMICO EN GASES. A.-En un recipiente se introducen n0 moles de una sustancia gaseosa A2. La presión y la temperatura son constantes en el experimento. La sustancia se descompone parcialmente según la estequiometria A2(g) 2A(g) hasta alcanzar el equilibrio. Si se disocian n moles de A2, y si se define el grado de disociación =n/n0, halle las expresiones para (1) el número de moles totales en equilibrio y el peso molecular promedio. Considere que el peso molecular de A2 es MA2 (2) las fracciones molares y presiones parciales de los gases en equilibrio (3) la constante de equilibrio en función de las fracciones molares y de las presiones parciales B.- Se ha determinado el peso molecular promedio <M> de una muestra de ácido acético a distintas temperaturas y 1 atm de presión. Los resultados se indican en la siguiente tabla: t/°C 124,8 134,8 144,8 154,6 164,8 <M> 90,35 87,00 81,99 77,73 74,14 El comportamiento de este sistema indica que el ácido acético debe estar asociado en fase gaseosa, de acuerdo con el equilibrio: 2CH3COOH(g) (CH3COOH)2(g). La dependencia de la constante de equilibrio con la temperatura está descripto por la ecuación de Van't Hoff, (lnKp/T)p=H /RT2 , donde H es la entalpía de reacción en condiciones de 1 atm de presión para reactivos y productos (1) empleando los datos del enunciado evalúe el grado de disociación del dímero a las distintas temperaturas (2) calcule la constante de equilibrio a estas temperaturas y evalúe la entalpía de reacción correspondiente (3) calcule la densidad de la mezcla gaseosa a la temperatura de ebullición normal del ácido acético (t/°C=118) C.- En un balón se introduce cloruro de amonio y se calienta hasta la temperatura de 596,9 K. En estas condiciones la presión es 0,253 atm y la densidad de 0,1373 g/dm3 . Analice en que extensión está disociado el reactivo. Evalúe el peso molecular promedio de la mezcla. PROBLEMA 6: ECUACION DE CLAUSIUS-CLAPEYRON. A.- Como discutiremos en otros capítulos, la presión de vapor (p) de equilibrio entre un líquido (L) y un vapor (v) satisface la ecuación de Clapeyron, m m V T H dT dp    donde 1 . 2 . m m m H H H    y 1 . 2 . m m V V V    son respectivamente los aumentos de entalpías y volúmenes molares de la sustancia en cada fase. Halle la forma de la ecuación de Clausius-Clapeyron en condiciones alejadas del punto crítico. Tenga presente que la densidad del vapor es bastante menor que la del líquido y admita que el vapor se puede describir como si fuese un gas ideal. B.- La ecuación de Clausius-Clapeyron toma la forma 2 ln RT H dT p d mV V   . Integre esta ecuación admitiendo que la entalpía molar de vaporización no depende de la temperatura. Vincule esta ecuación con la que usted emplea en el laboratorio sobre presión de vapor de una sustancia pura. C.- En general, la hipótesis asociada a la independencia de la entalpía de vaporización con la temperatura no es correcta. Como se discutirá más adelante, una buena aproximación de esta dependencia toma la forma vHm=C1+Cp,m.T, donde Cp,m es la diferencia de las capacidades caloríficas molares entre las fases gaseosa y líquida. Introduzca esta expresión para Cp,m en la de Clausius-Clapyron e integre en forma indefinida. Verifique que bajo esta consideración la presión de vapor del éter etílico obedece a la ecuación: T R B RT A C pV ln ln    . Encuentre la relación entre los coeficientes de la ecuación integrada y las constantes en la expresión de vHm. Explique cómo procedería a determinar C1 en la ecuación para vHm. D.- Se conocen valores de la presión de vapor del éter etílico con la temperatura. Las mismas se presentan en la siguiente tabla en un amplio intervalo de temperatura. t/°C -50,9 -10 0 20 34,5 50 70 100 120 pv/torr 8,77 112,3 184,9 439,8 760 1276 2304 4855 7531 (1) Empleando la integral definida entre dos temperaturas consecutivas en esta tabla, verifique que la entalpía de vaporización no es constante. Represente vHm como función de la temperatura media en cada intervalo. (2) Si se admite que la presión de vapor se describe con la ecuación T R B RT A C pV ln ln    , evalúe los parámetros A, B y C a partir de los datos de la tabla trabajando en el intercalo entre 0 y 50 C usando los siguientes procedimientos (i) resolviendo un sistema de tres ecuaciones con tres incógnitas, (ii) por análisis regresión. (3) Calcule la entalpía de vaporización a la temperatura de ebullición normal y la entropía de vaporización. (4) Analice si esta sustancia cumple con la regla de Trouton, según la cual la entropía de vaporización a esa temperatura se halla aproximadamente entre 84 y 96 J/K.mol (5) analice si las aproximaciones realizadas en la deducción de la ecuación indicada anteriormente permiten su aplicación en las cercanías del punto crítico. JUSTIFIQUE. m m V T H dT dp   
  • 4. SEMINARIOS DE FISICOQUIMICA –INGENIERÍA QUÍMICA (PRIMERA PARTE) 4 PROBLEMA 7: A.- 1 mol de éter etílico se introduce en un recipiente de 10 litros a 10°C en el que previamente se ha practicado vacío. (1) Halle la presión de vapor del éter etílico a 10 C empleando la información del Problema 6. (2) calcule cuantos moles de vapor pasan de la fase líquida a la fase vapor cuando se introduce el mol del éter en el recipiente. Suponga comportamiento ideal del vapor (3) ¿cuál debe ser el volumen del recipiente para que, al incorporar el mol del éter etílico líquido, éste se vaporice completamente. B.- Si el volumen del recipiente que contiene el mol de éter etílico fuese de 200 litros a 0°C, (1) ¿cuántas fases tiene el sistema? (2) En un diagrama P vs log V señale a escala como debe ser el comportamiento del sistema si se incrementa la presión de a 10 torr. Como información adicional y de utilidad para esta construcción, se conoce que la densidad del éter líquido a esta temperatura d=0,736 g/ml. PROBLEMA 8: A.- Dos balones de igual volumen, en los que se ha practicado vacío, se hallan conectados por un capilar de volumen despreciable. Inicialmente, se introduce nitrógeno a 200 °C y se lee una presión de 1 atm. Luego, se cierra el sistema y uno de los extremos se coloca en un recipiente con agua e hielo a 0°C y el otro en agua a 100°C. Calcular la nueva presión de equilibrio. B.- Una mezcla de ciclohexeno e hidrógeno, con esta última en exceso se hace reaccionar sobre platino como catalizador. La mezcla gaseosa, inicialmente ejerce una presión de 500 torr y la que sale del reactor una presión de 450 Torr ambas medidas en las mismas condiciones de volumen y temperatura. Si la reacción es completa, calcular la fracción molar de los reactivos en la mezcla inicial. C.-Un compuesto de fórmula C4H4(g) se quema en presencia de oxígeno en exceso, en un reactor a volumen constante. La presión inicial es de 2 atm, mientras que la presión final es de 1,9 atm. La mezcla con los productos y el oxigeno en exceso se hace pasar a través de un tren de secado, que retiene el agua. Se mide la presión después del secado y se lee una presión de 1,8 atm. Con la información suministrada, determinar la cantidad de moles de reactivos inicial y la temperatura a la cual se investigó esta reacción. La entalpía de vaporización del agua es de 42 kJ/mol. Considere que volumen del reactor es de 1 litro. PROBLEMA 6: Diagramas de fases y curvas de Andrews. Considere la sustancia agua. Para este compuesto se dispone de la siguiente información: A 25°C, la presión de vapor del agua en equilibrio con su líquido es 23,3 Torr. Su temperatura de ebullición normal es 100°C. Su temperatura, volumen y presión crítica valen 647.4 K, 55,3 cm3 .mol- 1 y 218 atm. Se sabe además que la densidad del agua líquida es aproximadamente 1 g.cm-1 a 25 y que el coeficiente de expansión cúbico   1x10-4 K-1 y el de compresibilidad   5x10-5 atm-1 . (a) Asimismo, estime la entalpía de vaporización y calcule la presión de vapor del agua a 150C y 200 C (suponga que el vHm no cambia con la temperatura). (b) Esquematice en un diagrama p versus volumen el comportamiento de las isotermas que describe esta sustancia en un amplio intervalo de volúmenes a las temperaturas de 25, 100, temperatura crítica y por encima de esta temperatura. (IMPORTANTE). Considere que  y  no dependen de p y de T. BIBLIOGRAFIA: 1.- Cualquiera de los textos generales de Fisicoquímica 2.- H. Avery & D.J. Shaw, Cálculos Básicos en Química Física 3.- H. Avery & D.J. Shaw, Cálculos Superiores en Química Física 4.- A. Woods, Problemas en Química Física 5.- Adamson, Understanding Physical Chemistry. Existe una versión en español.
  • 5. SEMINARIOS DE FISICOQUIMICA –INGENIERÍA QUÍMICA (PRIMERA PARTE) 5 CATEDRA DE FISICOQUIMICA I SEMINARIO 2 TEORIA CINETICA DEL GAS IDEAL PROBLEMA 1: A.- (1) Calcular el número de moléculas por unidad de volumen en un gas a 300 K a las presiones de 1 atm, 100 atm y 0.001 atm (2) ¿cuál es la presión a la que debería hallarse 1 mol de gas a 300 K para que su densidad sea comparable a la de un líquido (= 1 g, cm-3 ) y cuantas moléculas debería encontrar por unidad de volumen? Considere los gases hidrógeno, oxígeno y metano como ejemplos. B.- (1) ¿Qué entiende por espacio de velocidades? ¿Cómo lo construye? (2) ¿Qué entiende por moléculas de la clase c? Emplee para su explicación el espacio auxiliar de velocidades moleculares. C.- ¿Cuál es la fracción de moléculas, que independientemente de sus velocidades tienen valores de  comprendidos respectivamente entre 44,5 y 45,5  y de  entre 59,5 y 60,5. PROBLEMA 2: A.- (1) Calcular la energía cinética media de traslación de los siguientes gases a 300 K: hidrógeno, oxígeno y metano (1) calcular la velocidad cuadrática media de cada uno de los gases anteriores. B.- Calcular las temperaturas a la cual el oxigeno y el metano deben tener la misma velocidad cuadrática media. C.- La masa molar de un gas vale 60 g.mol-1 . En ciertas condiciones experimentales el número de moléculas por unidad de volumen es 4x1019 moléculas.cm-3 . Si la velocidad cuadrática media es de 400 m/s, calcular la presión del gas sobre la base de la expresión deducida a partir de la teoría cinética D.- ¿Cuál hubiera sido la expresión correspondiente para la presión total en términos de la teoría cinética para un sistema constituido por una mezcla gaseosa ideal? PROBLEMA 3: Cuando una molécula choca elásticamente contra una pared inmóvil, el valor de la componente normal de la velocidad no varía. Analice la situación para el caso de una pared que se desplaza como consecuencia de las colisiones moleculares, tal como ocurre con un pistón. Considere por simplicidad un proceso adiabático y demuestre que la velocidad con que se le entrega trabajo mecánico al émbolo es exactamente igual a la velocidad de disminución de la energía molecular con la consecuente disminución de la temperatura del gas. (Consultar el texto de F. Sears, Termodinámica e Introducción a la Teoría Cinética de Gases y Termodinámica Estadística, 1ra Edición) PROBLEMA 4: A.- (1) ¿Cuántos impactos moleculares recibe por segundo 1 cm2 de superficie cuando se lo expone al aire a la presión de 1 atm a 300 K (2) ¿Cuál será la altura de un cilindro de la misma área que contiene este número de moléculas a 1 atm y 300 K? B.- Un recipiente cerrado contiene agua líquida en equilibrio con su vapor a 100 C. En estas condiciones 1 g de vapor ocupa un volumen aproximado de 1670 cm3 . La entalpia de vaporización del agua a esta temperatura es de 2250 julios/g. (1) ¿cuántas moléculas hay en 1 cm3 de vapor? (2) ¿Cuántas moléculas de vapor alcanzan cada cm2 de superficie de líquido cada segundo? (3) Comparar la energía cinética media de las moléculas en fase gaseosa con la energía requerida para transferir a esta moléculas desde la fase líquida a la fase vapor? Discutir el resultado. PROBLEMA 5: A.- Un recipiente de paredes delgadas de volumen V que se mantiene a temperatura constante, contiene un gas que escapa lentamente a través de un orificio pequeño de área A. La presión en el exterior es lo suficientemente baja como para que las moléculas no retornen al recipiente. Hallar una expresión que permita conocer la dependencia de la presión en el interior del recipiente como función del tiempo. B.- Un recipiente está dividido en dos compartimientos de volúmenes iguales mediante un tabique de paredes delgadas. En uno de ellos se hace vacío, mientras que en el otro se llena con un gas hasta la presión P0. Si en un instante de tiempo que tomamos como el inicial se practica una abertura pequeña de área A, deducir una expresión que de cuenta de la evolución de la presión en los compartimientos como función del tiempo. C.- En una línea de vacío, cuyo volumen es 1 litro, la presión se reduce a 0,001 torr. En un instante dado, se produce una abertura de 10-6 cm2 de área, y comienzan ingresar las moléculas presentes en el aire a temperatura ambiente. Suponiendo que las paredes son delgadas, hallar una expresión para la dependencia de la presión en el interior del recipiente como función del tiempo. D.- En un recipiente de características similares a la del inciso B, se le aísla adiabáticamente, de manera que no se transfiera calor de un compartimiento al otro. Uno de los compartimientos se mantiene a la temperatura T1 y el otro a la temperatura T2. ¿Qué relación existe entre las presiones de ambos compartimientos cuando se alcanza el equilibrio? Discuta el resultado.
  • 6. SEMINARIOS DE FISICOQUIMICA –INGENIERÍA QUÍMICA (PRIMERA PARTE) 6 PROBLEMA 6: Evaluación de la presión de equilibrio entre una fase condensada y un vapor. A.- El fenómeno de efusión puede emplearse para medir la presión de vapor de sólidos o de líquidos de alta temperatura de fusión. Holden et al, determinaron la presión de vapor del Be sólido por este método. El orificio de efusión fue de 0,318 cm de diámetro y se encontró una pérdida de peso de 9,54 mg en 60,1 minutos de tiempo a 1457 K. Calcule la presión de vapor. (Rpta: 9,55x10-6 atm) B.- Se tiene un recipiente que contiene una mezcla de H2(g) y de O2(g) en la relación estequiométrica para dar agua. La presión inicial de la mezcla es de 1 atm a 300 K. El volumen del recipiente es de 1 L. Calcule la relación de moléculas de H2 a O2 que pueden escapar por efusión sin retornar al sistema al momento de generarse una abertura de 0,01 cm2 . C.- Se ha medido la velocidad de efusión (dw/dt=w/t) de Colesterol a distintas temperaturas (V. Oja et al, J. Chem. Eng. Data 2009, 54, 730–734). La abertura a través de la cuál estas moléculas escapan del sistema en equilibrio sólido-vapor (sublimación) tiene un diámetro de 0,65 mm. Los valores obtenidos a distintas temperaturas se muestran en la siguiente tabla y el peso molecular del compuesto es 386,65. T/K 386,6 392,0 397,3 403,1 408,2 413,8 dw/dt (g.s-1 ) 4,302x10-8 7,724x10-8 1,420x10-7 2,548x10-7 4,439x10-7 7,564x10-7 (1) Evaluar la presión de vapor y la entalpía de sublimación (valor de literatura 142.5 kJ.mol-1 ) recurriendo a la ecuación de Clausius-Clapeyron. Recuerde además que la presión de vapor/sublimación en estas condiciones se expresa como M RT A t w t A M M RT RT w p . 2 . . . . . 8 . . 4 *     La ecuación así aplicada no tiene en cuenta problemas de diseño experimental que deben ser incluidas en el cálculo. A los efectos del cálculo de la entalpía de sublimación, este factor no debe influir si no se modifica las condiciones de trabajo o se cambia el dispositivo con el que se realizan los experimentos. (2) La ecuación empleada requiere que las moléculas no choquen dentro de la abertura. En la literatura se han propuesto distintas correcciones para considerar factores geométricos asociados con el diseño del experimento y de la celda o recipiente de medida. Esta corrección tiene en cuenta estos factores. La relación entre la presión calculada y la experimental es el factor de Clausing C, de manera que la expresión corregida a emplear es la siguiente: M RT A t w C p . 2 . *   . Calcule el factor de Clausing sabiendo que a 397,3 K, la presión de sublimación del colesterol vale 0,0990 Pa. PROBLEMA 7: A.- Diferencie entre la distribución componentes de velocidades y distribución escalar de velocidades o velocidades moleculares. B.- Discuta al menos dos métodos experimentales para verificar la ley de distribución de velocidades de Maxwell. C.- Construir la funciones de distribución p(u), f(c) y de densidad de puntos representativos en el espacio de velocidades para oxígeno a 100, 300, 500 y 1000 K. D.- Para el caso de considerar 1 mol de moléculas, calcular (sin resolver integrales) cuántas de estas tienen (1) componentes de velocidad sobre el eje u entre  y 1,01 (2) ídem entre u y v comprendidas simultáneamente entre  y 1,01 (3) ídem, entre u, v y w comprendidas simultáneamente entre  y 1,01 . (4) ídem con velocidad c entre  y 1,01. PROBLEMA 8: A.- Calcular la fracción de moléculas que tienen (1) componente sobre el eje u de velocidades entre - y  (2) superiores a . Usar la tabla de la función error correspondiente. B.- Calcular la fracción de moléculas que tienen velocidades moleculares menores que  (ii) idem >. C.- Considere tres sistemas cada de los cuales contiene respectivamente hidrógeno, oxígeno y Hg. Calcule la fracción de moléculas que a 100, 300, 500 K superan el valor de c0=1000 m/s. Discuta el resultado. Represente el logaritmo de esta fracción vs 1/T y analice el resultado para los distintos gases. PROBLEMA 9: A.- Esquematice la función de distribución de energía para las moléculas de hidrógeno y oxígeno a 300 K. B.- Represente la función de distribución de energía para las moléculas de oxígeno a 100, 300 y 500 K. C.- Calcule la fracción de moléculas de (1) hidrógeno, (2) oxígeno y (3) metano que a superan la energía de 4184 J.mol-1 a 100, 300 y 500 K. Discuta los resultados. Represente el logaritmo de esta fracción vs 1/T y analice el resultado para cada gas. D.- Calcule la fracción de moléculas que a 300 y 310 K superan (i) la energía de k.300, donde k es la constante de Boltzmann (ii) ídem de 20.k.300 (el número 300 se refiere a la temperatura en la expresión kT).
  • 7. SEMINARIOS DE FISICOQUIMICA –INGENIERÍA QUÍMICA (PRIMERA PARTE) 7 PROBLEMA 10: A.- Enuncie el principio de equipartición de la energía. B.- Aplique este principio al cálculo de las capacidades caloríficas de (1) un gas monoatómico (2) un gas diatómico (3) CO2 (4) agua. En el caso de estos dos últimos ejemplos, estime cuantos modos normales de vibración están activos a 300 K. C.- Muestre que el principio de equipartición justifica la ley experimental de Dulong y Petit para sólidos monoatómicos, según la cual, CV,m=3R. PROBLEMA 11: El principio de equipartición falla en describir el comportamiento de las sustancias puras a bajas temperaturas. Empleando los conceptos simples asociados con la cuantización de la energía, explique las razones de esta discrepancia. ==================================================================== x TABLA DE LA FUNCION ERROR erf(x) = (1/2)1/2 exp(-x2 ).dx 0 x erf(x) x erf(x) x erf(x) x erf(x) x erf(x) x erf(x) 0.0 0.000 0.2 0.2270 0.5 0.5205 0.8 0.7421 1.2 0.9103 1.8 0.9890 0.1 0.1125 0.3 0.3286 0.6 0.6039 0.9 0.7969 1.4 0.9523 2.0 0.9953 0.4 0.4284 0.7 0.6778 1.0 0.8427 1.6 0.9763 3.0 0.9999
  • 8. SEMINARIOS DE FISICOQUIMICA –INGENIERÍA QUÍMICA (PRIMERA PARTE) 8 CÁTEDRA DE FISICOQUIMICA 1 SEMINARIO 3 GASES REALES PROBLEMA 1: A.- a 250 K y 15 atm un gas ocupa un volumen molar 12% menor que el calculado a partir de la ecuación de estado del gas ideal. Calcular (1) el factor de compresibilidad Z=pVm/RT (2) el volumen molar del gas.¿Qué tipo de fuerzas predominan en el comportamiento observado del gas? B.- A 300K y 20 atm, el factor de compresibilidad de un gas Z=0.86. Calcular: (1) el volumen que ocupan 8.2 milimoles del gas (b) un valor aproximado del segundo coeficiente del virial Bp (3) A partir de Bp evalúe los valores de A2, BV a esta temperatura. PROBLEMA 2: A una temperatura dada, el volumen molar ocupado por argón a distintas presiones se da en la siguiente tabla: p/atm 8,99 17,65 26,01 34,10 41,92 49,5 58,86 64,02 Vm/dm3 mol-1 2,00 1,00 0,667 0,500 0,400 0,333 0,275 0,250 Calcular: (1) la temperatura del gas y el valor Z a las distintas presiones. (2) representar Z vs P y comparar con el comportamiento ideal. (3) calcular el error relativo que se comete al calcular estos volúmenes con la ecuación del gas ideal. PROBLEMA 3: A.- Para 1 mol de metano a 25 °C, el desarrollo del virial en serie de potencias en 1/V para el factor de compresibilidad es: Z= 1 - 4.28x10-3 (1/Vm) +2.1x10-3 (1/Vm)2 +1.3x10-5 (1/Vm)3 . (1) Hallar la expresión de Z serie de potencias de p (2) que error comete al calcular el volumen ocupado por 1 mol de metano mediante la ecuación del gas ideal a 0.5, 10, 50 y 100 atm. (3) Hallar gráfica y analíticamente los valores de presión a los cuales Z=1 y Z toma su valor mínimo. B.- Los coeficientes del virial del metano a distintas temperaturas se presentan en la siguiente tabla t/°C 0 25 50 100 150 200 250 300 350 Bv10² -5.34 -4.28 -3.42 -2.1 -1.11 -0.42 0.149 0.598 0.97 Cv103 2.39 2.1 2.15 1.834 1.64 1.542 1.42 1.36 1.33 Dv105 26 15 1.3 2.7 3.5 4.3 5.2 5.7 5.9 Cp106 ---- 0.45 ? 1.486 ? 1.252 0.993 ? ? Dp109 30 18 8.2 4.3 2.1 1.03 0.58 0.32 0.17 Las unidades son: Bv/L mol-1 , Cv/L²mol-2 , Dv/L3 mol-3 , Cp/atm-2 , Dp/atm-3 . (1) Calcular los valores de Bp y Cp correspondientes al desarrollo de Z en potencias de p. Se puede probar (ejercicio optativo) que Cp=Cv/(RT)2 – (Bv/RT)2 (2) encontrar los coeficientes del desarrollo del producto pVm en serie de potencias en p y en 1/Vm. (3) determine la temperatura de Boyle de esta sustancia representado Bv vs T. (4) represente Z vs p a una temperatura inferior, a una igual y a una superior a la de Boyle. (6) encuentre una expresión que le permita relacionar la densidad del gas real con la correspondiente al gas ideal (7) represente Bv vs 1/T y discuta el comportamiento observado (8) Muestre por análisis de regresión que un ajuste de Bv/Vc= a + a1/Tr + a2/Tr 2 es también una representación adecuada del comportamiento de Bv, donde Vc es el volumen crítico molar y Tr es la temperatura reducida T/Tc. Calcule las constantes de esta ecuación (9) Por regresión no lineal, muestre que Bv/Vc=  + 1 . exp(3/Tr) representa adecuadamente el comportamiento de este coeficiente. Calcule las constantes de esta relación (10) Halle una expresión que relacione las constantes k y los parámetros ak de las expresiones planteadas en los incisos 8 y 9. PROBLEMA 4: La densidad del NO a bajas presiones y a 0°C se listan a continuación P/atm 1.000 0.8000 0.5000 0.3000 /g L 1.3402 1.0719 0.66973 0.40174 (1) Determine el peso molecular del gas por el método de las densidades limites (Represente P/ vs P). (2) Estime el factor de compresibilidad a 10 atm. PROBLEMA 5: A.- En la Tabla 1 que se adjunta, se listan los valores de las constantes críticas de distintos gases. Calcular el factor de compresibilidad de esas sustancias en el punto crítico. Enuncie la ley de los estados correspondientes. B.- (1) Sobre el diagrama de compresibilidad generalizado (Figura 1) represente el comportamiento del metano a distintas temperaturas reducidas. (2) Verifique que a la misma temperatura reducida, el segundo coeficiente del virial reducido Bp,r es el mismo para los gases que cumplen la ley de los estados correspondientes. C.- Determine la presión y la temperatura a la cual un mol de (1) NH3 (2) Xe (3) CH4, se comportan en estados correspondientes a un mol de O2 a 25C y 1 atm.
  • 9. SEMINARIOS DE FISICOQUIMICA –INGENIERÍA QUÍMICA (PRIMERA PARTE) 9 D.- Verifique que el segundo coeficiente del virial reducido (Bv/Vc) para los gases de la tabla 2 cumplen con la ley de los estados correspondientes. PROBLEMA 6: A.- La ecuación de Van der Waals es una ecuación de estado a 3 constantes:(p+a/Vm²)(Vm-b)=RT por mol de sustancia. (1) Empleando los datos de la Tabla 1, calcule las constantes a y b (2) Muestre que el modelo no permite calcular la constante general de los gases. Compare el valor calculado con el valor experimental (3) Como Zc=8/3 para este gas, el Vc no es una magnitud independiente y se puede calcular como Vc=3.RTc/8pc empleando las otras propiedades criticas. Con esta consideración, reevalúe las constates a y b del modelo, reemplazando Vc por esta última expresión. B.- Desarrolle la ecuación de Van der Waals en la forma del virial y encuentre las expresiones de los coeficientes del virial en P y en 1/V. C.- Calcular la temperatura de Boyle de estas sustancias y cuando sea posible, compare con valores experimentales. D.- Represente el comportamiento de Bv en función de T para el metano en términos de este modelo y compare con los valores experimentales, indicados en el Problema 3. E.- Escriba la ecuación de Van der Waals en términos de la ley de los estados correspondientes. PROBLEMA 7: A.- Estimar el volumen molar del Cl2 a 300 K y 2,3 atm utilizando la ecuación de los gases ideales y la ecuación de Van der Waals. B.- Calcular el radio de la molécula de cloro a partir del conocimiento del co-volumen indicado en la tabla adjunta. C.- Calcular la presión que ejercen 0,8 moles de N2 en una vasija de 100 litros a 25C. PROBLEMA 8: OTRAS ECUACIONES DE ESTADO A.- Halle la expresión para el segundo coeficiente del virial para un gas que obedece la ecuación de estado de Berthelot. Encuentre para este gas la expresión simplificada de Z en términos de las variables reducidas B.- Halle la expresión para el segundo coeficiente del virial para un gas que obedece de Dierterici. Muestre que Bv es el mismo que para el gas de Van der Waals. C.- La ecuación de Redlich-Kwong es la siguiente; RT b V b V V T a p m m m RK       ) ( ) ) ( ( 2 1 con 3 1  c Z . Analice si es posible hallar la expresión para el segundo coeficiente del virial para un gas que obedece la ecuación de Redlich-Kwong. Los coeficientes de esta ecuación valen c c RK p T R a 2 5 2 4275 , 0   y c c p T R b   086645 , 0 . PROBLEMA 9: Si el comportamiento de un gas real a 2 temperaturas distintas admite los desarrollos para el producto PV: T1 PV= 10.086 - 0.10 P + 4.0 10-5 P² T2 PV= 49.99 + 0.0094 P + 6.0 10-5 P² Estime (1) la temperatura de Boyle. (2) las constantes a y b de Van der Waals (3) su peso molecular, si la densidad del gas a 1 atm y 0C vale 0.0011g/cm3 . PROBLEMA 10: Empleando la ecuación de Rayleigh para el segundo coeficiente del virial Bv = 2  No   (1-exp(-/kT))r² dr discutida en la clase teórica, verifique A.- Que el modelo de Van der Waals se caracteriza por un potencial del tipo  para r< (r)= -A/r6 para r> (1) Sobre la base de las constantes a y b de Van der Waals, estime las constantes  y A de este modelo para alguno de los gases indicados en la Tabla 1 y contraste con la información experimental disponible o existente en tablas (2) Evalúe el potencial de interacción entre las moléculas -A/6 a la distancia de máximo acercamiento entre las esferas duras- y correlacione el potencial de interacción atractivo a  y la entalpías de vaporización, temperaturas de ebullición, etc. (3) ídem entre el valor de la constante A y la polarizabilidad molecular. B.- Para un modelo de potencial (de pozo) del tipo  para r< (r)= -u para <r<1,5  con u una constante que represente el valor del potencial en el pozo 0 para r>1,5 
  • 10. SEMINARIOS DE FISICOQUIMICA –INGENIERÍA QUÍMICA (PRIMERA PARTE) 10 puede obtenerse la siguiente expresión para el segundo coeficiente del virial (expresado como coeficiente del virial reducido) Bv/Vc = 0.438 - 0.881 Tc/T - 0.757 Tc²/T² Represente el comportamiento indicado por esta ecuación y compare con el correspondiente para el metano. Discuta el significado de estas correlaciones (Compare el comportamiento con el indicado en la Figura 3). PROBLEMA 11: Analice el comportamiento de la presión de vapor reducido y describa que debe esperar de la entalpía de vapor de las sustancias en estado reducido (Figura 4) ==============================TABLAS Y FIGURAS================================ TABLA 1: Parámetros críticos y temperaturas de Boyle de distintos gases Gas Pc/atm Vc/ml.mol-1 Tc/K TBK Gas Pc/atm Vc/ml.mol-1 Tc/K TBK He 2.26 57.76 5.21 22.64 Br2 102.0 135.0 584.0 Ne 26.86 41.74 44.44 122.1 HCl 81.50 81.0 324.7 Ar 48.00 75.25 150.7 411.5 BrH 84.0 -- 363.0 Kr 54.27 92.24 209.4 575.0 IH 80.8 -- 423.2 Xe 58.00 118.8 289.8 768.0 CO2 72.85 94.0 304.2 714.8 H2 12.8 64.99 33.23 110.0 H2O 218.3 55.3 647.4 N2 33.54 90.10 126.3 327.2 NH3 111.3 72.5 405.5 O2 50.14 78.00 154.8 405.9 CH4 45.6 98.7 190.6 510.0 F2 55.00 -- 144.0 C2H4 50.50 124.0 283.1 Cl2 76.1 124.0 417.2 C2H6 48.20 148.0 305.4 C6H6 48.6 260.0 562.7 TABLA 2: Segundo coeficiente del virial Bv (ml/mol) para distintos gases y temperaturas Gas 100 K 200 K 373 K 600 K He 11.4 12.0 11.3 10.4 Ne -4.8 10.4 12.3 13.8 Ar -187.0 -21.7 -4.2 11.9 Kr -62.9 -28.7 2.0 Xe -153.7 -81.7 -19.6 H2 -2.5 13.7 15.6 -- N2 -160.0 -10.5 6.2 21.7 O2 -197.5 -22.0 -3.7 12.9 CO2 -149.7 -72.2 -12.4 CH4 -53.6 -21.2 8.1 aire -167.3 -13.5 3.4 19.0 TABLA 3: Constantes a y b de Van der Waals Gas a /atm.L-2 .mol2 b /ml.mol-1 Gas a /atm.L-2 .mol2 b /ml.mol-1 He 0.03412 23.7 CO2 3.592 42.67 Ne 0.2107 17.09 SO2 6.714 56.36 Ar 1.345 32.19 H2O 5.464 30.49 Kr 2.318 39.78 H2S 4.431 42.87 Xe 4.194 51.05 NH3 4.174 37.07 H2 0.2444 26.61 CH4 2.253 42.78 N2 1.390 39.13 C2H4 4.471 57.14 O2 1.360 31.83 C2H6 5.489 63.80 Cl2 6.493 56.22 C6H6 18.00 115.4 CO 1.485 39.85
  • 11. SEMINARIOS DE FISICOQUIMICA –INGENIERÍA QUÍMICA (PRIMERA PARTE) 11 Figura 1: Diagrama de compresibilidad generalizado (Gouq-Jen Su, Ind. Eng. Chem., 38 (1946), 803) Figura 2: Estados correspondientes en gases simples (Tomado de E.A. Guggenheim, Termodinámica, Editorial Tecnos, pg. 177, 1970)
  • 12. SEMINARIOS DE FISICOQUIMICA –INGENIERÍA QUÍMICA (PRIMERA PARTE) 12 Figura 3: Segundo Coeficiente del virial reducido (Bv/Vc) versus la temperatura reducida (T/Tc). La curva inferior expande el comportamiento del segundo coeficiente del virial reducido para temperaturas reducidas menores a la unidad. Figura 4: Comportamiento de la Presión de vapor reducida (p*/pc) en función de la inversa de la temperatura reducida. Observar que esta magnitud satisface la ley de los estados correspondientes.
  • 13. SEMINARIOS DE FISICOQUIMICA –INGENIERÍA QUÍMICA (PRIMERA PARTE) 13 CATEDRA DE FISICOQUIMICA I SEMINARIO 4 PRIMER PRINCIPIO DE LA TERMODINAMICA PROBLEMA 1: CALCULO DE TRABAJO DE EXPANSION EN DISTINTOS SISTEMAS. A.- Calcule el trabajo ejecutado cuando 50g de Fe se disuelven en HCl en: (1) un recipiente de volumen constante (2) un recipiente abierto a la presión de 1 atm. Considere que el experimento se realiza a 300 K. B.- Un mol de CaCO3 se calienta a 700 C. En esas condiciones de temperatura la sustancia se descompone completamente. La reacción se lleva a cabo en un recipiente provisto de un pistón móvil, inicialmente apoyado sobre la muestra y sometido a una presión de 1 atm. ¿Cuál es el trabajo ejecutado durante la descomposición? ¿Cuál es el trabajo si el proceso se realiza en un recipiente abierto? C.- Calcular el trabajo realizado cuando 1 mol de agua se: (1) calienta desde 25 a 55 C a 1 atm de presión (2) se incrementa la presión desde 1 a 30 atm a 25 C. Los coeficientes de expansión isobárico () y de compresibilidad isotérmico () valen /K-1 =2.1 x 10-4 y /atm-1 =49.4 x 10-6 respectivamente. Considere la densidad de agua =1g/cm3 . D.- Calcular el trabajo de expansión asociado: i) a la evaporación de 1g de agua líquida a 100 C y 1 atm ii) a la fusión de 1g de hielo a 0 C. La densidad del hielo es =0.9998g/cm3 . E.- A 300K los coeficientes de expansión isobárico para cobre y para neón valen respectivamente =5.01 x 10-5 K- 1 y =3.3 x 10-3 K-1 . ¿Cuál es el aumento de volumen cuando 50 cm3 de ambas sustancias se calientan en 5 C? ¿Cuál es el trabajo ejecutado si el calentamiento se realiza a 1 atm de presión? F.- ¿Cuál es el trabajo de expansión que realiza 1 mol de un gas cuyo comportamiento PVT a 0 C puede expresarse por la ecuación Z = 1 - 21.7 x 10-3 (1/Vm) + 1200 x 10-6 (1/Vm)². (1) si se somete a una expansión en el vacío desde un volumen de 5 litros hasta que su volumen se triplica. (2) ídem pero contra un presión externa de 1 atm. (3) se somete a una expansión reversible desde 5 litros hasta que su volumen se triplica. G.- repita los cálculos del inciso anterior considerando que el gas es ideal. PROBLEMA 2: Se desea diferenciar una transformación reversible de otra que no lo es. Para ello se imagina el siguiente proceso. Un mol de gas ideal se confina a un recipiente cilíndrico de altura suficiente. La tapa superior es un pistón que por si solo ejerce una presión de 1 atm. Sobre él se colocan 2000 pesitas idénticas, de forma que la presión total, incluido el pistón, sea de 2 atm. El sistema se halla en contacto con una fuente a 300K y por simplicidad exteriormente se ha practicado vacío, de manera tal que al retirarse todas las pesas, la presión del gas es de 1 atm. Con el sistema en esas condiciones se realizan los siguientes procesos -en todos los casos se parte siempre de las condiciones iníciales- (1) se retiran todas las pesas y se libera el pistón. (2) se retira la mitad de las pesas en una primera etapa y luego se retiran las pesas restantes. (3) se retiran números idénticos de pesas en cuatro etapas. (4) se retiran números idénticos de pesas en diez etapas. (5) se retiran las pesas de a una en cada etapa. Calcular el trabajo ejecutado en cada etapa y representarlo en un diagrama p-V. Discuta los resultados y compárelos con el trabajo que se hubiera ejecutado en un proceso estrictamente reversible. PROBLEMA 3: Considere un sistema similar al señalado en el problema anterior. Una vez que el sistema ha alcanzado el estado final, se retorna a las condiciones iniciales incorporando las pesas en cantidades similares a las retiradas en cada etapa. Represente en un diagrama P vs V. Discuta el resultado. PROBLEMA 4: El rendimiento en un ciclo de Carnot es independiente de su tamaño y de la naturaleza de la sustancia que se emplea. Verifique estas dos condiciones considerando que entre 400 y 200K se hace funcionar una máquina de Carnot: A.- empleando un gas que obedece a la ecuación del gas ideal. (1) Suponga que en la etapa de expansión isotérmica el volumen se duplica, (2) Suponga que en la expansión isotérmica el volumen se quintuplica. B.- empleando un gas que obedece a la ecuación del gas de esferas duras o rígidas. Para este gas (dU/dV)T=0. Suponga condiciones similares a las de los incisos del punto A. C.- empleando un gas que obedece a la ecuación de Van der Waals. Para este gas (dU/dV)T=a/Vm². Suponga condiciones similares a las de los incisos del punto A. PROBLEMA 5: Un mol de una sustancia se somete a un ciclo de Carnot (a b c d) en la región líquido-vapor tal como se detalla en la figura adjunta. En el estado -a-, se tiene que la sustancia se halla en estado líquido mientras que en el estado -b- se ha vaporizado completamente. La presión de vapor es de 3 atm y la temperatura de 400K. La expansión adiabática se realiza hasta que la presión de vapor se reduce a 1 atm. En el estado -c-, el sistema se halla constituido por un líquido en equilibrio con su vapor, cuyo título en vapor es 0.97. Si la entalpía de vaporización de
  • 14. SEMINARIOS DE FISICOQUIMICA –INGENIERÍA QUÍMICA (PRIMERA PARTE) 14 esa sustancia es de 10 Kcal/mol y no se modifica con la temperatura, calcular: (1) el rendimiento de ciclo. (2) el trabajo ejecutado en las etapas isotérmicas. (3) el trabajo total en las etapas adiabáticas. (4) el trabajo en el ciclo. (5) el título de vapor en el estado -d-. PROBLEMA 6: Calcular el aumento de U y de H que acompaña a la transformación irreversible de Hg sobreenfriado a -45 C en Hg sólido a esa temperatura y 1 atm de presión. La temperatura de fusión de esta sustancia es -39 C y la entalpía de fusión de 560 cal/mol. Se conocen las capacidades caloríficas del sólido (CP(s)= 6.4 cal K-1 .mol-1 ) y del líquido (CP(l)= 7.1 - 1.6x10-3 T cal K-1 .mol-1 ). PROBLEMA 7: A.- un mol de gas ideal se somete a una expansión isotérmica contra una presión externa constante e igual a 1 atm desde una condición inicial de 10 atm y 300K hasta que su volumen se triplica. Calcular q, w, y los aumentos de U y H. B.- ídem para un gas que obedece a la ecuación de estado P(V-b)=RT. (b=0.038 lt/mol) C.- calcular q, w, y los aumentos de U y H para un gas de Van der Waals, desde un volumen inicial idéntico al calculado en el punto -a- hasta que su volumen se triplica. Las constantes del modelo a aplicar valen b=0.038 L.mol-1 y a=1.39 L².atm.mol-2 . PROBLEMA 8: Analizar (1) la expansión libre del gas ideal. (2) la expansión libre del gas de esferas rígidas. (3) la expansión isotérmica en el vacio del gas de Van der Waals. (4) la expansión adiabática en el vacío del gas de Van der Waals. Emplee Cv y Cp correspondiente al gas monoatómico. PROBLEMA 9: A.- un mol de gas ideal se somete a una expansión adiabática contra una presión externa constante e igual a 1 atm desde una condición inicial de 10 atm y 300K hasta que su volumen se triplica. Calcular q, w, y los aumentos de U y H. B.- ídem para un gas que obedece la ecuación de estado P(Vm-b)=RT. (b=0.038 l/mol) C.- calcular q, w, y los aumentos de U y H para un gas de Van der Waals, desde un volumen inicial idéntico al calculado en el punto -a- hasta que su volumen se triplica. Las constantes del modelo a aplicar valen b=0.038 L.mol-1 y a=1.39 L².atm.mol-2 . PROBLEMA 10: La diferencia entre Cp y Cv está dado por la relación Cp-Cv = [ P + (U/V)T]( V/T)P. Sobre la base de esta ecuación A.- calcular el coeficiente (U/V)T para Cu(s), sabiendo que Cp-Cv = 0,159 J/mol a 1 atm y 25°C, =4,96 10-6 K-1 y =8,93 g.cm-3 . B.- estime el trabajo y el aumento de energía interna cuando 1000 g de este material se comprimen aplicando una presión externa constante de 100 atm a 25°C. PROBLEMA 11: a e b c f d P/atm V T1=400 K T2 3 1
  • 15. SEMINARIOS DE FISICOQUIMICA –INGENIERÍA QUÍMICA (PRIMERA PARTE) 15 A.- Un mol de gas ideal se somete a un proceso reversible no especificado desde sus condiciones iniciales a 0°C y 1 atm. En este proceso, el volumen del sistema se duplica, observándose un H=500 cal/mol, mientras que el calor puesto en juego es de 400 cal/mol. Calcule la temperatura final, el U y el trabajo realizado por el sistema en este proceso. B.- Un mol de un gas ideal se somete a una transformación cíclica, como se indica en la siguiente diagrama P-V. Calcular, empleando el menor número de relaciones termodinámicas, (1) el trabajo y el calor neto en el ciclo (2) el H y U entre C y A (3) el calor absorbido al pasar de A  C por el camino ABC (4) ídem ADC. Observe que H no coincide con los valores de los calores calculados en los incisos (3) y (4) a pesar de que la presión inicial y final es la misma. Justifique su respuesta. PROBLEMA 3: Las entalpías de combustión del grafito y del diamante valen respectivamente –94,030 y –94,484 kcal.mol-1 . Las densidades son 2,250 y 3,513 g.cm-3 respectivamente. Calcular, U, H, q y w para la transformación: diamante  grafito a 25°C. A B C D 3 2 1 P/atm 1 2 3 V/l.mol-1
  • 16. SEMINARIOS DE FISICOQUIMICA –INGENIERÍA QUÍMICA (PRIMERA PARTE) 16 CATEDRA DE FISICOQUIMICA I SEMINARIO 5 SEGUNDO PRINCIPIO DE LA TERMODINAMICA PROBLEMA 1: Trabajando con la información pertinente de la literatura, explique, plantee y/o calcule el aumento de entropía asociado a los problemas del seminario previo. PROBLEMA 2: A.- 1 kg de agua evoluciona según un ciclo de Carnot en la región líquido-vapor, entre 180 y 40°C. De tablas de vapor de agua disponibles en la literatura se dispone la siguiente información (Los estados que se mencionan corresponden a los que se muestran en al figura correspondiente del seminario anterior. estado t/°C p/ kg.cm2 (U-Uo ) kcal/kg (H-Ho) kcal/kg S kcal/K.kg a 180 10,225 181,9 182,2 0,5107 b 180 10,225 616,7 663,2 1,5721 e 40 0,0752 39,98 39,98 0,1366 f 40 0,0752 580,3 614,7 1,9718 (1) Calcular el rendimiento del ciclo,(2) calcular U, H, q y w en cada etapa del ciclo a  b  c  d, y el título del vapor en c y d. (3) calcular S del sistema y del exterior en cada etapa del ciclo. (4) Si el vapor, inicialmente en el estado b sufre una expansión irreversible hasta que la temperatura es 40°C, y se observa que, en valor absoluto, el título del vapor difiere en un 5% respecto del valor que hubiera observado para el proceso b  c, calcular el S del sistema, del exterior y del universo en esta expansión adiabática irreversible. PROBLEMA 3: A.- Una muestra de Al(s) de 1.75 Kg de masa se enfría reversiblemente a presión constante desde 300K a 265K. ¿Cuál es el aumento de entropía en el sistema? B.- Un bloque de cobre de 500g de masa inicialmente a 293K está en contacto térmico con un calentador eléctrico de 1000 ohm de resistencia y masa despreciable. Durante 15 seg circula una corriente eléctrica de 1 A. ¿Cuál es el cambio de entropía del Cu? CP= 24.4 J/mol.K C.- Si el bloque anterior se lo sumerge en una corriente de agua que se encuentra a 293K, cual es el cambio de entropía de (1) el metal. (2) del agua. PROBLEMA 4: Analice si es posible que (1) una isoterma y una adiabática pueden interceptarse más que una vez (2) una isoterma y una adiabática puedan tener más de punto de contacto (3) dos adiabáticas no pueden interceptarse mutuamente. PROBLEMA 5: A.- Sea S la expansión de un gas real referida a las condiciones de gas ideal a la presión de 1 atm y sea S la entropía experimental a la misma presión.(1) deduzca una expresión que vincule ambas cantidades a la misma presión y temperatura. (2) aplique la ecuación hallada en el inciso anterior para evaluar la diferencia S -S para el gas nitrógeno a su temperatura de ebullición trabajando con la ecuación de Berthelot. Considere que el factor de compresibilidad a bajas presiones según este modelo de gas puede escribirse como: r r r p T T Z ) 6 1 ).( 128 1 ( 1 2     . B.- Calcule la diferencia de entalpía A 25 °C de 1 mol de O2(g) a 1 atm de presión y del mismo gas en las condiciones ideales. Emplee la ecuación previa. C.- Calcule la diferencia de CP-CV a 25 °C y a las presiones de 1, 10 y 200 atm para O2. Considere el modelo de gas de Van der Waals. PROBLEMA 6: A.- 10g de hielo a 0 °C se agregan a 25g de agua a 90 °C en un vaso Dewar. El calor de fusión del hielo es de 80 cal/g. Si se desprecia la capacidad calorífica del termo, calcule la temperatura final y los aumentos de entalpía y de entropía del sistema, fuentes y universo. Considere CP=18 cal/K.mol B.- Una masa m de agua a la temperatura T1 se mezcla adiabática e isobáricamente con otra masa igual de agua a T2. Demuestre que S = 2 m CP ln (0.5(T1+T2)/(T1T2)1/2 ).
  • 17. SEMINARIOS DE FISICOQUIMICA –INGENIERÍA QUÍMICA (PRIMERA PARTE) 17 EFECTO JOULE-THOMSON PROBLEMA 7: A.- Encuentre una expresión que le permita determinar el coeficiente de Joule-Thomson en términos de los coeficientes del virial. Demuestre que para un gas real, este coeficiente no es necesariamente nulo a bajas presiones y aún a p  0. Contraste este comportamiento con un gas ideal. B.- Encuentre una expresión para el coeficiente de Joule-Thomson en términos de los parámetros a y b del gas de Van der Waals. C.- El valor del coeficiente de Joule-Thomson para el amoníaco a 300 °C y 40 atm es de 0.370 K.atm-1. Compare este valor con (1) los obtenidos a partir de los siguientes datos P-V-T para el amoníaco de 40 atm. En estas condiciones Cp,m=46,02 J.K-1 .mol-1 . t/°C 225 250 275 300 325 Vm/cm3 mol-1 962 1017 1076 1136 1186 (2) con los obtenidos a partir de la ecuación de Van der Waals (Rta ≈0,28 K.atm-1 ). PROBLEMA 8: A.- La variación del coeficiente de Joule-Thomson para aire a distintas temperaturas y 1 atm de presión está dado por: (dT/dp)H = J-T = -0.1975 + 138.3 - 319.0/T² en K.atm-1 Mientras que CP/cal/K.mol = 6.5 + 1 10-3 x T. Halle una expresión para (H/P)T como función de la temperatura. (1) Evalúe las temperaturas de inversión a 1 atm de presión para esta sustancia. (2) Encuentre una expresión que permita evaluar la curva de temperatura de inversión en términos de los coeficientes del virial. PROBLEMA 9: A.- En la figura 1 se presentan curvas isoentálpicas para N2. ¿Cuál es el cambio de temperatura que experimenta el gas cuando se lo somete al experimento de Joule-Thomson desde 400 atm y 300K hasta una presión de 1 atm. B.- En la siguiente tabla se presentan las temperaturas de inversión máxima y mínima para N2 a distintas presiones: p/atm 1 20 60 100 180 220 300 376 Tsup/ °C 348 330 299.6 277.2 235 212.5 158.7 40 Tinf/ °C -- -167 -162.4 -156.5 -134.7 -117.2 -68.7 40 Represente estos datos en un diagrama T-P y compárelos con la curva de inversión predicha sobre la base del modelo del gas de Van der Waals. C.- Al diseñar un refrigerador se ha empleado un freón cuyo coeficiente de Joule-Thomson vale 1.2K/atm. ¿Cuál es la diferencia de presión que debe emplearse en la expansión para que la temperatura del gas refrigerante disminuya en 5 °C. PROBLEMA 10: A.- Un cierto gas sufre una expansión de Joule-Thomson. En el proceso se observa que el trabajo neto es cero. ¿Puede asegurar que el gas se comporta idealmente? B.- Calcule el aumento de S que experimenta el gas ideal en ese experimento, cuando se lo expande desde 10 atm y 300K hasta que la presión se reduce a 1 atm. ISOENTALPICAS Y CURVA DE INVERSIÓN PARA N2. OBSERVE EN ESTE DIAGRAMA LA POSICIÓN REGIÓN CRÍTICA Y DE EQUILIBRIO LÍQUIDO-VAPOR VÁLVULA DE EXTRANGULACIÓN E INTERCAMBIADOR DE CALOR EN RÉGIMEN ESTACIONARIO
  • 18. SEMINARIOS DE FISICOQUIMICA –INGENIERÍA QUÍMICA (PRIMERA PARTE) 18 CONDENSACIÓN DE UN GAS POR MEDIO DEL EFECTO JOULE-THOMSON. OBSERVE QUE EL MISMO GAS ENFRIADO ACTÚA COMO REFRIGERANTE DEL GAS QUE INGRESA AL SISTEMA
  • 19. SEMINARIOS DE FISICOQUIMICA –INGENIERÍA QUÍMICA (PRIMERA PARTE) 19 CÁTEDRA DE FISICOQUIMICA I SEMINARIO 6 ENERGÍA LIBRE PROBLEMA 1: A.- (1) Calcule el aumento de energía libre que experimenta 1 mol de gas ideal de 10 a 1 atm de presión a 300K, cuando se lo somete a la expansión libre. (2) Si el gas desde el estado final descrito en el inciso anterior se comprime isotérmicamente aplicando una presión externa constante de 10 atm a 300K, ¿Cuál es el aumento de G que experimenta el sistema? ¿Es éste proceso espontáneo? Justifique su respuesta. B.- El coeficiente de compresibilidad de un agua líquida vale =5 10-5 atm-1 a 25°C y 1 atm de presión. Calcular el aumento de energía libre G que acompaña al proceso H2O (líq, 760 torr)  H2O(líq, pv= 23,70 torr) a 25°C. PROBLEMA 2: A.- Calcule el aumento de G que acompaña a un cambio de fase reversible. B.- En las proximidades del punto de transición las presiones de vapor del S(r) y S(m) -rómbico y monoclínico respectivamente- pueden expresarse como log p(r)/torr = 11.866-5267/T y log p(m)/torr = 11.364-5082/T (1) Hallar el punto de transición. (2) el aumento de S, H y G a esa temperatura. (3) el aumento de S, H y G a 70C . C.- La entalpía de fusión del agua vale 80 cal/g y las capacidades caloríficas del líquido y del sólido 1 y 0.5 cal/K respectivamente. Evalúe los aumentos de S y G para el proceso H2O (l, -5C, 1 atm) a H2O (s, -5C, 1 atm). PROBLEMA 3: A.- La entalpía estándar de formación para la reacción 2Hg(l)+Cl2(g)Hg2Cl2 es de -264.6 kJ.mol-1 . La energía libre de formación es de -210.2 kJ.mol-1 . Evalúe el aumento de entropía que acompaña a esta reacción. La información se da a 25C y 1 atm de presión. B.- Para la reacción C6H6(g)+3H2C6H12(g), G está dado por G /kJ.mol-1 =-179.3- 0.375T+ 0.12lnT- 1.05x10-4 T²+ 1.3x10-8 T3 . Evalúe los aumentos de S, H, y de CP a 400 K. C.- La FEM de la batería de plomo está dada por E/V = 1.91737 + 56.1x10-3 t + 1.08x10-6 t², con t en C. Evalúe G , S , H y CP  a 0; 25 y 60 C. D.- A 25C. la entalpía estándar de combustión de la glucosa es H=-5645 kJ/mol y la correspondiente función de Gibbs G =-5797 kJ/mol. Si la temperatura se eleva a 35C, ¿Cuál será el trabajo independiente de volumen w* por mol de reacción que debería obtenerse? PROBLEMA 4: A.- Suponiendo que el agua puede tratarse como un líquido incompresible, estime el aumento de energía libre cuando sobre 100 cm3 de ese líquido la presión se incrementa de 1 a 100 atm a 25C. ¿Cuál es el error que se comete al suponer que el volumen no cambia durante el proceso? La densidad del agua y el factor de compresibilidad isotérmico valen 0.997 g/cm3 y 4.94x10-4 atm respectivamente. B.- En la transición de aragonita a calcita a 25C y 1 atm, el volumen se incrementa en 2.73 mm3 /mol y la energía libre estándar disminuye en 0.794 kJ/mol. La entalpía de formación de la aragonita y de la calcita valen (en kJ/mol) -1207.8 y -1207.6 respectivamente . Calcular (1) el incremento de la función de Helmholtz (A) y el aumento de entropía. (2) ¿cuál es la presión necesaria para que la aragonita sea fase estable a 25C? C.- A 25C y 1 atm de presión, el aumento de entropía que acompaña a la transformación diamante  grafito es de 3.26 J.K-1 .mol-1 . Las entalpías de combustión de ambas sustancias pueden extraerse de tablas. Calcular el incremento en la función de Gibbs y en la función de Helmholtz. ¿Cuál es la forma estable en esas condiciones? D.- Calcular el incremento de energía libre, cuando 1 mol de agua es trasladado por destilación isotérmica a 25C, desde agua pura a una disolución al 9.9% de glicerina en agua, cuya presión de vapor de agua a esa temperatura es de 23.30 torr. La presión de vapor de agua pura es de 23.70 torr. PROBLEMA 5: FUGACIDAD: En estos ejercicios emplee métodos de regresión no lineal y/o el método de Simpson para resolver integrales numéricamente. A.- La fugacidad del oxígeno a 200K puede determinarse a partir del conocimiento del factor de compresibilidad Z a distintas presiones: p/atm 1 4 7 10 40 70 100 Z 0.99701 0.98796 0,97880 0.96956 0.8734 0,7764 0,6871 B.- El factor de compresibilidad del N2 a 0C a distintas presiones es de p/atm 50 100 200 400 800 1000 Z 0.9846 0.9846 1.0365 1.2557 1.7959 2.0641
  • 20. SEMINARIOS DE FISICOQUIMICA –INGENIERÍA QUÍMICA (PRIMERA PARTE) 20 Con esta información (1) representar la fugacidad vs p. (2) representar el coeficiente f/p vs p. (3) calcular el aumento de energía libre, cuando 1 mol de N2 se comprime de 1 a 1000 atm. ¿Cuál hubiera sido el error relativo en G, si se admite comportamiento ideal de la sustancia? C.- (1) enuncie la regla de Lewis para estimar la fugacidad de un componente gaseoso en una mezcla. (2) Merz y Whitaker (J.Am.Chem.Soc. 1925, 50, 1552) midieron a distintas presiones, el volumen molar parcial de hidrógeno a 0C en mezclas de este gas con nitrógeno. Los valores son: p/atm 50 100 200 300 400 Vm/ cm3 .mol-1 466.4 241.3 129.1 91.1 72.5 Calcule la fugacidad del hidrógeno a esa temperatura y 400 atm si estos datos corresponden a una mezcla de fracción molar 0,60 en H2. PROBLEMA 6: A.- Calcular la fugacidad del nitrógeno admitiendo que el gas cumple con (1) la ecuación de Berthelot. (2) de Van der Waals. La información que requiere la puede extraer de tablas. B.- Para un gas a 0C, PVm=RT(1.- 9.93x10-4 P + 2.19x10-6 P².). Calcular la fugacidad a 1, 10 y 50 atm y la presión a la que el volumen ideal coincide con el volumen real. Calcule además la energía libre del gas real comparada con la del gas ideal. Determine la presión a la cual estas magnitudes son iguales. C.- La presión de vapor del cloro líquido es de 3.66 atm a 0C y su volumen molar de 6.01 dm3 mol-1 . Calcular su fugacidad. D.- Calcular la fugacidad del vapor de agua a 25C, si su presión de vapor y su volumen molar son de 23.76 torr y 43.4 dm3 g-1 .
  • 21. SEMINARIOS DE FISICOQUIMICA –INGENIERÍA QUÍMICA (PRIMERA PARTE) 21 CÁTEDRA DE FISICOQUÍMICA I SEMINARIO 7 PROPIEDADES MOLARES PARCIALES Y PROPIEDADES COLIGATIVAS PROBLEMA 1: A.- Defina qué entiende por propiedad molar parcial (F) y propiedad molar aparente (F). B.- Para un sistema de dos componentes, F = a + b m + c m2 (m molalidad). Encuentre las expresiones correspondientes para las propiedades molares parciales de ambos componentes. C.- Para soluciones acuosas de electrolitos 1:1 valentes, la propiedad molar aparente F se expresa mas convenientemente en función de la raíz cuadrada de la molalidad F = a + b m1/2 . Hallar las propiedades molares parciales del solvente y soluto. PROBLEMA 2: A.- El volumen de una solución acuosa de NaCl a 25º C se expresa como una función de la molalidad según: V/ml mol-1 = 1002,9 + 16,40 m + 2,5 m2 - 1,2 m3 . Si el volumen molar del agua pura es 18,069 cm3 .mol-1 , hallar las expresiones correspondientes para el volumen molar parcial y el volumen molar aparente (1) del NaCl (2) agua. B.- Los siguientes datos corresponden a los volúmenes de soluciones de NaCl en agua a 25º C a distintas molalidades. m/mol kg-1 0 0,255 0,511 1,03 2,56 5,14 V/cm3 mol-1 1002,96 1007,45 1012,22 1022,11 1053,81 1112,36 Evalúe los volúmenes parciales del solvente y del soluto. C.- Cuando 1 mol de MgSO4 se disuelve en un gran volumen de agua, el volumen total disminuye en 1,4 cm3 /mol. ¿Cuál es el volumen molar parcial del soluto? Justifique el resultado. D.- Para evaluar propiedades molares parciales, puede recurrirse al denominado método de las ordenadas al origen. (1) Explique cómo se procede a su aplicación- ver por ejemplo el texto de Atkins, 3ra Edición- (2) Recurra a este método para tratar los siguientes datos que corresponden a las densidades de HNO3 en agua: 100 w 2,162 10,98 20,8 30,0 39,2 51,68 62,6 71,57 82,33 93,4 99,6  1,01 1,06 1,12 1,18 1,24 1,32 1,38 1,42 1,46 1,49 1,51 En la tabla w se da en fracción en peso de HNO3 y la densidad en g/cm3 . PROBLEMA 3: A.- La capacidad calorífica molar aparente de sacarosa está dada por: a (Cp) = 152,5 + 1,13 m - 0,0466 m2 cal / k mol Encuentre las expresiones para las capacidades molares parciales. La capacidad calorífica molar del agua es de 18,02 cal.K-1 mol-1 . B.- Para la disolución de NaCl en 1000 g de agua para dar soluciones de molalidad m, se midió la entalpía de solución (en cal/mol) a 18º C: m 1,0 0,64 0,36 0,25 0,09 0,04 0,01 H 1067 1141 1197 1212 1226 1220 1202 Calcule (1) la entalpía de solución de 1 mol de NaCl en un volumen muy grande de agua; (2) la entalpía molar parcial del NaCl para dar soluciones 0,04 y 0,25 molal. PROBLEMA 4: En la siguiente tabla se presentan valores asociados al proceso de disolución de una solución acuosa 0,1 M de NaCl. El cambio de entalpía correspondiente al proceso de agregar la cantidad suficiente de agua para diluir la solución desde una concentración 0,1 M hasta un valor c, se indica en esta tabla como H0,1c y son cambios de entalpía por mol de soluto. Evaluar el aumento de entalpía correspondiente al proceso de dilución infinita Hc0 de una solución de concentración c que contiene n2 moles de NaCl. c/ M 0,05 0,025 0,0125 0,00625 0,00305 0,00153 0,00076 0,00039 H0,1c/cal.mol-1 12,8 27,8 42,9 52,9 61,5 67,4 73,1 75,7 Evalúe las entalpías molares parciales correspondientes al solvente y soluto. PROBLEMA 5: A.- Que entiende por potencial químico? Defina esta magnitud en términos de G, A, H, U y S, especificando claramente cuáles son las variables que se deben fijar para que estas magnitudes puedan ser empleadas como definiciones alternativas del potencial químico. B.- Demuestre que al formar una solución ideal a partir de dos componentes líquidos, no deben observarse cambios de volumen, de energía interna ni de entalpía;
  • 22. SEMINARIOS DE FISICOQUIMICA –INGENIERÍA QUÍMICA (PRIMERA PARTE) 22 C.- Evalúe el aumento de entropía asociada a la mezcla de componentes líquidos en cantidades n1, n2, .., ni,.. para dar una solución ideal. Exprese este resultado por mol de mezcla y demuestre que siempre es positivo. PROBLEMA 6: A.- Un teorema importante del análisis matemático indica que una función f(x,y,z) es una función homogénea de orden m, si f(x,y,z)= m (f/x) + m (f/y) + m (f/z). Si m=1, la función se dice homogénea de orden 1. La energía libre G de un sistema G(n1, n2, ..., ni, ..) a T y P constantes es una función homogénea del número de moles de orden 1. Luego, G =  ni.i. Analice si este teorema asegura que, a relación de moles constantes, el valor de cada i es independiente del tamaño del sistema. (Lea un texto de análisis matemático) B.- Demuestre que i = Hi – T.Si, donde las magnitudes presentes en el segundo miembro de esta ecuación representan la entalpía y la entropía molar parcial del i-ésimo componente.
  • 23. SEMINARIOS DE FISICOQUIMICA –INGENIERÍA QUÍMICA (PRIMERA PARTE) 23 CATEDRA DE FISICOQUIMICA SEMINARIO 8 SOLUCIONES IDEALES-PROPIEDADES COLIGATIVAS PROBLEMA 1: A.- Las mezclas de benceno y tolueno se comportan casi idealmente. A 30º C las presiones de vapor de benceno y tolueno puros son 118,2 y 35,7 Torr respectivamente. Determine la presión parcial y la composición en equilibrio con una solución que consiste de 100 g de ambas sustancias. B.- Represente la variación de la presión de vapor de cada uno de los constituyentes de esta mezcla y de la presión total como función de la composición. C.- Si admite que ambas sustancias cumplen la regla de Trouton reconstruya el gráfico de temperatura de ebullición-composición. Las temperaturas de ebullición normales son 80,2º C y 110,6º C para benceno y tolueno respectivamente. PROBLEMA 2: A.- Las presiones de vapor de metano líquido a 173,2 y 153,2 K valen 25,7 y 11,84 atm respectivamente. Estime la solubilidad ideal del metano en cualquier solvente a 25º C. B.- A 300 K las presiones de vapor de equilibrio de HCl(g) sobre una solución muy diluída de GeCl4 (l) son: x HCl 0,005 0,012 0,019 p / kPa 32,0 76,9 121,8 Analice si este sistema obedece la ley de Henry en ese rango de concentraciones y estime la constante de Henry. C.- (1) Cuál es su significado físico que le asigna a KH? (2) La dependencia con la temperatura de la constante de Henry para el sistema anterior está dada por: ln KH / kPa = 12,147 - 1010 / T (K) ¿Qué información puede obtenerse a partir de esta ecuación? D.- Una planta para carbonatar agua de uso doméstico opera con CO2 a 10 atm. Estime la composición de la soda que se produce. ¿Qué información requiere para este cálculo? Busque esta información en tablas. E.- Las presiones de vapor de cloruro de metilo en equilibrio con una solución acuosa de esta sustancia a la temperatura de 25º C son: m/mol kg-1 0,029 0,051 0,106 0,131 p/torr 205,2 363,2 756,1 945,9 Calcule la constante de Henry, explicando el procedimiento. PROBLEMA 3: A.- La presión de vapor de una muestra de 300 g de benceno es de 400 torr a 60,6º C pero desciende a 386 torr cuando se le adicionan 19 g de un compuesto orgánico no volátil. Calcular su peso molecular. B.- ¿Cuál es el punto de congelación de 250 cm3 de agua azucarada que contiene 7,5 g de sacarosa (C12H22O11) aproximadamente 5 cucharadas de azúcar? C.- Estime el punto de congelación de 100 g de agua que contienen 2 g de NaCl. D.- Al estudiar soluciones acuosas de Th(NO3)4 se observó un descenso de la temperatura de fusión de 0,07031 para una concentración 0,009 molal. ¿Cuál es el número aparente de iones por moléculas de soluto? Para estos cálculos considere kc = 1,86 K/mol kg-1 . PROBLEMA 4: A.- Las temperatura de fusión y entalpía de fusión del Pb son 327º C y 5,2 kJ/mol respectivamente. Calcular la solubilidad ideal del Pb en Bi a 298º C. B.- La entalpía molar del naftaleno está dada por la expresión: H(T) = Ho + 4,8 T. Su temperatura de fusión es 80,2º C y su entalpía de fusión a esa temperatura es 4540 cal/mol. (1) Determine la solubilidad ideal de naftaleno en p-diclorobenceno (tf = 53,2º C) a 50º C. Este sistema presenta un eutéctico simple a 30,2º C y su composición es 0,394 en fracción molar de naftaleno. (2) Reconstruya el diagrama correspondiente y aplique la regla de las fases a cada situación de equilibrio de fases. PROBLEMA 5: A.- Se midió la presión osmótica de soluciones de polietileno en tolueno. La presión se expresa en términos de la altura del solvente. A 25º C se obtuvieron los siguientes resultados: c / g l-1 2,042 6,613 9,521 12,602 h / cm 0,592 1,910 2,750 3,600 Calcular la masa molecular relativa del polímero (densidad del tolueno 0,867 g cm-3 , densidad de las soluciones 1,004 g cm-3 ).
  • 24. SEMINARIOS DE FISICOQUIMICA –INGENIERÍA QUÍMICA (PRIMERA PARTE) 24 B.- La masa molecular relativa de una nueva enzima fue determinada disolviéndola en agua midiendo su presión osmótica en diferentes soluciones, extrapolando hasta concentración cero. Se obtuvieron los siguientes resultados: c / mg cm-3 3,221 4,618 5,112 6,722 h / cm 5,746 8,238 9,119 11, 9 ¿Cuál es la masa molecular de la enzima?
  • 25. SEMINARIOS DE FISICOQUIMICA –INGENIERÍA QUÍMICA (PRIMERA PARTE) 25 CÁTEDRA DE FISICOQUIMICA SEMINARIO 9 SOLUCIONES REALES-ACTIVIDAD PROBLEMA 1: En la siguiente tabla se dan las presiones de vapor (en KPa) de equilibrio y las composiciones del vapor y del liquido (en fracción molar) para el sistema tolueno (A)-metiletilacetona (B) a 303.13K xA 0.000 0.0898 0.2476 0.3577 0.5194 0.6036 0.7188 0.8019 0.9105 1.000 yA 0.000 0.0410 0.1154 0.1762 0.2772 0.3393 0.4450 0.5435 0.7284 1.000 pT/kPa 36.066 34.121 30.900 28.626 25.239 23.402 20.698 18.592 15.496 12.295 Considere el vapor como gas ideal. Calcule: (1) La presión parcial de cada componente en la mezcla y represéntela gráficamente. Estime la constante de Henry para cada componente. (2) Calcule el factor de actividad adoptando como estados de referencia (a) la ley de Raoult para cada componente (b) la ley de Raoult para el solvente y de Henry para el soluto. (3) Represente los potenciales químicos correspondientes y discuta el comportamiento de las pendientes en los extremos y en x=0.5. Analice las mismas sobre la base de la relación de Gibbs-Duhem. PROBLEMA 3: Para soluciones de n-propanol en agua se obtuvieron los siguientes valores de las presiones parciales de cada componente mol%(n-prop) 0 9.04 16.97 30.47 41.14 55.5 73.9 82.0 100 p agua/Torr 92 87.2 87.9 84.5 83.0 78.2 60.9 49.2 0.0 p n-prop/Torr 0 49.2 51.7 54.6 57.4 60.2 68.4 72.1 90 Calcule la actividad de cada componente empleando el estado de referencia (1) solvente o ley de Raoult para cada uno (2) solvente para el agua y soluto para el n-propanol. Estime la KH de n-propanol. PROBLEMA 4: Lewis y Storch midieron la presión de vapor de Br2 sobre CCl4 a muy bajas concentraciones. Los datos experimentales obtenidos a 25C fueron: 103 xBr2 3.94 4.20 5.99 10.2 13.0 23.6 23.8 25.0 pBr2/Torr 1.52 1.60 2.39 4.27 5.43 9.57 9.87 10.22 (1) Adoptando como estado de referencia la solución ideal diluida (ley de Henry), calcular la presión de vapor que le hubiera correspondido al bromo puro si la ley de Henry se hubiera cumplido en todo el intervalo de composiciones. (2) La presión de vapor del bromo puro a esa temperatura es 213 torr. ¿Cual es el carácter de la desviación del sistema respecto a la ley de Raoult? (3) Calcule la actividad del bromo puro tomando como estado de referencia (a) la ley de Raoult (b)la ley de Henry. Discuta los resultados. PROBLEMA 5: A.- (1) Las medidas de presión de vapor de Hg en amalgamas de talio a 325C pueden representarse por log p1/(p1 * x1)= -0.096/(1+0.263x1/x2)2 , donde p1 y p1 * son las presiones de vapor del Hg en las amalgamas y del Hg puro a esa temperatura, x1 es la fracción molar del Hg. Calcular la actividad del talio en estas soluciones si se elige como estado de referencia i) la solución diluida ii) la ley de Raoult. Discuta las diferencias. (2) Verificar que la variación de energía libre al diluir una amalgama de talio de 0.5 a 0.2 en talio es independiente del estado de referencia elegido. B.- A partir de medidas de FEM se ha determinado la actividad de Tl en amalgamas de Hg a 20C. En la siguiente tabla se presentan los resultados obtenidos para los factores de actividad del talio como soluto x2 0.0 0.005 0.01 0.05 0.1 0.2 0.3 0.4 0.5 f2 1.0 1.06 1.15 1.8 2.48 4.98 6.6 7.57 7.98 Calcular la actividad del Tl y del Hg a diferentes concentraciones. PROBLEMA 6: A.- Las presiones de vapor del metano liquido a 173.2 y 153.2K valen 25.7 y 11.84 atm respectivamente. Estime la solubilidad ideal del metano en cualquier solvente a 25C. B.- A 300K las presiones de vapor de equilibrio de HCl(g)sobre una solución muy diluida de GeCl4(l) son: xHCl 0.005 0.012 0.019 p/kPa 32.0 76.9 121.8 Muestre que este sistema obedece la ley de Henry en ese rango de concentraciones y calcule la constante de Henry. C.- Una planta para carbonatar agua de uso doméstico opera con CO2 a 10 atm. Estime la composición de la soda que se produce. ¿Que información requiere para este cálculo? D.- Las presiones de vapor de cloruro de metilo en equilibrio con una solución acuosa de esta sustancia a la temperatura de 25C son
  • 26. SEMINARIOS DE FISICOQUIMICA –INGENIERÍA QUÍMICA (PRIMERA PARTE) 26 m/mol kg-1 0.029 0.051 0.106 0.131 p/Torr 205.2 363.2 756.1 945.9 Calcule la constante de Henry. PROBLEMA 7: Determinación de los factores de actividad (El sistema es exactamente el estudiado en el laboratorio). Para mezclas de acetona- benceno se obtuvieron los siguientes datos de presión de vapor en función de la composición en acetona: xlíquido xvapor Ptotal(mmHg) xlíquido xvapor P(mmHg) xlíquido xvapor P(mmHg) 0.0 0.0000 95.3 0.4 0.6210 178.7 0.8 0.8640 213.1 0.1 0.3232 131.3 0.5 0.6810 188.3 0.9 0.9307 220.3 0.2 0.4661 152.5 0.6 0.7380 197.1 1.0 1.0000 229.2 0.3 0.5531 167.3 0.7 0.8007 205.3 A.- Construya el diagrama presión de vapor de acetona y de benceno como función de la fracción molar e indique que tipo de desviaciones presenta respecto de la ley de Raoult B.- Estimar el valor de la constante de Henry para ambos componentes usando la información de las soluciones 0,1 en acetona y 0,1 en benceno. C.- Los coeficientes de actividad para ambos componentes en las soluciones de xacetona = 1, 0,2; 0,4 ; 0,6; 0,8 y 1 (1) considerando el estado de referencia solvente (ley de Raoult) para ambos componentes (2) el estado de referencia solvente para la acetona y la solución real diluida como estado de referencia (ley de Henry) para el benceno (3) el estado de referencia solvente para el benceno y la solución real diluida como estado de referencia (ley de Henry) para la acetona. D.- representar el potencial químico (a menos de una constante) para ambos componentes E.- verificar que el potencial de la acetona pura es independiente del estado de referencia elegido F.- haciendo uso de la regla de Trouton y del conocimiento que la temperatura de ebullición de la acetona es 56,5°, estime su constante de Henry y el factor de actividad correspondiente de la acetona pura a esta temperatura (56,5°). PROBLEMA 8: Modelo de Van Laar. El modelo de Van Laar para soluciones regulares muestra que el factor de actividad puede representarse como ln k =  (1-xk)², donde  es un parámetro a ser evaluado a partir del análisis de los comportamientos límites de la presión de equilibrio (ver S. Glasstone, Tratado de Fisicoquímica o Termodinámica para Químicos para el tratamiento general. Se conoce también como ecuación de Margulles). (1) Halle las expresiones para las pendientes de las presiones totales para xi=0 y xi=1, (2) Establezca el vínculo entre estas pendientes con las constantes de Henry y las presiones de vapor. (3) Empleando la información del problema 1 del presente seminario, estime el valor del parámetro ß promedio. PROBLEMA 9: El modelo de van Laar (soluciones regulares) expresa el coeficiente de actividad de cada componente en una mezcla de moléculas de no-electrolitos de igual volumen molecular, en términos de las energías de interacción entre las distintas moléculas. En este modelo, siendo una hipótesis importante, se admite que la entropía por mol de mezcla en la solución real es la misma que la que debiera medirse al formar una solución ideal. Bajo estas condiciones, el potencial químico de cada componente se expresa como: 1 = 1 * + kT ln x1 + x2 2 u0 . Los distintos componentes se representan por los subíndices 1 y 2. El término energético u0 = 1,2 -1/2 (1,1 + 2,2 ) mide la extensión de la desviación del comportamiento predicho para una solución ideal en términos de la ley de Raoult. En esta igualdad, 1,2 , 1,1 y 2,2 son las energías de interacción de pares de moléculas; del tipo 1-2, 1-1 y 2-2. Una expresión similar vale para el componente 2. Considerando que el componente 1 está en equilibrio con su vapor y que éste se comporta como ideal: 1. Encuentre una expresión para la presión de vapor en función de la composición. 2. Grafique la presión de vapor en función de la composición para  = u0 /kT = -3, 0, 1, 3. 3. Analice que debe esperarse para las desviaciones cuando  > 2. Aplicar al sistema fenol-agua. 4. Represente en este caso, en forma cualitativa el comportamiento del potencial químico de uno de los componentes y verifique que existe una región donde el sistema no puede existir en dos fases en equilibrio (un líquido y un vapor), y que esta situación conduce a la separación de dos fases líquidas parcialmente miscibles en equilibrio con un mismo vapor.
  • 27. SEMINARIOS DE FISICOQUIMICA –INGENIERÍA QUÍMICA (PRIMERA PARTE) 27 PROBLEMA 11: Emplee la información de los problemas 7 y 9. 1.- Si admite como válida la aproximación de Van Laar, pk=xk.pk*.exp[(1-xk)2 ], estime el parámetro  y las constantes de Henry para estas sustancias a esta temperatura (25C) para una solución x =0,5. 2.- Reconstruya el gráfico de temperaturas de ebullición normales-composición para este sistema. La temperatura de ebullición de B es 79.6C. 3.- Encuentre la expresión entre la entalpía del líquido puro y la entalpía del líquido en el estado hipotético que se obtiene por extrapolación de la ley de Henry hasta xk=1. Analice como deben ser estas diferencias de entalpía para desviaciones positivas y negativas. Considere ideal al vapor y que el volumen de un líquido no se modifica apreciablemente ante cambios de presión. Recuerde la ecuación termodinámica de estado para el volumen.
  • 28. SEMINARIOS DE FISICOQUIMICA –INGENIERÍA QUÍMICA (PRIMERA PARTE) 28 CATEDRA DE FISICOQUIMICA I SEMINARIO 9 EQUILIBRIO QUIMICO PROBLEMA 1: A.- Cual de los siguientes diagramas de G y H como función de T satisface el teorema del calor de Nerst? Justifique. H H G G T/K T/K B.- Indique si este enunciado del tercer principio de la termodinámica es correcto: En el 0 K de temperatura la entropía de cualquier sustancia es cero. Justifique. PROBLEMA 2: A.- Se ha determinado la Cp  (en J/K mol) del Pb(s) entre 10 y 298 K. Los resultados obtenidos son los siguientes: T/K 10 15 20 25 30 50 70 100 150 200 250 298 Cp  2.8 7.0 10.8 14.1 16.5 21.5 23.3 24.5 25.3 25.8 26.2 26.6 Calcule la S del Pb a 298 K. Para la integración por debajo de 10 K emplee la ecuación de Debye a bajas temperaturas para Cv. B.- Se ha medido la Cp  (en J/K mol) de la sal K4Fe(CN)6. Los datos obtenidos son: T/K 0 10 20 30 40 50 60 70 80 90 100 120 130 140 Cp  0 2.09 14.43 34.44 62.55 87.03 111 131,4 149,4 165,3 179.6 205 216,6 227,3 T/K 150 160 170 180 190 200 60 Cp  237.6 247.3 256.5 265.1 273 287.03 280.3 Cual es la entropía de esta sal a 200 K y 1 atm? PROBLEMA 3: En el estudio termodinámico de N2 se ha obtenido la siguiente información a partir de datos térmicos 1 1 0 . . 2 , 27 ) (      mol K J dT T C T S t T p t Ttrans= 35.61 K, Ht=0.229 kJ.mol-1 1 1 . 4 , 23 ) ( ) (       mol K J dT T C T S T S f t T T p t f Tfus= 63.14 K, Hf= 0.721 kJ/mol 1 1 . 4 , 11 ) ( ) (       mol K J dT T C T S T S e f T T p f e Teb= 77.32 K, Hv= 5.58 kJ/mol (1) ¿Cual es la entropía del N2 a 77.32 K? (2) ¿Cual es la entropía del N2 a 25C y 1 atm de presión? Emplee la capacidad calorífica de tablas para este cálculo. (3) ¿Cual es el aumento de energía libre que experimenta el N2 si se lo sometiera a una expansión adiabática reversible entre 25 y -25C. Suponga comportamiento ideal del gas. PROBLEMA 4: Se ha estudiado la dependencia de la Cp del NF3 desde 12.85 K hasta 140 K. T/K 12.95 15 20 25 30 35 40 45 55 58.41 60 65 70 75 80 Cp 2.154 3.077 5.031 6.719 8.1 9.193 10.12 10.99 11.95 16.62 16.53 16.62 17.27 17.17 17.07 T/K 85 90 95 100 105 110 115 120 125 130 135 140 Cp 16,98 16,9 16,82 16,75 16,71 16,69 16,7 16,74 16,81 16,93 17,08 17,26 Se conoce que los cambios de entalpía en los distintos cambios de fase son: Cambio de fase T/K H/Kcal mol-1 transición cristalina 56,62 381,8 fusión 66,37 95,1 ebullición 144,15 2769,0 G H H G
  • 29. SEMINARIOS DE FISICOQUIMICA –INGENIERÍA QUÍMICA (PRIMERA PARTE) 29 La Cv esta dada por la ecuación de Debye Cv=(48/5).R..(T/)3 , donde  es la temperatura caracteristica de Debye para este solido (=120.8 K). Evalúe gráficamente la entropía absoluta del NF3 a 140 K. PROBLEMA 5: A.- Una mezcla constituida inicialmente por 3 partes de H2 y 1 de N2 se deja reacción ar a 200C y 1 atm. En estas condición es se determinó la formación de 15.3% en volumen de NH3. Calcular Kp, Kx y G. Si a esta mezcla se le introduce He hasta que la presión se eleva a 2 atm, calcular KX. C.- Grado de avance . Sea la reacción: N2 (g) + 3 H2 (g)  2 NH3 (g). Si inicialmente hay 10 moles de N2 en un exceso de H2 diga cuantos moles de cada sustancia han reaccionado o se han formado cuando  = 0,1. C.- Consideremos la reacción 2A(g) + B2(g)  A2B2(g), en la que se supone comportamiento ideal de las especies. Se parte de 1 mol de A y 1 mol de B2 y 0,5 moles de A2B2 y se observa que el equilibrio se establece cuando el grado de avance es 0,1. Calcule las fracciones molares en el equilibrio y si la presión total es de 1 atm, calcular la constante de equilibrio en términos de las fracciones molares y la constante de equilibrio Kp. La presión de trabajo es 2 atm. D.- analice qué papel juega la energía libre de mezcla en la posición del equilibrio químico. PROBLEMA 6: B.- Al calentar vapores de PCl5 se descompone parcialmente en PCl3(g) y Cl2(g). A 1 atm de presión y 230C, la densidad de la mezcla es 4.8 g/l. Calcular Kp, Kx y G. C.- Para la reacción Cl2(g) 2 Cl(g), a 1000 K y 1 atm, Kp= 2.45 10-7 , cuando las presiones se dan en atm. Calcular las composiciones en equilibrio en estas condiciones. D.- Para la reacción de Cl2(g) + CO(g) Cl2CO(g) (fosgeno), se ha determinado que G= -48.77 Kcal/mol a 25C. Calcular Kp y Kx. E.- Para esta reacción, G/cal.mol-1 =-24100 + 4 T lnT + 3.5 T Calcular la presión parcial de Cl2(g), H y S a 200 C y 1 atm. Encuentre CP a 200C y 1 atm, y una función de la temperatura para este. PROBLEMA 7: A.- Se ha medido el volumen ocupado por N2O4 en equilibrio con NO2 a 27C y 111C a P = 1 atm, siendo 2.95 l y 6.07 l respectivamente. El sistema se ha preparado agregando 9.2 g de N2O4. Sobre la base de esta información calcular (1) el grado de disociación ,  rH,  rG (2) temperatura a la cual se igualan las presiones de dióxido y tetróxido a la unidad (3) Composición a 111C (4) la presión a la cual =0.5 a 111C. B.- Para la reacción de formación de H2O(g) se ha estudiado la dependencia de la constante de equilibrio con la temperatura. Los datos obtenidos son: T/K 1000 1200 1400 1500 1750 2000 lg Kp 10.059 7.893 6.34 5.716 4.365 3.528 Además se dispone de la siguiente información sobre las capacidades caloríficas O2(g) Cp/cal K-1 mol-1 = 8.27 + 2.58 10-4 T - 1.877 105 T-2 (Válido entre 273 y 5000 K) H2(g) Cp/cal K-1 mol-1 = 6.65 + 6.9 10-4 T (Válido entre 273 y 3500 K) H2O(g) Cp/cal K-1 mol-1 = 7.2 + 2.7 10-3 T (Válido entre 273 y 2000 K) (1) Obtener una expresión para Kp como función T (2) calcular la entropía de la reacción a 298 K y 1 atm (3) el aumento de entropía a 298 K y 1 atm (4) la composición del sistema a 1 y 2 atm, resolviendo numérica y gráficamente. PROBLEMA 8: Cuando el NH4Cl se calienta su presión de vapor aumenta de 427C a 459C, la presión de vapor se incrementa de 608 kPa a 1115 kPa. Calcular (1) Kp a 427C y el aumento de la función de Gibbs en condiciones estándar (2) los aumentos de entropía y entalpía a esa temperatura. PROBLEMA 9: Cálculo de entropías de sustancias puras. a.- Ecuación de Debye para Cv a bajas temperaturas. Debye demostró que, a bajas temperaturas, la capacidad calorífica a volumen cte. puede escribirse: CV = 464,6 (T/)3 , donde  es un parámetro propio de cada sólido cristalino conocida como su temperatura de Debye y su valor puede consultarse en tablas. Para el Ar,  = 84 K. (a) Calcule CV del Ar a 10K. (b) Con este valor calcule la entropía del Ar a 10K. (c) Calcule el aumento de entropía que debiera observarse si un mol de un sólido cristalino se calienta desde 0 hasta 15 K a 1 atm de presión. (d) Analice porque puede utilizar Cv en lugar de Cp. b.- Cálculo de entropías a partir de datos térmicos En el estudio termodinámico de N2 se ha obtenido la siguiente información a partir de datos térmicos
  • 30. SEMINARIOS DE FISICOQUIMICA –INGENIERÍA QUÍMICA (PRIMERA PARTE) 30 1 1 1 1 1 1 1 1 1 0 . 580 . 5 32 . 77 . 4 . 11 ln . . 271 . 0 14 . 63 . 4 . 23 ln . . 229 . 0 61 . 35 . 2 . 27 ln .                         mol kJ H K T mol K J T d C mol kJ H K T mol K J T d C mol kJ H K T mol K J T d C m f f p T Tf m f f p T Tt m t t p T e f t a.- ¿Cual es la entropía del N2 líquido y del N2 gaseoso a 77.32 K? b.- Calcule es la entropía del N2(g) a 25C y 1 atm de presión (i) conociendo la dependencia experimental 2 6 3 1 1 10 50 . 0 . 10 77 . 3 58 . 28 / T x T x mol JK Cp       (ii) recurriendo al principio de equipartición para estimar Cp. c.- ¿Cual es el aumento de energía libre que experimenta el N2(g) si se lo sometiera a una expansión adiabática reversible entre 25 y -25C. Suponga comportamiento ideal del gas. PROBLEMA 9: A.- Considere la reacción N2O4(g)  2 NO2(g). En la siguiente tabla se presenta información termodinámica para estas sustancias a 298 K y 1 atm. sustancia fH /kJ.mol-1 S /J.K-1 mol-1 Cp  /J.K-1 mol-1 (1) Calcule la constante de equilibrio a 298 K y a 398 K. (2) determine la composición del sistema a 398 K y 1 atm. N2O4(g) 11,1 304,4 79,2 NO2(g) 33,2 240,1 37,2 B.- Empleando la información que se indica en la tabla siguiente y la contenida en la previa, evalúe las energías libres de formación de las especies involucradas en la reacción de este problema a 298 y a 373 K. En este caso evalúe la constante de equilibrio empleando las energías libres de formación. sustancia fH /kJ.mol-1 S /J.K-1 mol-1 Cp  /J.K-1 mol-1 N2(g) -- 191,6 29,1 O2(g) -- 205,2 29,4 PROBLEMA 10: Se ha estudiado en fase gaseosa la producción de ciclohexano por hidrogenación del benceno. Las entalpías de formación y las entropías en estado estándar a 298 K se muestran en la siguiente tabla. Por su parte, en esta tabla se indican los coeficientes a, b y c que expresan la dependencia de la capacidad calorífica estándar de cada sustancia función de la temperatura. Estos coeficientes son válidos en el intervalo de temperatura entre 25 y 525°C. Sustancia fHm  Sm  Cp,m/J.K-1 .mol-1 = a + b.T + c.T2 kJ.mol-1 J.K-1 .mol-1 a b 104 c Benceno 82,93 269,03 -14,81 0,378 -1,52 Ciclohexeno -123,13 298,32 52,13 0,599 -2,30 Hidrógeno 0 130,66 27,12 0,0038 0 Con la información precedente estudie las siguientes situaciones y analice los alcances de las distintas consideraciones que se emplean en cálculos termodinámicos de esta naturaleza. (1) Calcule las capacidades caloríficas a 298 K empleando la dependencia de esta propiedad con la temperatura que se indica en la Tabla. Con esta información Calcule los aumentos de entalpía (fHm  ) de entropía (rSm  ) y de energía libre (rGm  ) correspondientes y evalúe la constante de equilibrio a 400 C suponiendo que Cp  no es función de T. (2) Evalúe los aumentos de entalpía (fHm  ) de entropía (rSm  ) y de energía libre (rGm  ) correspondientes y evalúe la constante de equilibrio a 400 C considerando la dependencia de Cp,m indicadas en la tabla para cada sustancia. Compare estos resultados con los obtenidos en el punto previo. Calcule el error relativo que se comete al suponer que las capacidades caloríficas no dependen de la temperatura. (3) Calcule la capacidad calorífica promedio <Cp,m  > en el intervalo de temperatura entre 25 y 525C. Con esta información calcule los aumentos de entalpía (fHm  ) de entropía (rSm  ) y de energía libre (rGm  ) correspondientes y evalúe la constante de equilibrio a 400 C empleando este valor promedio. Compare estos
  • 31. SEMINARIOS DE FISICOQUIMICA –INGENIERÍA QUÍMICA (PRIMERA PARTE) 31 resultados con los obtenidos en el punto previo. Calcule el error relativo que se comete al suponer que las capacidades caloríficas no dependen de la temperatura. PROBLEMA 11: A.- En el intervalo de temperatura de 450 a 715 K, los valores de la presión de equilibrio de H2(g) sobre -U(s) y UH3(s) puede representarse mediante ln p(H2)= 69.32 - 14.64 103 /T - 5.65 ln T, donde p esta en Pa y T en K. Encontrar una expresión de la entalpía estándar de formación del -UH3(s) como función de T y a partir de ella calcule Cp. B.- En el intervalo de temperatura de 350 a 470 K el aumento de entalpía standard para la disociación de CaCl2.NH3(s) en NH3(g) y CaCl2(s) es constante e igual a 78.15 kJ/mol. La presión de disociación de equilibrio del NH3(g) es de 12.8 torr a 400 K. Hallar una expresión para la función energía libre estándar de Gibbs como función de T. PROBLEMA 12: Se sabe que Gf  para la reacción: H2 (g) + (1/2) O2 (g)  H2O (g) es Gf  = -54,507 kcal mol-1 . Se desea calcular Gf  de la reacción: H2 (g) + (1/2) O2 (g)  H2O (l) y con ella su constante de equilibrio a 298.16K. La presión de vapor del agua a 298,16K es 23,8 Torr. PROBLEMA 13: Encuentre una expresión para la constante de equilibrio para la siguiente reacción: CO2 (g) + 2 NH3 (g) = CO(NH2)2(solución)+ H2O(l) a partir de la igualdad de los respectivos potenciales químicos. Indique que estado de referencia elije para evaluar la actividad de cada substancia y cuales son los estados estándares que le corresponden. Datos: Gf  (CO2 , g) = -94,26 kcal mol-1 Gf  (CO(NH2)2 , solución) = -48.720 kcal mol-1 Gf  (NH3 , g) = -3,976 kcal mol-1 Gf  (H2O , líquido.) = -56,69 kcal mol-1 Calcule la constante de equilibrio. PROBLEMA 14: Coeficiente de reparto de una especie entre dos líquidos/solvente. Demuestre que el coeficiente de partición de un soluto entre dos solventes distintos es, en soluciones diluidas, igual a la relación entre las constantes de Henry.
  • 32. SEMINARIOS DE FISICOQUIMICA –INGENIERÍA QUÍMICA (PRIMERA PARTE) 32 FISICOQUIMICA I SEMINARIOS SOBRE TEMAS RELACIONADOS CON LOS TRABAJOS PRACTICOS ELECTROQUIMICA PROBLEMA N° 1 En la medida de la fuerza electromotriz de una pila, se aplicó el método de Poggendorff o de oposición. En el trabajo práctico correspondiente se ha representado el circuito básico del potenciómetro Student. Este instrumento consta de 16 resistencias calibradas en serie, de 10 ohms cada una, de manera que la resistencia total sea de 160 ohms. Estas resistencias están agrupadas de manera tal que una de ellas pueda variarse en forma contínua entre 0 y 10 ohms. Las 15 restantes se varían en pasos de 10 ohms. Durante su calibración, debe asegurarse que por el instrumento circules 10 mA. De esta manera la caída total de tensión entre la entrada y la salida corresponden siempre a 1,6 voltios. (1) Esquematice el sistema de resistencias internas indicado en la figura antes mencionada y señale en una escala adecuada cuál es la resistencia a intercalar para que la caída de tensión en las condiciones de medida sea de 1,2 voltios. (2) En la calibración usted ha intercalado dos resistencias externas en paralelo. Si la FEM de la pila patrón vale 1,0185 voltios, ¿cuál es el valor de la resistencia equivalente que debe intercalar para asegurar que la caída de tensión sobre 101,85 ohms del potenciómetro correspondan a la FEM deseada? Suponga que su batería tiene una FEM de 2 voltios. (3) Con el valor de la resistencia equivalente hallado en el inciso anterior estime cuál será la menor variación que puede obtener en la resistencia equivalente si en una de ellas ajusta un valor de 250 ohms. Para problemas tipo del circuito de oposición elemental puede consultar cualquier texto sobre electricidad que haya usado en los cursos de física. PROBLEMA N° 2 Considere el sistema electroquímico T / M / S / M' / T' , donde T y T' representan las terminales eléctricas de la pila y cuya naturaleza debe ser exactamente la misma con la que se construye el equipo potenciométrico (por ejemplo Cu). Las barras indican una interfase entre las distintas partes del sistema, M y M' son dos metales determinados y S es la solución, que aquí se ha simplificado al no indicar su naturaleza. Se llama fuerza electromotriz de la pila a la diferencia de potencial medida entre los terminales T cuando no existe circulación de corriente eléctrica. (1) Justifique desde el punto de vista termodinámico por qué no debe circular corriente eléctrica al medir la FEM de la pila. (2) Justifique por qué T y T' deben ser del mismo material con el cual se construye el potenciómetro. PROBLEMA N°3 A.- Considere un metal M que se sumerge en una solución de sus iones Mz+; si a+ es la actividad de estos iones en la solución y aM la correspondiente a los iones en el retículo metálico, y por ejemplo a+ > aM entonces ocurrirá espontáneamente el siguiente proceso: Mz+ + ze-  M. Cada vez que un ion se incorpora a la red del metal, este aumenta su potencial eléctrico respecto del de la solución, es decir, se hace positivo respecto a la solución. El proceso continúa hasta que la diferencia de potencial en la interfaz crece lo suficiente para oponerse a la tendencia espontánea originada en la diferencia de actividades (potenciales químicos) entre los iones en la solución y la red metálica. (1) Muestre que la condición de equilibrio se establece cuando se igualan los potenciales electroquímicos i de las especies consideradas, definidos en términos generales como    F zk k k   donde  es el potencial eléctrico de la fase considerada. (2) Encuentre que la diferencia de potencial metal solución (M - S), para el proceso considerado en este enunciado puede escribirse como:  = (M - S) = (1/zF) (+  + z e(M) - M  ) + (RT/zF) ln (a+ / aM) donde los k  corresponden a los potenciales químicos de los iones en la solución, en la red y de los electrones en estado estándar (actividad unitaria a la presión de una atmósfera). B.- Sobre la base de la expresión deducida anteriormente encuentre las expresiones para los potenciales de electrodo correspondientes a los siguientes sistemas: (1) electrodo de primera especie M / M2+ por ejemplo Cu / Cu++(a+) (2) electrodo gaseoso, por ejemplo Pt / H2 (g, p) / H+ (a+) (3) electrodo de segunda especie, por ejemplo Hg / Hg2Cl2(s) / Cl- (a-) (4) electrodo redox o de tercera especie (5) electrodo de amalgama.
  • 33. SEMINARIOS DE FISICOQUIMICA –INGENIERÍA QUÍMICA (PRIMERA PARTE) 33 C.- Calcule el cambio de la diferencia de potencial a 25°C entre un electrodo de plata / cloruro de plata y una solución saturada de cloruro de plata, cuando se agrega una solución 0,01 M de KCl. Considere el KpsAgCl = 1,0 10-10 a 25°C. PROBLEMA N°4 A.- Si (T), (T'), (M), (M') y (S) son los potenciales eléctricos de cada una de las fases y además ED y EI son los potenciales de electrodo correspondiente a los electrodos de la derecha (D) e izquierda (I), entonces la FEM de la pila está dada por FEM = ED - EI . Encuentre una expresión general para esta diferencia en términos de los potenciales eléctricos anteriormente mencionados. Discuta la convención de signos. B.- Encuentre la expresión de la FEM de la pila de Daniell. C.- Cuando se enfrenta el electrodo de Ag/AgCl(s)/Cl(a-) contra el electrodo normal de H2 en un experimento hipotético con todas las especies en estado estándar, se lee una FEM de 0,2224 V a la temperatura de trabajo. Se observa que la corriente eléctrica se mueve desde el electrodo de Ag/AgCl(s)/Cl(a-) al ENH. Escriba la pila e indique cuánto vale el potencial normal del electrodo en cuestión. Justifique el procedimiento y represente la pila. D.- Se enfrenta el electrodo de Pb(s)/PbSO4(s)/SO4 -2 (a_ ) contra el ENH en un experimento hipotético en condiciones estándar. La corriente eléctrica fluye desde el ENH al electrodo en cuestión. Si la FEM que se lee en este experimento vale 0,36 V, halle el potencial normal del electrodo Pb(s)/PbSO4(s)/SO4 -2 (a_ ). Justifique el procedimiento y represente la pila. PROBLEMA N°5 A.- Defina actividad iónica, media y actividad del electrolito para los ejemplos (i) HCl (ii) BaCl2, (iii) ZnSO4 (iv) FeCl3. Para estudiar el temario que sigue puede consultarse la siguiente literatura: Lewis & Randal (2° ed.): Termodynamics; Guggenheim & Prue, Physicochemical Calculations y la literatura general indicada para el curso. B.- La actividad de un electrolito puede evaluarse por distintos procedimientos. En el caso de las soluciones de HCl, las medidas pueden realizarse determinando las presiones de vapor, medidas de FEM de pilas del tipo Pt / H2(g , 1 atm) / HCl(m) / AgCl / Ag o a partir del descenso crioscópico. El objetivo de este problema es la familiarización del estudiante con estos tres procedimientos. (1) A partir de medidas de descenso crioscópico Tf En el texto de Glasstone (Tratado de Química Física, Capítulos IX y XII) y en el texto de Castellan (Fisicoquímica) puede consultarse las deducciones de las expresiones que permiten evaluar coeficientes de actividad media de electrolitos a partir del descenso crioscópico. A partir de la siguiente información evaluar los coeficientes de actividad de las soluciones 0,3 molal (m) y 1 m. m/(moles/Kg) 0,001 0,002 0,005 0,010 0,020 0,050 Tc103 / K 3,675 7,318 18,152 36,028 71,43 176,66 m/(moles/Kg) 0,100 0,200 0,300 0,500 0,700 1,000 Tc103 / K 325,09 706,4 1068,9 1822,5 2580,1 351 La constante crioscópica del agua es kc = 1,858 K moles-1Kg (2) A partir de medidas de presión de vapor de soluciones de HCl m/ moles/Kg 4 5 6 7 8 9 10 p 104 / atm 0,235 0,6974 1,842 4,579 11,10 23,39 55,26 ± 1,74 2,38 3,23 4,36 5,95 7,97 10,61 ± es el factor de actividad medio de esas soluciones. Verifique que en soluciones de electrolitos 1-1 valentes como HCl, la presión de vapor debe depender del cuadrado de la molalidad en el límite de bajas concentraciones, es decir lim m 0 (p/m2) = kH, donde kH es la constante de Henry correspondiente. Determinar la constante de Henry y evaluar el factor de actividad medio de soluciones 5,5 y 8,5 m de HCl. (3) A partir de medidas de FEM de la pila indicada anteriormente: m 0,1238 0,05391 0,02563 0,01341 0,09138 0,005619 0,003215 FEM 0,34199 0,38222 0,41824 0,44974 0,46860 0,49257 0,52053 En la tabla anterior m se da en moles de HCl por 1000 g de solvente y la FEM en voltios.